You are on page 1of 533

For a triangle of given three sides, say a, b, and c, the formula for the area is given by

where s is the semi perimeter equal P = (a + b + c).



Derivation of Heron's Formula
Area of triangle ABC
equation (1)

From triangle ADB




From triangle CDB




Substitute the values of x and x
2




Square both sides

























note: P = perimeter














Substitute h to equation (1)






Recall that P = s. Thus,


Mensuration is the branch of mathematics which deals with the study of Geometric shapes ,
Their area , Volume and different parameters in geometric objects.
Some important mensuration formulas are:
1. Area of rectangle (A) = length(l) * Breath(b)
2. Perimeter of a rectangle (P) = 2 * (Length(l) + Breath(b))
3. Area of a square (A) = Length (l) * Length (l)
4. Perimeter of a square (P) = 4 * Length (l)
5. Area of a parallelogram(A) = Length(l) * Height(h)













6. Perimeter of a parallelogram (P) = 2 * (length(l) + Breadth(b))
7. Area of a triangle (A) = (Base(b) * Height(b)) / 2

And for a triangle with sides measuring a , b and c , Perimeter = a+b+c
and s = semi perimeter = perimeter / 2 = (a+b+c)/2
And also . Area of triangle =
This formulas is also knows as Heroes formula.
8. Area of triangle(A) =
Where , A , B and C are the vertex and angle A , B , C are respective angles of triangles and a ,
b , c are the respective opposite sides of the angles as shown in figure below:













area of triangle - mensuration
9. Area of isosceles triangle =
Where , a = length of two equal side , b= length of base of isosceles triangle.
10. Area of trapezium (A) =
Where , a and b are the length of parallel sides and h is the perpendicular distance between
a and b .

11. Perimeter of a trapezium (P) = sum of all sides
12. Area f rhombus (A) = Product of diagonals / 2












13. Perimeter of a rhombus (P) = 4 * l
where l = length of a side
14. Area of quadrilateral (A) = 1/2 * Diagonal * (Sum of offsets)

15. Area of a Kite (A) = 1/2 * product of its diagonals
16. Perimeter of a Kite (A) = 2 * Sum on non-adjacent sides
17. Area of a Circle (A) =
Where , r= radius of the circle and d= diameter of the circle.
18. Circumference of a Circle =
r= radius of circle
d= diameter of circle
19. Total surface area of
cuboid =
where , l= length , b=breadth , h=height












20. Total surface area of
cuboid =
where , l= length
21. length of diagonal of
cuboid =
22. length of diagonal of
cube =
23. Volume of cuboid = l * b * h
24. Volume of cube = l * l* l
25. Area of base of a cone =
26. Curved surface area of a cone =C
Where , r = radius of base , l = slanting height of cone
27. Total surface area of a cone =
28. Volume of right circular cone =
Where , r = radius of base of cone , h= height of the cone (perpendicular to base)
29. Surface area of triangular prism = (P * height) + (2 * area of triangle)
Where , p = perimeter of base












30. Surface area of polygonal prism = (Perimeter of base * height ) + (Area of polygonal base *
2)
31. Lateral surface area of prism = Perimeter of base * height
32. Volume of Triangular prism = Area of the triangular base * height
33. Curved surface area of a cylinder =
Where , r = radius of base , h = height of cylinder
34. Total surface area of a cylinder =
35. Volume of a cylinder =
36. Surface area of sphere =
where , r= radius of sphere , d= diameter of sphere
37. Volume of a sphere =
38. Volume of hollow cylinder =
where , R = radius of cylinder , r= radius of hollow , h = height of cylinder
39. Surface area of a right square pyramid =
Where , a = length of base , b= length of equal side ; of the isosceles triangle forming the
slanting face.
40. Volume of a right square pyramid =












41. Area of a regular hexagon =
42. area of equilateral triangle =
43. Curved surface area of a Frustums =
44. Total surface area of a Frustums =
45. Curved surface area of a Hemisphere =
46. Total surface area of a Hemisphere =
47. Volume of a Hemisphere =
48. Area of sector of a circle =
where , = measure of angle of the sector , r= radius of the sector





Total Surface Area of a Cone



















Note:
A cone does not have uniform (or congruent) cross-sections.















Example 31

Solution:

























Area of a Rhombus
A rhombus with diagonals x units and y units has an
area of A square units given by the formula















Example 17
Find the area of a rhombus whose diagonals measure 12 cm and 18 cm.
Solution:

























Area of a Trapezium
A trapezium whose parallel sides are of length a units
and b units and the perpendicular distance between
them is h units has an area of A square units given by
the formula



Example 18
Find the area of a trapezium whose parallel sides are 10 cm and 8 cm where the perpendicular
distance between the sides is 4 cm.
Solution:


























Area of a Composite Figure
A figure obtained from combining two different figures is called a composite figure and its area, A, is
found by adding together the areas of the individual figures.



Example 21
Find the area of the following composite figure.













Solution:






Total Surface Area of a Cuboid












Recall:
The total surface area (TSA) of a cuboid is the sum of the areas of its six faces. That is:



Example 27
Find the total surface area of a cuboid with dimensions 8 cm by 6 cm by 5 cm.
Solution:






























Surface Area of a Sphere
A sphere is a body bounded by a surface whose every point is equidistant (i.e. the same
distance) from a fixed point, called the centre. For example, a shot (a heavy iron ball) is a solid
sphere and a tennis ball is a hollow sphere.

One-half of a sphere is called a hemisphere.

Archimedes discovered that a cylinder that circumscribes a sphere, as shown in the following
diagram, has a curved surface area equal to the surface area, S, of the sphere.















Example 32

Solution:

















Example 33

Solution:




Note:
We often use a circle to represent a sphere. The three-dimensional figure of a sphere is drawn by
a circle and an ellipse as illustrated above.

Curved Surface Area of a Cylinder












Recall:




Example 28
Find the area of the curved surface of a cylindrical tin with radius 8 cm and height 10 cm.
Solution:















Note:
The circular base of the cylinder is drawn as an ellipse.






Volume


Volume of a Solid or Prism
A solid with parallel sides is called a prism. For example, solids such as a cube or cylinder are
prisms.


Recall:
The volumes of the following solids are often required to solve real world problems involving
quantity, capacity, mass and strength of materials including liquids.

















Volume of a Cube
Example 34
Find the volume of a cube of side-length 5.2 cm.
Solution:
















Key Terms
volume, cubic units, prism, volume of a solid





Capacity
Capacity is the volume contained in a vessel; e.g. in a hot water tank or petrol tank. Capacity refers to
the amount of liquid a vessel can hold, and it is often measured in litres (L) or millilitres (mL).














Note:


Example 38
A hot water tank with a square base of side 1.20 m contains 3600 litres of water. What is the
depth of the water in centimetres?
Solution:
Let h be the depth in cm.


So, the depth of the water is 250 cm.


















Volume of a Pyramid
The volume, V, of a pyramid in cubic units is given by

where A is the area of the base and h is the height of the pyramid.

Volume of a Square-based Pyramid

The volume of a square-based pyramid is given by


Example 39
A pyramid has a square base of side 4 cm and a height of 9 cm. Find its volume.
Solution:
















Volume of a Rectangular-based Pyramid

The volume of a rectangular-based pyramid is given by


Example 40
Find the volume of a rectangular-based pyramid whose base is 8 cm by 6 cm and height is 5 cm.
Solution:
















Volume of a Triangular Pyramid
The volume of a triangular pyramid is given by














Example 41
Find the volume of the following triangular pyramid, rounding your answer to two decimal
places.

Solution:


























Volume of a Cone
The volume of a cone is given by















Example 42

Solution:












Volume of a Sphere
If four points on the surface of a sphere are joined to the centre of the sphere, then a pyramid of
perpendicular height r is formed, as shown in the diagram. Consider the solid sphere to be built












with a large number of such solid pyramids that have a very small base which represents a small
portion of the surface area of a sphere.


















Example 43
Find the volume of a sphere of radius 9.6 m, rounding your answer to two decimal places.
Solution:



Volume of a Cuboid












Example 35
Find the volume of a brick with dimensions 20 cm by 15 cm by 10 cm.
Solution:








Volume of a Triangular Prism












Example 37
The cross-section of a prism is a horizontal triangle whose sides are 6 cm, 8 cm and 10 cm. The
vertical height of the triangular prism is 12 cm. Calculate the volume of the prism.
Solution:
















Problem Solving
Example 44
The length of a cuboid is 25 cm, its width is 21 cm, and its volume is 3675 cm. Find its height.
Solution:


So, the height of the cuboid is 7 cm.



Example 45
How many 6 cm cubes will fit into a rectangular box whose internal measurements are 60 cm
by 54 cm by 42 cm?
Solution:




















Example 46
A cylindrical tank of diameter 7 m and depth 4 m is full of water. The water is drained into a
rectangular prism 11 m long, 7 m wide and 3 m high, as shown in the following diagram.














Solution:
For the cylinder:


Let the height of the water in the prism be h m.

So, the water will rise to a height of 2 m in the prism.


Problem Solving Unit
Problem 14.1 A Ladder
















Problem 14.2 A Speedboat
Five equally spaced buoys represent the direct passage across Lake Theta from the Stanford
to the Fairview landing.
Outboard and Speedboat depart from Stanford in their respective boats to go to Fairview.
Outboard advances direct whereas Speedboat needs to stop at the Cranfield landing to refuel.
Outboard just passes the second buoy as Speedboat stops at Cranfield. Outboard is passing
the third buoy as Speedboat leaves Cranfield.
They landed at Fairview simultaneously.














a. How far is Cranfield from Stanford?
b. How far is Fairview from Cranfield?

Problem 14.3 The Crease
How long is the crease that is made by folding a 15 cm by 8 cm piece of paper so that two
opposite corners of the papers coincide?

Problem 14.4 A Prism
A rectangular prism's surface area is equal to 324 cm
2
. If its length and width are 12 cm and
8 cm respectively, find the prism's height.

Problem 14.5 Christmas Cards
Determine the maximum number of 20 cm by 15 cm Christmas cards you can make from a 3
m by 2 m cardboard sheet, if each card has four pages (i.e. 2 leaves).













Problem 14.6 A Car's Engine
A car's engine has 6 cylinders, each of diameter 9.6 cm and height 10.4 cm. Find the capacity
of the engine's cylinders in cc (cubic centimetres).

Problem 14.7 A Closed Cube
A cube's volume is 125 cm
3
. Find the cube's side-length and hence its surface area.

Problem 14.8 Capacity of an Engine
A car's engine has 4 cylinders, with a total capacity of 2000 cc.
a. If the diameter of each cylinder is 9 cm, calculate the height of each cylinder.
b. If the height of each cylinder is 8 cm, calculate the diameter of each cylinder.

Problem 14.9 Capacity of a Pool
A rectangular swimming pool's depth is 1 m at the shallow end and gradually increases to 4
m at the deep end, as shown in the following diagram.













Calculate:
a. the cost of painting the pool at $5 per square metre
b. the amount of water (measured in litres) required to fill the pool

Problem 14.10 Capacity of a Pool
A swimming pool has a length of 50 m and a width of 36 m. The pool's depth is 1 m at the
shallow end and gradually increases to 2.5 m at the deep end. Calculate the capacity of the
pool in litres.

Problem 14.11 A Steel Pipe
The cross-section of a 12 m long steel pipe is an annulus with inner and outer radius of 14 cm
and 21 cm respectively. Calculate the volume of metal used to make the pipe.

Problem 14.12 A Square Sheet
A square sheet of metal 24 cm by 24 cm has a square piece cut out from each corner. The
remaining piece is folded to form a cube with an open top.

Develop a formula that can be used to calculate the volume of the cube.

Problem 14.13 Water Tank
Half of a 7 m by 6 m by 2 m rectangular prism is filled with water. The water is allowed to












flow out at the base of the prism into a cylindrical tank of diameter 3.2 m. Calculate the
height to which the water will rise in the tank.

Problem 14.14 Slicing a Cube
1. Can you obtain a cross-section of an equilateral triangle by slicing a cube? Justify your
answer by stating the reasons.

2. Can you obtain a cross-section of a regular hexagon by slicing a cube? Justify your
answer by stating the reasons.
3. Can you obtain a cross-section of a regular pentagon by slicing a cube? Justify your
answer by stating the reasons.

Problem 14.15 Two Spheres


Problem 14.16 Shoemaker's Knife
A shoemaker's knife consists of a large semicircle and two small semicircles with diameters
that add up to that of the large semicircle, as shown in the following diagram.













1. Use a compass to draw a shoemaker's knife.

3. Find the area of the face of the knife.
4. If the thickness of the knife is 1 mm, find the volume of the knife.

Extension
5. Draw a shoemaker's knife on graph paper.
6. Draw a line PQ perpendicular to AB; and draw a circle of diameter PQ as shown in the
following diagram.



Your report should include all working and diagrams.













Problem 14.17 Paper Rolls
Paper is wrapped on a roll with an outer radius of 14.61 cm and an inner radius of 4.45 cm,
and a paper thickness of 0.008 cm. The inner radius forms a cylindrical core around which
the paper is wrapped.
a. Find N, the number of wraps around the core.
b. Find L, the length of the paper wrapped on a roll.
c. If the paper is being unwrapped at a constant rate of 3 metres per minute, find the time, T,
remaining until the paper supply is exhausted.

Extension
Assume that the paper is stored on a roll with an outer radius of R cm and an inner radius of r
cm and paper thickness of t cm. The inner radius forms a cylindrical core around which the
paper is wrapped.
d. Find N (number of wraps) in terms of R, r and t.
e. Find L (paper length) in terms of r, t and N.





Projects
Project 14.1 Building a House












Simple floor plans drawn to scale are less expensive than architectural plans. That is why
builders prefer to start work from simple floor plans.
You are a builder and have been approached to quote on building the following house.















1. What are the dimensions (in metres) of each room and corridor?
2. What is the total floor area of the house? Ignore the thickness of the walls.

4. What price will you quote? Assume that the quote is based on $875 per square metre?

Your report should include the following:












- Statement of the problem in your own words
- All working and diagrams
- Conclusion
- References
- Acknowledgments

Project 14.2 Dehydration
In this project you are required to consider the ratio of the surface area to the volume of a cube
and then explain why a baby may suffer more from heat than an adult.
We know that for a cube:


Part A
1.
a. Using a Microsoft Excel worksheet, key in 1 in cell A4. Then insert the formula =A4+1 in
cell A5; and copy it down to cell A22.
b. To calculate the surface area of the cube, key in the formula =6*A4^2 in cell B4 and press
Enter. The surface area of the cube is displayed in cell B4. So, the surface area of the cube whose
edge is 1 unit is 6 unit
2
.
c. To calculate the volume of the cube, key in the formula =A4^3 in cell C4 and press Enter. The
total volume of the cube is displayed in cell C4. So, the volume of the cube whose edge is 1 unit
is 1 unit
3
.
d. To calculate the ratio of the surface area to the volume of the cube of edge 1 unit, key in the
formula =B4/C4 in cell D4 and press Enter. The ratio of the surface area to the volume of the












cube is displayed in cell D4. So, the ratio of the surface area to the volume of the cube whose
edge is 1 unit is 6.
e. Copy each formula down to row 22.
f. Save the file and print the worksheet.


3. Look at the cube edge and the ratio of surface area to volume; and explain why babies
dehydrate more quickly than adults.

Part B - Limiting Value
4.
a. Using another worksheet, key in 0.05 in cell A4. Then insert the formula =A4+0.05 in cell
A5; and copy it down to cell A22.
b. Save the spreadsheet.

5.
a. Using another worksheet, key in 100 in cell A4. Then insert the formula =A4+100 in A5; and
copy it down to cell A22.
b. Save the spreadsheet.














Project 14.3 Hanging Wallpaper
Some home owners use wallpaper instead of paint to decorate their house. Hanging wallpaper is
cheaper if you do it yourself. If wallpaper is expensive, using too much is a waste of money. So,
it is important to be able to calculate the exact amount needed for the job.


1. Use a ruler and the given scale to determine the dimensions of the room from the plan.
2. The room is to have all walls covered with wallpaper. Guess what the project will cost.

4. What area of the walls will a roll cover if the roll is 10 m long and 520 mm wide?
5. Find the minimum number of rolls needed to cover the walls.

Extension
Prepare a leaflet that will contain a mathematical model (i.e. table) to determine how many rolls
are needed to paper a room of given dimensions:












a. including doors and windows
b. excluding doors and windows
Your report should include the following:
- Statement of the problem in your own words
- All working and diagrams
- Conclusion
- References
- Acknowledgments

Project 14.4 Estimating Body Surface Area
A person's body surface area is needed to work out the correct dose of a medicine. It is difficult
to measure directly the body surface area. However, a formula that connects the body surface
area to weight and height can be used quite effectively.
The formula used for calculating the body surface area is

1. Use a calculator to estimate the body surface area of the following patients:
a. Mr. Smith - height = 183 cm, weight = 67 kg.
b. Mr. Shepherd, height = 170 cm, weight = 50 kg.
c. Mr. Strong, height = 150 cm, weight = 65 kg.
2. Mr. Shepherd and Mr. Strong have approximately the same body surface area, yet Mr.
Shepherd is 20 cm taller than Mr Strong is. How can you justify this?
3a. Investigate your body surface area by using the following diagram as a model of yourself.













3b. What are the limitations of this model?
4. Compare the computational performance of the different methods.






























































Roof Pitch to Degrees Equivalents
This drawing demonstrates how many degrees rise for each pitch of a typical roof. Look
at the column labeled pitch,
then look under degrees to get the corresponding amount of degrees. Example; 8/12
pitch = 33.75 degrees.



Roof pitch (slope truthfully, but everyone calls it pitch) refers to the amount of
rise a roof has compared to the horizontal measurement of the roof.

The picture below shows the pitch of a 7-12 roof slope meaning that for 12" of
horizontal measurement (roof run) the vertical measurement (roof rise) is 7".

This measurement is best done on a bare roof because curled up roofing












shingles will impair your measurement. If this isn't practical then perform the
same measurement on the underside of the roof. We included a roof pitch chart
below the 7-12 example image and the calculator below that.
7/12 Roofing Pitch

Roof Pitch Examples


























Roof Pitches
12/12 pitch angle 45
11/12 pitch angle 42.51
10/12 roof pitch angle 39.81
9/12 roof pitch angle 36.87
8/12 - roof pitch degrees 33.69
7/12 - 30.26
6/12 - 26.57
5/12 - still walkable 22.62
4/12 - standard roof pitch 18.43
3/12 typical in southern climates 14.04
2/12 - low roof pitch 9.46



Roof Pitch Table
1 12 4.76
2 12 9.46












3 12 14.04
4 12 18.43
5 12 22.62
6 12 26.57
7 12 30.26
8 12 33.69
9 12 36.87
10 12 39.81
11 12 42.51
12 12 45
Roof Pitch Calculator
If none of the above is working for you this tool will convert pitch to angle or angle
to pitch for half degree roof slope calculations.

Enter any pitch or fraction of pitch to find angle. Enter any angle or fraction of
angle to find pitch.

Roofing Pitch Calculator












Pitch
6
Angle
26.57



RIDGE HEIGHT CALCULATOR
Building Width Wall Height Roof Pitch
FT FT /12













Building Profile Generator
(Tool rounds to whole numbers)
Width Height Pitch Length






Binomial Theorem

The Binomial Theorem states

A combinatorial proof:
Consider how to get a term of the form out of the product of n terms each (x + y):
. Such terms are formed by picking j ys and n - j xs. Since once the
ys are picked, there is really no choice for the xs, there are such terms.

A computational proof:
Proof by induction on n.












Basis case:
n = 0 works in the sense that both sides are 1, but most beginners like to start at 1,
so look at n = 1.
The left side for n = 1 is simply x + y.
The right side is
Induction case:
Assume the formula works for n = k and show it works for n = k + 1.
Assume:
Show:
Using the induction assumption, we can rewrite the left side of this equation as
follows.

Using Pascals identity , we get that the
right side of what we are trying to show can be expanded as follows. Note that we
need to be careful since Pascals identity does not apply for k = 0 or for k = n. In
our sum, this means we need to split out the j = 0 and j = k + 1 terms before
applying Pascals identity.













Now we have the problem of matching the two sums from the left side with the
two from the right side. First, notice that the out front on the right side can be
thought of as the j = 0 term in the first sum. Next, the out front on the right
side can be thought of as the j = k + 1 term in the second sum. This gives that the
right side is equal to the following.

From above, .
The first sums are clearly equal since the only difference is that the exponent of x
is changed from k + 1 - j to k - j + 1. All that is left is to show that the second
sums are equal.
To show the second sums are the same, take the second sum from the right side
and change the summation index j to something that starts at zero. Keeping the
same name is allowed, but it is easier to follow if we pick a different letter, so let
q = j - 1. This gives j = q + 1 and 1 - j = -q. Also, since j goes from 1 to k + 1, q
will go from 0 to k. Substituting these into the second sum for the right side gives
the following.













This is the same as the second sum on the left side since the name of the
summation index is the only difference between the two.
Binomial Expansion
Is this is as heavy into combinatorics as we're going to go? Right, you wish. First off recall the
binomial expansion formula:

(1.5)


But what are all those terms in the middle? We write them in terms of combination notation


(1.6)


This means the "combination of n things taken m at a time". This is also sometimes written as .
Then we can write the binomial expansion as

(1.7)














Also the notation "!" means "factorial. So for example . This can be
confusing when you want to write "how about that 5!", but mathematicians seldom get excited in
papers.
Well let's review how you derive this binomial expansion and what the right hand side means.
Start off expanding .
Let's now try .
What's happening here is that this expansion generates all possible combinations a's and b's
which I grouped together as above, so that you can easily write this as .
The coefficients of this are simple cases of that general combination written above. In this case
represents then is the number of ways of choosing two 's from a three
possibilities. Another way of representing this is in terms of Pascal's triangle, but the factorial
formula is usually easier to work with mathematically.
I'm not going to go over a proof of this formula, but there are a lot of discussions of it. You've
likely seen it before in any case. It's a variation on the idea of a permutation. For example if we
have three things a, b, and c, then there are 6 permutations: abc, bac, cab, acb, bca, and cba. It is
pretty easy to show that in general for things, there are permutations of them. That's sort of
the reason why appears three times in the combination formulation.



Binomial Theorem
The theorem is called binomial because it is concerned with a sum of two numbers (bi
means two) raised to a power. Where the sum involves more than two numbers, the
theorem is called the Multi-nomial Theorem. The Binomial Theorem was first discovered
by Sir Isaac Newton.
Notation












We can write a Binomial Coefficient as:
[0.1]

Similarly, combinations can be written:
[0.2]
"n" as a nonnegative integer
The Binomial Theorem or Formula, when n is a nonnegative integer and k=0, 1, 2...n is
the kth term, is:
[1.1]

When k>n, and both are nonnegative integers, then the Binomial Coefficient is zero. This
explains why the above series appears to terminate. That is, it has (n+1) terms.

It can also be written:
[1.2]
Where n and k are references to numbers in Pascal's Triangle, is called a Binomial
Coefficient and is read as "n over k". When it is a combination, it may be read as "n
choose k".












Proof of the Binomial Theorem
The Binomial Theorem was stated without proof by Sir Isaac Newton (1642-1727). The
Swiss Mathematician, Jacques Bernoulli (Jakob Bernoulli) (1654-1705), proved it for
nonnegative integers. Leonhart Euler (1707-1783) presented a faulty proof for negative
and fractional powers. Finally, the Norwegian mathematician, Niels Henrik Abel (1802-
1829), proved the theorem.
Proof when n and k are positive integers
When n and k are nonnegative integers, then the Binomial Coefficients in:
[1.2, repeated]

can be considered combinations, and read "n choose k", as appropriate. Sir Isaac
Newton just rote the formula down in his notebook, without proof, perhaps because he
thought the formula was self-evident.

Looking at 1.2, we choose the number of x's, and first we choose none at all. n choose 0
is 1.

Similarly, we choose 1, 2, ... n x's.

When we choose 1 from n, this is the combination n choose 1, which we know is n.
When we consider the x-squared term, we choose 2 x's from n, and the formula for this is
n(n-1)/2. The combinations appear to prove the theorem.

Another approach, is to assume
[2.1]












where the subscripts are simply for labelling the factors.

The first term of this expansion will be a
n
, and the coefficient is 1. We will think about
choosing x's. So the first terms, which uses all the a's and none of the x's is the number
of ways we can choose 0 x's from n, which is only one way.

The next term is the x term. We need to choose 1 x from n possibilities, and this is n
ways. The coefficient of x is therefore n. This will be of the power (n-1). The sum of the
powers of the n's and a's will always be n.

The x
2
term, is made up of choosing one x and then choosing another (so they make x-
squared). One way to think of this is to pick the first x from the first factor, and then we
have a choice of (n-1) factors for the next x. We can choose the first x in n ways and
there are (n-1) other choices for the next x. There are therefore n(n-1)/2! ways of
choosing two x's and this is the x
2
coefficient. The factorial 2 comes about in the
following way. Consider factors 1 and 2. We can choose an x from factor 1 and then one
from factor 2, or we can choose an x from factor 2 and one from factor 1. Clearly, these
are effectively the same so we are double counting. So we divide by factorial 2. (This is
the number of permutations of 2 objects).

The x3 term is made up of three x's. We have a choice of n x's for the first x, but once we
have chosen, we have only (n-1) x's remaining for the second x. And for the third x we
have (n-2) choices. This means we can choose three x's in n(n-1)(n-2)/3! ways and this is
the coefficient of x
3
. The factorial 3 makes allowance for identical permutations. With
three things, we can arrange them in 6 ways, but, for our purposes, these three x's,
irrespective of the order of selection, are the same combinations. For instance, we could
choose three x's from the first three factors in 6 ways: 123, 132, 213, 231, 312, 321, but
they all make up the same x
3
!













In general, we can select k x's from n factors in n(n-1)(n-2)... (n-k+1)/k! ways. And this is
the coefficient of the general term.

We have proved the Binomial Theorem for nonnegative integers n and k, essentially by
creating its terms and showing they are the same as the terms claimed for the Binomial
Theorem.
Proof when r is any real number
We now prove the Binomial Theorem when the power r, is any real number: positive,
negative, rational, irrational, fractional.. any real number.
I have use (1+x) instead of (a+x) for simplicity, and because we often use the Binomial
Theorem in this way. If there is an a, we simply take it out of the brackets.
The following proof uses simple calculus, and the proof rests on the truth of simple
calculus. It assumes that a Binomial Expansion can be written as:
[4.1]
where r is a real number and k is an integer. ak are the coefficients of the expansion.

When x=0, then a
0
=1

We differentiate and get:
[4.2]
When x=0, a
1
=r

Differentiating again and setting x=0
[4.3]












a2=r(r-1)/2

It is evident that we are extracting the Binomial Coefficients. To cut a long story short,
let us differentiate the function k times
[4.4]
Setting x=0 and rearranging:
[4.5]

Taking the general term (4.5), we show that the left-hand side equals:
[4.6]

And the right-hand side is the Binomial Theorem! We have therefore proved the
Binomial Theorem for all real numbers, so we can legitimately use it with positive and
negative and fractional r, and we are no longer limited to integers. The method of
expanding (1+x)
r
is known as a Maclaurin Expansion. The Binomial Theorem is so
versatile that x can even be a complex number, with a non-vanishing imaginary part!
Defining the Binomial Coefficients
When n and k are nonnegative integers, we can define the Binomial Coefficients as:
[6.1]
k is positive and less than n. If k is less than zero, we have negative factorials, which we
haven't defined (see using the Gamma Function to define factorials, for a method to define












factorials for fractions and complex numbers. Factorials of the negative integers do not
exist.) When k is greater than n, [6.1] is zero, as expected. (This is what makes the
Binomial Expansion with n as a nonnegative integer terminate after n+1 terms!)

When r is a real number, not equal to zero, we can define this Binomial Coefficient as:
[6.2]
When r is zero, *6.2+ gives zero instead of 1, so we restrict *6.2+ to r0. We define B(n,0)
as 1.

If the Binomial Coefficient is also a combination (n and r are positive integers), then we
can use the rules of combinations.
Sum of Binomial Coefficients
We can write the binomial theorem as:

Where n is a positive integer, and k is a nonnegative integer, 0, 1, ..., n and is the term
number.
If we let a=b=1, we find (1+1)
n
=2
n
is the sum of the terms, because the powers of a and b
are all 1, and only the coefficients remain. Naturally, the values of the coefficients are
not changed by the values of a and b, so the sum of the coefficients is always 2
n
,
whatever the values of a and b.

If we write a=1 and b=-1, then (1-1)
n
=0. We also notice that the even powers of b will be
positive and the odd powers will be negative.













The sum of the terms of a binomial expansion equals the sum of the even terms (and
the even powers of b), k=0, 2, etc plus the sum of the odd terms, k=1, 3, 5, etc:


Because when a=1 and b=-1, the odd terms and the even terms cancel out, and their
coefficients are therefore equal, we have:


The coefficients do not change with the values of a and b, so the sum of the coefficients
of the odd terms is always equal to the sum of the coefficients of the even terms, when
n is a positive integer.

Because the sum of the coefficients of the even terms equals the sum of the coefficients
of the odd ones, and because the total sum is 2
n
, we have the sum of the even terms
and the sum of the odd terms are both equal to half the total sum:

Convergence












When the Binomial Expansion is finite, when r is a nonnegative integer, then
the series is always convergent, being the finite sum of finite terms. It is when
the series is infinite that we need to question the when it converges.
According to the ratio test for series convergence a series converges when:
[7.1]
It diverges when:
[7.2]
And the result is indeterminate when:
[7.3]
For the Binomial Theorem the ratio of the kth and the k-1 terms is:
[7.4]

Applying the ratio test when the Binomial Expansion is an infinite series (r is
not a nonnegative integer), we find the limit is
[7.5]

That is, |x|. The Binomial Theorem converges when |x|<1.

When |x| is 1, the ratio test does not advise us on its status.
For example:












[7.6]

This is an infinite series. When x=1, the left-hand side is 1/2 and the right-
hand side is 1-1+1-1+... Yet it appears to be divergent, in the sense of meaning
not convergent (that is, it does not converge to a single finite value), because it
seems to oscillate between -1, 0 and 1. (It seems that the answer 1/2 may be
correct, however) When x=-1, the left-hand side is 1/0, which is infinite, and
the right-hand side is 1+1+1... which is also clearly infinite. In this case, the
series clearly diverges. When |x|=1, we need to examine these cases very
carefully. The simple answer, however, is that when |x|=1, the binomial series
is indeterminate, so discussing the value when x=1 is meaningless.1/(1+1) is a
half, but we cannot obtain this from the Binomial Theorem.

The conclusion here is that when the binomial series is infinite (n is negative
or fractional), then it converges when |x|<1






































MULTIPLICATION OF SUMS
The binomial coefficients

THE BINOMIAL THEOREM gives the coefficients of the product of n equal
binomials:
(x + b)
n
= (x + b)(x + b) (x + b).
If we expanded
(x + b)
4

for example, then before adding the like terms, we would find terms in
x
4
, x
3
b, xb, xb
3
, and b
4
.
What the binomial theorem does is tell how many terms there are of
each kind -- it adds those like terms. Those are the binomial coefficients.












We will see that determining those coefficients depends on the theory of combinations. First,
then, we will consider the multiplication of any sums; for example,
(x + 1)(x + 2)(x + 3).
The final product will have this form:
ax
3
+ bx + cx + d.
As with the binomial theorem, the question is: What are the
coefficients?
They will be some function of the constants 1, 2, 3. For the moment, we will simply show the
result:
(x + 1)(x + 2)(x + 3) = x
3
+ (1 + 2 + 3)x + (1 2 + 1 3 + 2 3)x + 1 2 3

= x
3
+ 6x + 11x + 6.
The coefficient of x
3
is simply 1. The coefficient of x is the sum of
the combinations of 1, 2, 3, taken one at a time.
The coefficient of x is the sum of the combinations of 1, 2, 3, taken two at a time.
And the constant term is their combination taken three at a time.
Why those combinations? We will see why as we continue. For the moment, the student
should attempt this problem.
Problem 1. Multiply out (x + 3)(x + 4)(x 1) by taking the correct
combinations of the integers.

x
3
+ (3 + 4 1)x + (3 4 + 3 1 + 4 1)x + 3 4 1

=
x
3
+ 6x + 5x 12












Let us now begin again, and analyze the multiplication of these
elementary sums:
(x + y)(a + b + c).
When we multiply out, we will find six terms. Each term will contain
two factors, namely one letter from each factor:
xa + xb + xc + ya + yb + yc.
Therefore, to multiply the following,
(x + y)(a + b)(m + n)
simply write the sum of all combinations of one letter from each factor:
xam + xan + xbm + xbn + yam + yan + ybm + ybn.
Each term in the product consists of three factors: one from each binomial.
Now consider these four binomial factors:
(x + a)(x + b)(x + c)(x + d)
Let us see how each term in the product will be produced.
If we were actually to multiply out, we would find 2
4
or 16 terms. For, multiplication of two
binomials gives 4 terms:
(p + q)(m + n) = pm + pn + qm + qn
If we multiply those with a binomial, we will have 8 terms; and finally
those multiplied with a binomial will produce 16 terms.
In general: Multiplication of n binomials produces 2
n
terms.
Now, in the multiplication of those four binomials, each of those 2
4

terms will consist of four factors (one from each binomial); hence we expect
to find terms such as xbcd, axcx, xbxx, etc. That is, we will find terms in x,
x
2
, x
3
, and x
4
.
It remains only to determine the coefficients.












How is a term x
4
produced? By taking x from each factor. xxxx = x
4
. There is only one such
term. The coefficient of x
4
is 1.
Terms with x
3
are formed by taking x from any three factors, in every possible way, and the
letter from the remaining factor.
axxx + xbxx + xxcx + xxxd = (a + b + c + d)x
3

The coefficient of x
3
, therefore, is the sum of the combinations of a, b,
c, d taken 1 at a time: a + b + c + d.
Terms with x will come from taking x from any two factors, in every
possible way, and the letters from the remaining two factors:
abxx + axcx + axxd + xbcx + xbxd + xxcd
= (ab + ac + ad + bc + bd
+ cd)x
The coefficient of x is thus the sum of the combinations of a, b, c, d
taken two at a time.
The coefficient of x will be the sum of those combinations taken three at a time:
abcx + abxd + axcd + xbcd = (abc + abd + acd + bcd)x
Finally, the term independent of x is formed by those combinations
taken four at a time: abcd.
We have,
(x + a)(x + b)(x + c)(x + d)
= x
4
+ (a + b + c + d)x
3
+ (ab + ac + ad + bc + bd + cd)x

+ (abc + abd + acd + bcd)x + abcd.
Notice again that each term has four factors.
Example. In this multiplication,












(x + 1)(x 2)(x + 2)(x + 3),
what number will be the coefficient of x.
Solution. Terms containing x are formed by taking x from any two
factors, in every possible way, and the numbers from the remaining two
factors. Therefore, the coefficient of x will be the sum of all possible
combinations of the numbers 1, 2, 2, 3, taken two at a time. There will
be
4
C2 or 6 such terms:
1 2 + 1 2 + 1 3 + 2 2 + 2 3 + 2 3 = 2 + 2 + 3 4 6 + 6

= 1.
The coefficient of x is 1.
The binomial coefficients
Now consider the product of these four equal binomials:
(x + b)(x + b)(x + b)(x + b)
That is, let us expand (x + b)
4
.
Again, if we were to actually multiply out, then before collecting like terms, we would find
2
4
or 16 terms; and again every term would consist of four factors (one from each
binomial). There will be terms such as
xxxx, xxbx, bbxx,
and so on. That is, there will be terms in x
4
, x
3
b, xb, xb
3
, and b
4
.
The binomial coefficients are how many terms there are of each kind.
Now, how is a term x
4
produced? By taking x from each factor. xxxx = x
4
. There is only
one such term. The coefficient of x
4
is 1.
How will a term x
3
b be produced?
(x + b)(x + b)(x + b)(x + b)












By taking b from any one factor, and x from the remaining three:
bxxx + xbxx + xxbx + xxxb
Therefore the number of terms equal to x
3
b will be the number of ways of taking 1 thing --
letter b -- from 4. There will be
4
C
1
= 4 terms equal to x
3
b. On adding those like terms, we
will have 4x
3
b.
A term xb will come from taking b from any two factors in every
possible way, and x from the remaining two.
bbxx + bxbx + bxxb + xbbx + xbxb + xxbb
Therefore, the number of terms equal to xb will be the number of ways
of taking 2 things -- letter b -- from 4. We will have
4
C2xb, that is, 6xb.
A term xb
3
will come from taking b from any three factors, and x from the remaining
factor; therefore, there will be
4
C
3
terms equal to xb
3
; we will have 4xb
3
.
And finally, a term b
4
comes from taking b from each of the four factors; there is only 1
way to do this; we will have
4
C
4
b
4
= b
4
.
This explains the binomial coefficients for the expansion of (x + b)
n
when n = 4. They are
1 4 6 4 1.
Again, the binomial coefficients are how many terms there are of each kind. They are
none other than the combinatorial numbers
n
C
k
, where k successively takes on the values
0, 1, 2, 3, 4.
This result is general. The binomial theorem states that in the expansion of (a + b)
n
, the
coefficients are the combinatorial numbers
n
C
k
, where k successively takes on the values
0, 1, 2, . . . , n. Each term in the
expansion will have this form:
n!
(n k)! k!
a
n k
b
k
.
Problem 2. Imagine multiplying out (x + y + z)(a + b + c).
a) How many terms would there be? 3
2
= 9
b) Each term would consist of how many factors? Two












Problem 3. Imagine multiplying out (x + a)(x + b)(x + c)(x + d).
a) How many terms would there be? 2
4
= 16
Thus a product of n binomials consists of how many terms? 2
n

b) Each term would consist of how many factors? Four
c) How is a term produced that contains three factors of x, that is, x
3
?
By taking x from any three of the factors, in every
possible way, and a letter from the remaining factor.
d) Therefore, what is the coefficient of x
3
? a + b + c + d
e) What is the coefficient of x
4
? 1
f) What is the coefficient of x? ab + ac + ad + bc + bd + cd
g) What is the coefficient of x? abc + abd + acd + bcd
h) What is the constant term? abcd
Problem 4. Multiply out by taking the correct combinations of the
integers.
a) (x + 1)(x + 3)(x + 4) = x
3
+ 8x + 19x + 12
b) (x + 1)(x + 2)(x + 3)(x 1) = x
4
+ 5x
3
+ 5x 5x 6
Problem 5. In this multiplication (x + 1)(x + 2)(x + 3)(x + 4)(x +
5) what will be the coefficient of x
3
? 85
Problem 6. (x + b)
5
= (x + b)(x + b)(x + b)(x + b)(x + b)
a) Upon multiplying out, and before collecting like terms, how many
a) terms will be produced? 2
5
= 32
b) How will a term x
3
b be produced?
By taking b from any two factors, in every possible
way, and x from the remaining three factors.












c) How many times will that term be produced? In other words,
upon adding those like terms, what number will be the coefficient of
x
3
b?
5
C2 = 10

Problem 7. In each row of Pascal's triangle, the sum of the
binomial coefficients is 2
n
. Why?
2
n
is the number of terms upon multiplying n binomials.
Each binomial coefficient tells how many terms of that
kind. Therefore, the sum of all of them will be 2
n
.









The Formal Rules of Algebra
ALGEBRA is a method of written calculations. Now what is a
calculation? It is replacing one set of symbols with another? In












arithmetic we may replace the symbols '2 + 2' with the symbol '4.' In
algebra we may replace 'a + (a)' with '0.'
a + (a) = 0.
A formal rule, then, shows how an expression written in one form may
be rewritten in a different form. The = sign means "may be rewritten as"
or "may be replaced by."
If p and q are statements (equations), then a rule
If p, then q,
or equivalently
p implies q,
means: We may replace statement p with statement q. For example,
x + a = b implies x = b a.
That means that we may replace the statement 'x + a = b' with the
statement 'x = b a.'
Algebra depends on how things look. We can say, then, that algebra is a system of formal
rules. The following are what we are permitted to write.
(See the complete course, Skill in Algebra.)

11. The axioms of "equals"
a = a Identity

If a = b, then b = a. Symmetry

If a = b and b = c, then a = c. Transitivity












These are the "rules" that govern the use of the = sign.

12. The commutative rules of addition and multiplication
a + b = b + a

a b = b a
13. The identity elements of addition and multiplication:
3. 0 and 1
a + 0 = 0 + a = a
a 1 = 1 a = a
Thus, if we "operate" on a number with an identity element,
it returns that number unchanged.
14. The additive inverse of a: a
a + (a) = a + a = 0
The "inverse" of a number undoes what the number does.
For example, if you start with 5 and add 2, then to get back to 5
you must add 2. Adding 2 + (2) is then the same as adding 0
-- which is the identity.
15. The multiplicative inverse or reciprocal of a,
5. symbolized as
1
(a 0)












a
a
1
a
=
1
a
a = 1
Two numbers are called reciprocals of one another if their product is 1.
Thus, 1/a symbolizes that number which, when multiplied by a, produces 1.
The reciprocal of
p
q
is
q
p
.

16. The algebraic definition of subtraction
a b = a + (b)
Subtraction, in algebra, is defined as addition of the
inverse.
17. The algebraic definition of division
a
b
= a
1
b
Division, in algebra, is defined as multiplication by the
reciprocal.
Hence, algebra has two fundamental operations: addition
and multiplication.
18. The inverse of the inverse
(a) = a












19. The relationship of b a to a b
b a = (a b)
Now, b + a is equal to a + b. But b a is the negative
of a b.
10. The Rule of Signs for multiplication, division, and
10. fractions
a(b) = ab. (a)b = ab. (a)(b) = ab.
a
b
=
a
b
.
a
b
=
a
b
.
a
b
=
a
b
.
"Like signs produce a positive number; unlike signs,
a negative number."




11. Rules for 0
a 0 = 0 a = 0
If a 0, then
0
a
= 0.
a
0
= No value.
0
0
= Any number.
Division by 0 is an excluded operation. (Skill in Algebra, Lesson 5.)












12. Multiplying/Factoring
m(a + b) = ma + mb The distributive rule/
Common factor

(x a)(x b) = x (a + b)x + ab
Quadratic trinomial



(a b) = a 2ab + b Perfect square trinomial



(a + b)(a b) = a b The difference of

two squares



(a b)(a ab + b) = a b
The sum or












difference of
two cubes

13. The same operation on both sides of an equation
If If


a = b, a = b,

then then

a + c = b + c. ac = bc.
We may add the same number to both sides of an equation; we may
multiply both sides by the same number.
14. Change of sign on both sides of an equation
If
a = b,
then












a = b.
We may change every sign on both sides of an equation.
15. Change of sign on both sides of an inequality:
15.

Change of sense
If
a < b,

then
a > b.
When we change the signs on both sides of an inequality, we must
change the sense of the inequality.
16. The Four Forms of equations corresponding to the Four
Operations and their inverses
If If













x + a = b, x a = b,

then then
x = b a. x = a + b.
* * *
If If

ax = b,
x
a
= b,

then then
x =
b
a
. x = ab.
See Skill in Algebra, Lesson 9.
17. Change of sense when solving an inequality
If












ax < b,

then

x >
b
a
.
18. Absolute value
If |x| = b, then x = b or x = b.
If |x| < b then b < x < b.
If |x| > b (and b > 0), then x > b or x < b.
19. The principle of equivalent fractions
x
y
=
ax
ay

and symmetrically,
ax
ay
=
x
y
We may multiply both the numerator and denominator by the same
factor; we may divide both by the same factor.












20. Multiplication of fractions
a
b

c
d
=
ac
bd

a
c
d
=
ac
d
21. Division of fractions (Complex fractions)


Division is multiplication by the reciprocal.
22. Addition of fractions
a
c
+
b
c
=
a + b
c
Same denominator

a
b
+
c
d
=
ad + bc
bd
Different denominators with
no common factors

a
bc
+
e
cd
=
ad + be
bcd
Different denominators with
common factors












The common denominator is the LCM of denominators.
23. The rules of exponents
a
m
a
n
= a
m+n
Multiplying or dividing

a
m

a
n

= a
mn

powers of the same base


(ab)
n
= a
n
b
n
Power of a product of factors



(a
m
)
n
= a
mn
Power of a power
24. The definition of a negative exponent
a
n
=
1
a
n

25. The definition of exponent 0












a
0
= 1
26. The definition of the square root radical

The square root radical squared produces the radicand.
27. Equations of the form a = b
If
a = b,
then
a = .
28. Multiplying/Factoring radicals

=

and symmetrically,

=

29. The definition of the nth root

30. The definition of a rational exponent













It is more skillful to take the root first.
31. The laws of logarithms
log xy = log x + log y.
log
x
y
= log x log y.
log x
n
= n log x.
log 1 = 0. log
b
b = 1.
32. The definition of the complex unit i

i = 1



































RATIONAL AND IRRATIONAL NUMBERS
What is a rational number?

Which numbers have rational square roots?
The decimal representation of irrationals
What is a real number?













CALCULUS IS A THEORY OF MEASUREMENT. The necessary numbers are the
rationals and irrationals. But let us start at the beginning.

The following numbers of arithmetic are the counting numbers or, as they are called, the natural
numbers:
1, 2, 3, 4, and so on.
If we include 0, we have the whole numbers:
0, 1, 2, 3, and so on.
And if we include their algebraic negatives, we have the integers:
0, 1, 2, 3, and so on.
("plus or minus") is called the double sign.
The following are the square numbers, or the perfect squares:
1 4 9 16 25 36 49 64, and so on.
They are the numbers 1 1, 2 2, 3 3, 4 4, and so on.

Rational and irrational numbers
1. What is a rational number?
Any ordinary number of arithmetic: Any whole number,
fraction, mixed number or decimal; together with its negative
image.
A rational number is a nameable number, in the sense that we
can name it in the standard way we name whole numbers,
fractions and mixed numbers. "Five," "Six thousand eight
hundred nine," "Nine hundred twelve millionths," "Three and
five-eighths."












What is more, we can in principle (by Euclid VI, 9) place any
rational number exactly on the number line.

We can say that we truly know a rational number.
2. Which of the following numbers are rational?
1 1 0
2
3

2
3
5 5 6.08 6.08 3.1415926535897932384626
All of them! All decimals are rational. That long one is an
approximation to , which, as we will see, is not equal to any
decimal.
3. A rational number can always be written in what form?
As a fraction
a
b
, where a and b are integers (b 0).
An integer itself can be written as a fraction: b = 1. And from arithmetic, we know that we
can write a decimal as a fraction.
When a and b are positive, that is, when they are natural numbers, then we
can always name their ratio. Hence the term, rational number.
At this point, the student might wonder, What is a number that is not
rational?
An example of such a number is ("Square root of 2"). It is not possible to name any
whole number, any fraction or any decimal whose
square is 2.
7
5
is close, because












7
5

7
5
=
49
25
-- which is almost 2.



To prove that there is no rational number whose square is 2, suppose
there were. Then we could express it as a fraction
m
n
in lowest terms.
That is, suppose
m
n

m
n
=
m m
n n
= 2.
But that is impossible. Since
m
n
is in lowest terms, then m and n have
no common divisors except 1. Therefore, m m and n n also have no common

divisors -- they are relatively prime -- and it will be impossible to divide n n

into m m and get 2.

There is no rational number -- no number of arithmetic -- whose square is 2.

Therefore we call an irrational number.

























By recalling the Pythagorean theorem, we can see that
irrational numbers are necessary. For if the sides of an
isosceles right triangle are called 1, then we will have 1 + 1 =
2, so that the hypotenuse is . There really is a length that
logically deserves the name, " ." Inasmuch as numbers name
the lengths of lines, then is a number.
4. Which natural numbers have rational square roots?
Only the square roots of the square numbers; that
is, the square roots of the perfect squares.
= 1 Rational
Irrational
Irrational
= 2 Rational












, , , Irrational
= 3 Rational
And so on.
Only the square roots of square numbers are rational.
The existence of these irrationals was first realized by Pythagoras in the
6th century B.C. He understood that in the isosceles right triangle, the
ratio of the hypotenuse to the side was not the same as two natural
numbers. They were, he said, "without a name." Because if we ask, "What
ratio has the hypotenuse to the side?" -- we cannot say. We can express it
only as "Square root of 2."
5. Say the name of each number.
a) "Square root of 3." b) "Square root
of 5."
c) "2." This is a rational -- nameable -- number.
d) "Square root of 3/5." e) "2/3."





In the same way we saw that only the square
roots of square numbers are rational, we could
prove that only the nth roots of nth powers are
rational. Thus, the 5th root of 32 is rational
because 32 is a 5th power, namely the 5th
power of 2. But the 5th root of 33 is irrational.
33 is not a perfect 5th power.













The decimal representation of irrationals
When we express a rational number as a decimal, then either the decimal
will

a predictable pattern of digits. But if we attempted to express an irrational
number as an exact decimal, then, clearly, we could not, because if we
could the number would be rational
Moreover, there will not be a predictable pattern of digits. For example,

1.4142135623730950488016887242097
Now, with rational numbers you sometimes see
1
11
= .090909. . .
By writing both the equal sign = and three dots (ellipsis) we mean:
"A decimal for
1
11
will never be complete or exact. However we can
approximate it with as many decimal digits as we please according to the
indicated pattern; and the more decimal digits we write, the closer we will
be to
1
11
."

(That explanation is an example of mathematical realism. It asserts that in
the mathematics of computation and measurement, which includes
calculus, what exists is what we actually observe or name, now. That
.090909 never ends is a doctrine that need not concern us because it is not












necessary. Such actual infinities have no practical effect on calculations in
arithmetic or calculus.)

We say that any decimal for
1
11
is inexact. But the decimal for ,
which is .25, is exact.

The symbol for decimal fractions was invented in the 16th
century. Now, of course, we take decimals for granted, but at
the time many thought it was not a very forward looking idea,
because the decimals for only a very limited number of
fractions are exact.

Even the decimal for as simple a fraction as
1
3
is inexact. See
Lesson 24 of Arithmetic.
As for the decimal for an irrational number, it is always inexact. An
example is the decimal for above.
If we write ellipsis --
= 1.41421356237. . .
-- we mean, "A decimal for will never be complete or exact.
Moreover, there will not be a predictable pattern of digits. We could
continue its rational approximation for as many decimal digits as we please,
by means of the algorithm, or method, for calculating each next digit (not
the subject of these Topics); and again, the more digits we calculate, the
closer we will be to ."












It is important to understand that no decimal that you or anyone will ever
see is equal to , or , or any irrational number. We know an irrational
number only as a rational approximation. And if we choose a decimal
approximation, then the more decimal digits we calculate, the closer we
will be to the value.
To sum up, a rational number is a number we can know exactly, either as
a whole number, a fraction or a mixed number, but not always exactly as a
decimal. An irrational number we can never know exactly in any form.
The language of arithmetic is ratio. It is with that language
that we relate each rational number to 1, which is the ground.
The whole numbers are the multiples of 1,
the fractions are its parts. But we cannot relate an irrational
number to 1. Like Pythagoras, we cannot say. That is the great
problem of the irrational. We cannot put into words how it is
related to 1.
One often hears however that an irrational number "is" an
actually infinite decimal.
= 1.41421356237. . .
But if a decimal, even as an idea, did not end, then it
would not be a number. Why not? Because decimals, to
be useful, have names, and therefore we can name their
sum, their difference, their product and their quotient. But
an infinite sequence of digits does not have a name. It is
not that we will never finish naming it. We cannot even
begin.
See The mathematical existence of numbers.

Real numbers












5. What is a real number?
A real number is distinguished from an imaginary number.
It is a number we require for measuring rather than counting.
Hence a real number is any rational or irrational number that
we can name. They are the numbers we expect to find on the
number line.
(An actual measurement can result only in a rational number.
An irrational number can result only from a theoretical
calculation; examples are the Pythagorean theorem, and
solving
an equation such as x = 5.
Any serious theory of measurement must address the question:
Which irrational numbers are theoretically possible? Which
ones could be actually predictive of a measurement?)
Problem 1. We have categorized numbers as real, rational,
irrational, and integer. Name all the categories to which each of the
following belongs.
3 Real, rational, integer. 3 Real, rational, integer.

Real, rational. Real, irrational.

5 Real, rational. 11/2 Real, rational.

1.732 Real, rational.
6.920920920. . . Real,
rational.













6.9205729744. . . Real. And let us assume that it is irrational, that is,
that the digits do not repeat. Moreover, we must assume that there is
an effective procedure for computing each next digit. For if there
were not, then we could not name which number we are computing.
That symbol would not then refer to any "number."
6.9205729744 Real, rational. Every exact decimal is rational.
7. What is a real variable?
A variable is a symbol that takes on values. A value is a
number.
A real variable takes on values that are real numbers.
Calculus is the study of functions of a real variable.
Problem 2. Let x be a real variable, and let 3 < x < 4. Name five
values that x might have.














FRACTIONS INTO DECIMALS
Exact versus inexact
In the previous section we saw the most frequent
and therefore the most important decimal and percent equivalents.
Nevertheless, we now ask the following:

3. What is a general method for changing a fraction to a decimal?
4
11
Divide the numerator by the denominator.


Example 1. Write
4
11
as a decimal.
Solution.
4
11
= 4 11. As they said in the Little Red School House,
"Let 4 fall into the house "

11 does not go into 4. Write 0 in the quotient, place a decimal
point,













and add a 0 onto the dividend. (Lesson 12)
"11 goes into 40 three (3) times (33) with 7 left over.

"11 goes into 70 six (6) times (66) with 4 left over."

Since we are dividing 11 into 40 again, we see that this division will never be exact. We
will have 36 repeated as a pattern:
4
11
= 0.363636. . .
By writing three dots (called ellipsis), we mean, "No decimal for
4
11
will ever be complete or exact. However we can approximate it with as
many decimal digits as we please according to the indicated pattern; and
the more decimal digits we write, the closer we will be to
4
11
."
That explanation is an example of mathematical realism. It
asserts that in the mathematics of computation and
measurement, what exists is what we actually observe or
name, now. That 0.363636 never ends is a doctrine that
need not concern us because it is not necessary. Such actual
infinities have no practical effect on calculations in arithmetic
or calculus.
What is more, if the decimal really did not end, it would not
be a number. Why not? Because a decimal, to be useful, has












a name. It is not that we will never finish naming an infinite
sequence of digits. We cannot even begin.
And so we cannot express
4
11
exactly as a decimal. Therefore if we
want to use that number as a decimal, we must approximate it. Let us
approximate it with three decimal digits (Lesson 12):
4
11

0.364
Example 2. Write 5
4
11
as a decimal.

Answer. According to what we just saw:
5
4
11

5.364
Exact versus inexact decimals
We say then that any decimal for
4
11
is inexact. But the decimal for ,
which is .25, is exact.
The decimal .363636 in and of itself is exact. But
as a value for
4
11
, it is inexact.
Fractions, then, when expressed as decimals, will be either exact or
inexact. Inexact decimals nevertheless exhibit a pattern of digits. The
pattern for
4
is .3636363636.












11

Which fractions will have exact decimals? Only those in which the
factors of the denominator are made up of 2's and/or 5's. They are
fractions with these denominators: 2, 4, 5, 8, 10, 16, 20, 25, 40, 50,
and so on.
Example 3.
a) Show the decimal pattern that
1
9
generates.

9 goes into 1 zero (0).
9 goes into 10 one (1) time with 1 left over.
Again, 9 goes into 10 one (1) time with 1 left over.
And so on. This division will never be exact -- we will keep getting 1's in the quotient.
1
9
= 0.111111. . .
b) Use that value for
1
9
to find the value of
8
9
.
Solution.
8
9
= 8
1
9
= 8 0.111111. . . = 0.8888888. . .
c) Round off
8
9
to three decimal digits.












8
9
0.889.
See Problem 15 at the end the Lesson.
Example 4. Calculator problem. Write as a decimal:
73
96
.
Answer. Divide 73 by 96. Press
73 96 =
Displayed is
0.7604166
Therefore, to three decimal digits,
73
96

.760
Example 5. In a class of 52 students, 29 were women.
a) What fraction were women?
Answer. Since 29 out of 52 were women, then
29
52
were women.
(Lesson 20, Question 10.)
b) Use a calculator to express that fraction as a decimal.
Answer. Press
2 9 5 2 =












See
0.5576923
This is approximately .558.
c) What percent were women?
Answer. To change a number to a percent, multiply it by 100.
.558 = 55.8%
(Lesson 4.)
In summary, look at what we have done:
29 out of 52 =
29
52
= 29 52

.558 = 55.8%
"Out of," with a calculator, always signifies division: Division of a
smaller number by a larger.
























Lesson 4 Section 2
The Meaning of Percent
The student should first understand Section 1: Multiplying and
Dividing by Powers of 10.

5. What does "percent" mean?

Percent is an abbreviation for the Latin per centum, which means for each 100.

Thus, 100% means 100 for each 100, which is to say, all. 100% of
12 is 12.
50% is another way of saying half, because 50% means 50 for each 100, which is half.
50% of 12 is 6.
A percent means how many for each 100. 28% means 28 for each 100.
Example 1. Below are 100 small squares, and 32 have been shaded.

What percent of the squares have been shaded?
Answer. 32% -- 32 for each 100.
If we think of 100 being divided into one hundred equal












parts, that is, into hundredths, then a percent is a number of
hundredths.
When the percent is less than or equal to 100%, then we can say "out of"
100. 32% is 32 out of 100. But to say that 200% is 200 out of 100 makes
no sense. 200% is 200 for each 100, which is to say, twice as much.
Example 2. 100 people were surveyed, and 65 responded Yes. What
percent responded Yes?
Answer. 65% -- 65 out of 100.
Example 3. In a class of 30 students, all 30 came to school by bus. What
percent came to school by bus?
Answer. 100%. 100% means all.
(30 out of 30 is equivalent to 100 out of 100.)


6. How can we take 1% of a number?

1% of $200?

Divide it by 100.

















Because 1% is the hundredth part of 100%.
100% is made up of one hundred 1%'s.


We will see that if a number is divided into equal parts, then
we can know how many there are in each part
by dividing by the number of parts. Lesson 11.

Example 4.
1% of $200 is $2.00 Divide by 100: Separate two decimal digits.

1% of $250 is $2.50 Again, divide by 100: Separate two decimal digits.

1% of $6.00 is $.06 Move the point two places left. Do not write the 0's that remain on the
extreme right of the decimal:

6.00 100 = .0600 = .06

(Lesson 3, Question 8.)

1% of $1,200 is $12.00

Divide by 100: Separate two decimal digits, or simply drop the two 0's.
Example 5. How much is 1% of $400? How much is 2% of $400?












How much is 3%? How much is 9%?
Answer. 1% of $400 is $4.00. Separate two decimal digits.
Now, 2% is twice as much as 1%. Therefore 2% of $400 is $8. 3% is $12. 4% would be
$16. 9%, therefore, is 9 $4 = $36.

Example 6. How much is 8% of $600?
Answer. Since 1% is $6.00, then 8% is 8 $6.00 = $48.00.
Example 7. How much is 2% of $325? How much is 3%? 4%?
Answer. We can get everything from 1%, which is $3.25.
2%, therefore, is $6.50.
3% is $9.75.
And 4% is 4 $3.25 = 4 $3 + 4 $.25 = $12 + $1 = $13.
These are problems that do not require a calculator. The student should
practice them mentally


7. How can we take 10% of a number?

10% of $600

Divide it by 10.
















Because 10% is the tenth part of 100%. 100% is made up of ten 10%'s.


Example 8.
10% of $600 is $60. Divide by 10: Take off one 0, or separate one decimal digit.
10% of $625 is $62.50 Divide by 10. Separate one decimal digit: $62.5
But we write money with two decimal digits (cents). Therefore we
must add a 0 onto the right.
10% of $6.00 is $.60 Move the point one place left. Again, we write money with two
decimal digits.
Taking both 10% and 1% in this way takes advantage of our decimal
system and its positional numeration.
Example 9. How much is 20% of $80? How much is 30%? How much
is 90%
Answer. 20% is twice as much as 10%. Since 10% of $80 is $8, then
20% is 2 $8 = $16. 30% is 3 $8 = $24. 90% is 9 $8 = $72.
See especially Problem 23 at the end of the Lesson.
(In Lesson 16, Question 12, we will see that 5% is half of 10%, and
so it will be very easy to take 5%. We will also see that 15% is 10% plus
5% )












The next step in understanding percent is knowing what it
means to say that they are parts of 100%. Since 50% is half
of 100%, then 50% means half. Since 25% is a quarter of
100%, 25% means a quarter. 20% means a fifth. And so on:
Lesson 15.
To prepare for this next skill, can you name the powers of 10 backwards,
starting with Million?
Million, hundred thousand, ten thousand, one thousand,
hundred, ten, one.
Example 10. How much is 1% of One million?
Answer. To take 1%, we must divide by 100. But to do that, we can
divide by 10 twice -- and that will take us two powers of 10 less:
Hundred thousand, Ten thousand.
Example 11. How much is 6% of One million dollars?
Answer. Since 1% is Ten thousand dollars, then 6% is 6 times
Ten thousand dollars, which is Sixty thousand dollars.
Example 12. How much is 3% of Ninety thousand dollars?
Answer. First, 1% is two powers of 10 less: Nine hundred dollars.
Therefore 3% is Twenty-seven hundred dollars.
Example 13. How much is 8% of Three million dollars?
Answer. Since 10% is Three hundred thousand dollars, then 8% is a bit less:
Two hundred forty thousand dollars.
Topics in percent continue in Lessons 14, 15, 16, and 27.
*












Now, a percent is not a number. Rather, it expresses a relationship between numbers. What
percent -- what relationship --- has 6 to 12?
6 is 50% of 12. That kind of relationship is called a ratio (Lesson 17), and understanding
that percents are ratios will turn out to be extremely important. To accomplish certain
calculations, however, it is useful to represent a percent by a number. And since a percent
is how many for each 100 -- 24% is 24 for each 100 -- a percent is a number of hundredths.
24% is 24 hundredths.


8. How do we change a percent to a number?

24% = ?

Divide by 100, and drop the % sign.


(This is often called changing a percent to a decimal.)
24% = .24
Divide by 100 -- separate two decimal digits.
Division by 100 is indicated by the percent sign itself %, with its division slash / and two
0's.
Here are more examples:
.24% =

.0024

Divide by 100: Move the decimal point two places left.
9% =

.09

Divide by 100: Separate two decimal digits.












650% =

6.5

Divide by 100: Separate two decimal digits.
It is not necessary to write the 0 that remains on the right of the decimal. 650%
= 6.50 = 6.5

6.5% =

.065

Divide by 100: Move the decimal point two places left.
Inversely:


9. How do we change a number to a percent?

.24 = ? %


Multiply it by 100, and add the % sign.

Example 14.
.24 = 24% Multiply by 100: Move the point two places right.

2.4 = 240% Move the point two places right.

24 = 2400% Multiply by 100: Add on two 0's.













.024 = 2.4% Move the point two places right.
Number to a Percent

.24 = 24%

Percent to a Number





Lesson 5
MORE ELEMENTARY ADDITION
Mental calculation
We now begin the four problems of arithmetic: Addition,
subtraction, multiplication, and division. They are also called the
four operations. We continue to emphasize problems that do not
require a calculator -- problems that you should not even have to
write!














In this Lesson, we will answer the following:
1. How do we add mentally by composing a multiple of 10?
2. What does it mean to add by endings?
Complements.
3. How do we add mentally by place value?
Section 2
4. How do we add by rounding off? And how do we compensate?

The student should now have mastered the elementary addition of one-digit numbers, and
should be practicing the multiplication table.
For, once you know that
5 + 4 = 9,

then you would also know

50 + 40 = 90,

500 + 400 = 900,













5,000 + 4,000 = 9,000
These are not problems to write in a column.
Our first skill is based on the following properties of addition:
1. A sum does not change if we decompose a term into two
or more terms.

25 + 7 = 25 + 5 + 2 = 32.

2. A sum does not change no matter how we group the terms.



8 + 6 + 2 + 4 = (8 + 2) + (6 + 4) = 20.

Essential for mental calculation is knowing the multiples of 10:
10, 20, 30, 40, 50, and so on.

1. How do we add mentally by composing a multiple of 10?
25 + 7














Decompose -- break up -- the smaller number and regroup with the larger to form the next
multiple of 10. Then add what remains of the smaller number.


Example 1. 25 + 7 = (25 + 5) + 2 = 30 + 2 = 32.

Similarly,
38 + 4 = (38 + 2) + 2 = 42.
59 + 7 = (59 + 1) + 6 = 66.
96 + 8 = (96 + 4) + 4 = 104.
To add 96 + 8, then, an educated person does not have to take out
a
pencil, write in a column and say,
"6 plus 8 is 14 -- write 4, carry 1. etc., etc., etc."
Nor does an educated person count on her fingers. Rather, an
educated person knows elementary addition, and therefore that 96 + 8 is
not very different from 6 + 8. It ends in 4.
Example 2. Since
9 + 6 = 15,
then
90 + 60 = 150













$.90 + $.60 = $1.50

$.90 + $.64 = $1.54
A 9 plus a 6 always gives a "15." This brings us to the practical technique for adding
several numbers --
7 + 8 + 3 + 9 + 4
Adding by endings.


2. What does it mean to add by endings?


It means to know the digit in which the sum
will end.


For example, since
8 + 4 = 12,
you would know that if you add any number ending in 8 to any number
ending in 4, the sum will always end in 2.
28 + 4 = 32
38 + 4 = 42












68 + 4 = 72
And so on. Each answer falls in the next decade.
Similarly, since
6 + 5 = 11
then
26 + 5 = 31
46 + 5 = 51
76 + 5 = 81
They all end in 1. These are not problems to write down.
Example 3. Add mentally -- or aloud -- from left to right.
7 + 8 + 3 + 8 + 6 + 4 + 9
Say only the partial sum as you come to it. Do not say,
"7 plus 8 is 15" -- say only "15." Then look at 3 and say 18. And so on.
Use your knowledge of the ending; even exaggerate saying it.
7 + 8 + 3 + 8 + 6 + 4 + 9

Say: "15 18 26 32 36 45."

Complements
Two numbers are called complementary if their sum is 10. And since the
order of terms does not matter, we may take advantage of complements.
Example 4. Add 6 + 4 + 3 + 1 + 8 + 7 + 2.













Solution. 6 + 4 + 3 + 1 + 8 + 7 + 2 = 30 + 1 = 31.


3. How do we add mentally by place value?

Add the numbers as you read them -- from left to right. Add from higher place to lower.


435 + 461 = 896
First add the hundreds, then the tens, then the ones.
We now see the fundamental principle of all mental calculation:
Calculate from left to right, just as you read.
The last number you say is the answer
Example 5. Counting by 10's. 30 + 24 = 54.

Here are other examples:
20 + 16 = 36
40 + 38 = 78
40 + 62 = 102












40 + 82 = 122
90 + 73 = 163
Example 6. 43 + 25
First add the tens then the ones. Say
"Sixty --
43 + 25
-- eight."
Or, you could say,
"43 plus 20 is 63, plus 5 is 68."


Say, "150 plus 7 is 157."

Say only, "50 plus 14 is 64."
The art of mental calculation is to say as little as possible. The last
number you say is the answer.













"170 plus 11 is 181."
Example 10. 23 + 32 + 25 + 12
First add all the tens, then add on the ones. As you add each place,
say the partial sum. Say

The last number you say is the entire sum.
Example 11. 34 + 25 + 32




"50, 80, 89, 91."












Example 12. 653 + 224
First add the hundreds, then the tens, then the ones. Again, say each partial sum:
653 + 224
Say,
"800, 870, 877."
Example 13. Three tracks on a CD have the following times:
10:34
6:25
8:07
What is the total time?
(10:34 means 10 minutes 34 seconds. 60 seconds = 1 minute.
Therefore, 72 seconds = 1 minute 12 seconds. 1:12.)
Technique. Start with the minutes and count:
"16 plus 8 is 24 minutes."
Now add on the seconds.
"24:59 plus 7 is 24: 66."
The total time is 25 minutes 6 seconds.















THEORETIC ARITHMETIC
Triangular numbers
Square numbers
The sum of consecutive cubes
Triangular numbers
A formula for the triangular numbers
Square numbers
The sum of consecutive cubes
IN THIS TOPIC we will look at numbers themselves, and not just their
representatives: the symbols 1, 2, 3, 4, and so on. By doing so we
will see unexpected structures that are inherent in the natural
numbers.
Here is a number in the form of a triangular array:

This is number 10. (Count them ) Since 10 can be represented in this
way, we call 10 a triangular number.
Now, how do we generate a triangular number? We begin with 1:

We say that 1 is the first triangular number. To form the next, we add 2:

So the next triangular number is 3. The number we add to the previous
triangular number is called the gnomon (NOH-mon). We added the gnomon
2 to 1.
To form the next triangular number, we add the gnomon 3:













It produces the next triangular number, 6.
To form the next, we add 4:

And so the first four triangular numbers are 1, 3, 6, 10. But what is
important is that each one is the sum of consecutive numbers
1.
1 + 2 = 3.
1 + 2 + 3 = 6.
1 + 2 + 3 + 4 = 10.
Problem 1. Write the first ten triangular numbers.
To see the answer, pass your mouse over the colored area.
To cover the answer again, click "Refresh" ("Reload").
Do the problem yourself first!
1, 3, 6, 10, 15, 21, 28, 36,
45, 55.
The difference between consecutive triangles increases by 1.
A formula for the triangular numbers
We will now show that a triangular number -- the sum of consecutive
numbers -- is given by this algebraic formula:
n(n + 1),












where n is the last number in the sum.
To see that, look at this oblong number, in which the base is one more than the height:
<>
An oblong number is the product of a number with its successor, which
is the one after. Algebraically, it has the form
n(n + 1).
In this one, the height n = 4; that is, this oblong number is 4 5 = 20.
But an oblong is composed of two equal triangles:
<>
Therefore each triangle is half of the oblong. It is
n(n + 1).
The sum of consecutrive numbers is equal to half the product of the
last number in the sum with its successor.
Example. Find the sum of the first 50 numbers -- that is, find the 50th
triangular number.
Solution. In the formula, we will put n = 50. Then n + 1 = 51. Therefore
the sum is
(50 51) = (2550) = 1275.
Problem 2. What is the 200th triangular number?
(200 201) = (40,200) = 20,100.
Square numbers













Above is a 5 by 5 square array. 25 is a square number. If we call 1 the first
square number, then by adding what gnomons was 25 produced?

To 1 we added 3 to produce 4.
To 4 we added 5 to produce 9.
To 9 we added 7 to produce 16.
To 16 we added 9 to produce 25.
The gnomons of the squares are the odd numbers.
Every square number is a sum of consecutive odd numbers.
1.
1 + 3 = 4.
1 + 3 + 5 = 9.
1 + 3 + 5 + 7 = 16.
1 + 3 + 5 + 7 + 9 = 25.
Now, how are squares related to triangles?













Every square is composed of two consecutive triangles
Triangles: 1 3 6 10 15 21 28 . . .
1 + 3 = 4.
3 + 6 = 9.
6 + 10 = 16.
10 + 15 = 25.
15 + 21 = 36.
The sum of consecutive cubes
When the same number is repeated as a factor three times -- as 2 2 2 --
we call the product the 3rd power of that base, commonly called a cube.
(That is analogous to the volume of the solid figure called a cube.)
Here number 4:

Upon repeatedly adding it four times --

-- we have the 2nd power, or the square, of 4.
And upon repeatedly adding that four times --

-- we have the 3rd power, or the cube, of 4.












It will be convenient for the moment to express the cube of a number
with the exponent 3.
1
3
= 1.

2
3
= 8.
3
3
= 27.

4
3
= 64.
We come now to one of the most remarkable facts in the structure of
the natural numbers:
The sum of n consecutive cubes is equal to the square
of the nth triangle.
1
3
+ 2
3
+ 3
3
+ . . . + n
3
= (1 + 2 + 3 + . . . + n)
2
.
To see that, we will begin here:
The difference between the squares of two consecutive triangles
is a cube.
Triangles: 1 3 6 10 15 21 28
3
2
1
2
= 2
3
.
6
2
3
2
= 3
3
.
10
2
6
2
= 4
3
.












15
2
10
2
= 5
3
.
The base of the cube is the difference of the two triangles.
Look -- here is the cube of 4:

From it, let us separate this rectangular array --

-- and reposition it here:

Then we have the square of side 10 --

-- minus the square of side 6
The difference of the squares of those two consecutive triangles is equal to a cube.












Therefore,

The sum of those four cubes is equal to the square of the fourth triangle.
Alternatively, since every square number is the sum of consecutive odd
numbers, the square of a triangular number will be the sum of consecutive
odd numbers.
1
2
= 1
(1 + 2)
2
= 1 + 3 + 5 = 9
(1 + 2 + 3)
2
= 1 + 3 + 5 + 7 + 9 + 11 = 36
(1 + 2 + 3 + 4)
2
= 1 + 3 + 5 + 7 + 9 + 11 + 13 + 15 + 17 + 19 = 100
Therefore, the difference of two squares -- each cube -- will also be the
sum of consecutive odd numbers, although not starting with 1.
1 = 1
8 = 3 + 5
27 = 7 + 9 + 11
64 = 13 + 15 + 17 + 19
Again, the sum of those four cubes is equal to the square of the fourth
triangle.













We proved that by mathematical induction
in Topic 27 of Precalculus.





THE LAW OF COSINES
Proof of the Law of Cosines

WE USE THE LAW OF COSINES AND THE LAW OF SINES to solve triangles that
are not right-angled. Such triangles are called oblique triangles. The
Law of Cosines is used much more widely than the Law of Sines.
Specifically, when we know two sides of a triangle and their included angle, then
the Law of Cosines enables us to find the third side.

Thus if we know sides a and b and their included angle , then the Law of Cosines states:












c = a + b 2ab cos
(The Law of Cosines is a extension of the Pythagorean theorem; because if
were a right angle, we would have c = a + b.)
Example 1. In triangle DEF, side e = 8 cm, f = 10 cm, and the angle at D
is 60. Find side d.

Solution.. We know two sides and their included angle. Therefore,
according to the Law of Cosines,
d = e + f 2ef cos 60
d = 8 + 10 2 8 10 , since cos 60 = ,
d = 164 80
d = 84. d = .
Problem 1. In the oblique triangle ABC, find side b if side a = 5 cm,
c = cm, and they include and angle of 45. No Tables.














b = a + c 2ac cos 45
= 5 + ( ) 2 5 cos 45
= 25 + 2 10 , since cos 45 = ,
= 25 + 2 10, ( = 2)
= 17.
b = cm.
Problem 2. In the oblique triangle PQR, find side r if side p = 5 in, q = 10
in, and they include and angle of 14. (Table)

r = 5 + 10 2 5 10 cos 14
= 25 + 100 100(.970), from the Table.
= 125 97
= 28.
r = in.



Example 2. In Example 1, we found that d = , which is approximately
9.17.













Use the Law of Sines to complete the solution of triangle DEF. That is, find angles E and F.
Solution. To find angle F, we have this version of
Unknown
Known
:
sin F
sin D
=
f
d


sin F
sin 60
=
10
9.17


sin F = (.866)
10
9.17
from the Table,

.944 with the aid of a calculator.

Therefore, on inspecting the Table for the angle whose sine is closest to .944,












Angle F 71.
And therefore
Angle E = 180 (71 + 60)


= 180 131


= 49.
And so using the Laws of Sines and Cosines, we have completely solved the
triangle.
The Law of Cosines is valid when the included angle is obtuse. But
in that case, the cosine is negative. See Topic 15.

Proof of the Law of Cosines






Let ABC be a triangle with sides a, b, c. We will show












c = a + b 2ab cos C.
(The trigonometric functions are defined in terms of a right-angled
triangle. Therefore it is only with the aid of right-angled triangles that we
can prove anything )
Draw BD perpendicular to CA, separating triangle ABC into the two right
triangles BDC, BDA. BD is the height h of triangle ABC.
Call CD x. Then DA is the whole b minus the segment x: b x.
Also, since
x
a
= cos C,

then
x = a cos C . . . . . . . (1)
Now, in the right triangle BDC, according to the Pythagorean theorem,

h + x = a,
so that
h = a x. . . . . . (2)
In the right triangle BDA,
c = h + (b x)
= h + b 2bx + x.
(The square of a
binomial)
For h, let us substitute line (2):
c = a x + b 2bx + x
= a + b 2bx.












Finally, for x, let us substitute line (1):
= a + b 2b a cos C.
That is,
c = a + b 2ab cos C.
This is what we wanted to prove.

In the same way, we could prove that
a = b + c 2bc cos A
and
b = a + c 2ac cos B.

This is the Law of Cosines.



THE LAW OF SINES
Statement of the law of sines
The sine of an obtuse angle
The ambiguous case
Proof of the law of sines

This is a topic in traditional trigonometry. It does not come up in
calculus.












THE LAW OF SINES allows us to solve triangles that are not right-
angled, and are called oblique triangles. It states the following:
The sides of a triangle are to one another
in the same ratio as the sines of their opposite angles.

a : b : c = sin A : sin B : sin C.
Specifically, side a is to side b as the sine of angle A is to the sine of
angle B.
a
b
=
sin A
sin B
.
Similarly,
b
c
=
sin B
sin C
.
And so on, for any pair of sides and their opposite angles.
The law of sines is a theorem about the
geometry of any triangle. As any theorem of
geometry, it can be enunciated. The algebraic
statement of the law --
sin A
a
=
sin B
b
=
sin C
c












-- cannot be verbalized. sin A moreover, which is a number,
does not have a ratio to a, which is a length.
Secondly, to prove that algebraic form, it is necessary to state
and prove it correctly geometrically, and then transform it
algebraically.
Example 1. a) The three angles of a triangle are 40, 75, and 65. In
what ratio are the three sides? Sketch the figure and place the ratio
numbers.
Solution. To find the ratios of the sides, we must evaluate the sines of
their opposite angles. From the Table,
sin 40 = .643
sin 75 = .966
sin 65 = .906
These are the ratios of the sides opposite those angles:

Notice that we may express the ratios as ratios of whole numbers; we may ignore the
decimal points. Why? Because we have multiplied each side by the same number,
namely 1000. (The theorem of the same multiple.)
b) When the side opposite the 75 angle is 10 cm, how long is the side
opposite the 40 angle?













Solution. Let us call that side x. Now, according to the Law of Sines, in
every triangle with those angles, the sides are in the ratio 643 : 966 : 906.
Therefore,
x
10
=
643
966
x = 10
643
966
With the aid of a calculator,
x 6.656 cm.
Problem 1. The three angles of a triangle are A = 30, B = 70, and
C = 80.
a)

In what ratio are the three sides? Sketch the triangle and place
those ratio numbers. (Table)

sin 30 = .500
sin 70 = .940
sin 80 = .985












b) If side a = 5 cm, find sides b and c.
In every triangle with those angles, the sides are
in the ratio 500 : 940 : 985. But the side
corresponding to 500 has been divided by 100.
Therefore, each side will be divided by 100. Side
b will equal 9.4 cm, and side c = 9.85 cm.



The sine of an obtuse angle
The trigonometric functions (sine, cosine, etc.) are defined in a right
triangle in terms of an acute angle. What, then, shall we mean by the sine
of an obtuse angle ABC?

The sine of an obtuse angle is defined to be the sine of its supplement.
For example,
sin 120 = sin 60 =

2
.
(Topic 5.)
To see why we make this definition, let ABC be an obtuse angle, and













draw CD perpendicular to AB extended.
Then we define the sine of angle ABC as follows:
sin ABC =
CD
CB
.
But that is the sine of angle CBD -- opposite-over-hypotenuse. And
angle CBD is the supplement of angle ABC. So, by the sine of an obtuse
angle we mean the sine of its supplement.
For example,

sin 110 = sin (180 110)
= sin 70
= .940, from the Table.
Problem 1. Evaluate the following:
a) sin 135 = sin 45 = (Topic 4, Example 1)












b) sin 127 = sin (180 127) = sin 53 = .799
(From the Table)
Problem 2.
a) The three angles of a triangle are 105, 25, and 50. In
what ratio are
a) the sides? Sketch the triangle.
sin 105 = sin (180 - 105)
= sin 75
= .966, from the Table.

sin 25 = .423

sin 50 = .766
Therefore, the sides opposite those angles are in the ratio
966 : 423 : 766

b) If the side opposite 25 is 10 cm, how long is the side
opposite 50?













x
10
=
766
423
.
With the aid of a calculator, this implies:
x = 10
766
423
18.1 cm
The ambiguous case

The so-called ambiguous case arises from the fact that an acute angle and
an obtuse angle have the same sine. If we had to solve
sin x = ,
for example, we would have
x = 45 or x = 135.
(Topic 4, Example 1.)
In the following example, we will see how this ambiguity could arise.

In triangle ABC, angle A = 30, side a = 1.5 cm, and side b = 2 cm. Let
us use the law of sines to find angle B.












sin B
sin 30
=
2
1.5

Since sin 30 =

sin B =
20
15
=
10
15
=
2
3


.666
On inspecting the Table for the angle whose sine is closest to .666, we
find
B 42.
But the sine of an angle is equal to the sine of its supplement. That is,
.666 is also the sine of 180 42 = 138.
This problem has two solutions. Not only is angle CBA a solution,

but so is angle CB'A, which is the supplement of angle CBA. (We can
see that it is the supplement by looking at the isosceles triangle CB'B;












angle CB'A is the supplement of angle CB'B, which is equal to angle
CBA.)
Given two sides of a triangle a, b, then, and the acute angle opposite one of them, say
angle A, under what conditions will the triangle have two solutions, only one solution, or
no solution?

Let us first consider the case a < b. Upon applying the law of sines, we
arrive at this equation:

Let us first consider the case a < b. Upon applying the law of sines, we arrive
at this equation:
1) sin B = sin A
b
a
.
Now, since
h
b
= sin A , where h is the height of the triangle (Fig. 1),
then
b sin A = h.
On replacing this in the right-hand side of equation 1), it becomes
sin B =
h
a
.


There are now three possibilities:













h
a
< 1, which implies h < a (Fig. 1),
h
a
= 1, which implies h = a (Fig. 2),
h
a
> 1, which implies h > a (Fig. 3).
In the first of these -- h or b sin A < a -- there will be two triangles.
In the second -- h or b sin A = a -- there will be one right-angled triangle.
And in the third -- h or b sin A > a -- there will be no solution.

Example 2. Let a = 2 cm, b = 6 cm, and angle A = 60. How many
solutions are there for angle B?
Answer. We must calculate b sin A. If it is less than a, there will be
two solutions. If it is equal to a, there will be one solution. And if it is
greater than a, there will be no solution.
Now, sin 60 =

2
. (Topic 5.) Therefore,












b sin A = 6

2
= 3 .
Since a = 2, then b sin A > a. ( 1.732). There is no solution.


Finally, we will consider the case in which angle A is acute, and a > b.

In this case, there is only one solution, namely, the angle B in
triangle CBA. For, in triangle CAB', the angle CAB' is obtuse.
Problem 3. In each of the following, find the number of solutions.
a) Angle A = 45, a = , b = 2.
Since < 2, this is the case a < b. sin 45 =
/2. Therefore, b sin A = 2 /2 = , which
is equal to a. There is therefore one solution:
angle B is a right angle.
b) Angle A = 45, a = 1.8, b = 2.
Again, a < b. b sin A = 2 /2 = , which
is less than a. Therefore there are two
solutions.
c) Angle A = 45, a = 2, b = 1.5.












Here, a > b. Therefore there is one
solution.
d) Angle A = 45, a = 1.4, b = 2.
a < b. b sin A = 2 /2 = , which is
greater than a. Therefore there are no
solutions.


Proof of the law of sines

The sides of a triangle are to one another in the same
ratio as the sines
of their opposite angles.
Since the trigonometric functions are defined in terms of a right-
angled triangle, then it is only with the aid of right-angled triangles
that we can prove anything



















In triangle ABC, then, draw CD perpendicular to AB. Then CD is
the height h of the triangle. The height now separates triangle ABC
into two right triangles, CDA and CDB.
We will now show that
sin A
sin B
=
a
b
.

Now, in triangle CDA,
sin A =
h
b
.
While in triangle CDB,
sin B =
h
a
.
Therefore,
sin A
sin B
=
h/b
h/a
=
h
b

a
h
=
a
b
.
This is what we wanted to prove.
In the same way, we could prove that
sin B
sin C
=
b
c
and so on, for any pair of angles and their opposite sides.
























ANGLES AND THEIR MEASUREMENT
The definition of an angle
Degree measure
Standard position
The four quadrants
Coterminal angles
TRIGONOMETRY, as it actually used in calculus and science, is not
about solving triangles. It becomes the mathematical description
of things that rotate or vibrate, such as light, sound, the paths of
planets about the sun, or satellites about the earth. It is necessary
therefore to have angles of any size, and to extend to them the












meanings of the trigonometric functions. We do that in Topic
15.
Angles
An angle is the opening that two straight lines form when they meet.

When the straight line FA meets the straight line EA, they form
the angle we name as angle FAE. Letter A, which we place in the
middle, labels the point where the two lines meet, and is called the
vertex of the angle. When there is no confusion as to which point is
the vertex, we may speak of "the angle at the point A," or simply
"angle A."
The two straight lines that form an angle are called its sides. And the size of the angle
does not depend on the lengths of its sides. We can see that in the figure above. For if
the point C is in the same straight line as FA, and B is in the same straight line as EA,
then angles CAB and FAE are the same angle.
Now, to measure an angle, we place the vertex at the center of a
circle













(we call that a central angle), and we measure the length of the arc --
that portion of the circumference -- that the sides intercept. We then
determine what relationship that arc has to the entire circumference,
which is an agreed-upon number. (In degree measure that number is
360; in radian measure it is 2.)
The measure of angle A, then, will be length of the arc BC relative to the
circumference BCD -- or the length of arc EF relative to the circumference EFG. For
in any circles, equal central angles determine a unique ratio of arc to circumference.
(See the theorem of Topic 14. It is stated there in terms of the ratio of arc to radius,
but the circumference is proportional to the radius: C = 2r.)
There are two systems for measuring angles. One is the well-
known system of degree measure. The other is the strictly
mathematical system called radian measure, which we take up in the
next Topic.
Degree measure
To measure an angle in degrees, we imagine the circumference of a
circle divided into 360 equal parts, and we call each of those equal
parts a "degree." Its symbol is a small 0: 1 -- "1 degree." The full
circle, then, will be 360. But why the number 360? What is so
special about it? Why not 100 or 1000?
The answer is two-fold. First, 360 has many divisors, and therefore it will have many
whole number parts. It has an exact half and an exact third -- which a power of 10












does not have. 360 has a fourth part, a fifth, a sixth, and so on. Those are natural
divisions of the circle, and it is very convenient for their measures to be whole
numbers. (Even the ancients didn't like fractions )
Secondly, 360 is close to the number of days in the astronomical year: 365.

The measure of an angle, then, will be as many degrees as its
sides include. To say that angle BAC is 30 means that its sides
enclose 30
of those equal divisions. Arc BC is
30
360
of the entire circumference.
So, when 360 is the measure of a full circle, then 180 will be half a circle. 90 --
one right angle -- will be a quarter of a circle; and 270 will be three quarters of a
circle: three right angles.
Let us now see how we deal with angles in the x-y plane.
Standard position













We say that an angle is in standard position when its vertex A is at the
origin of the cordinate system, and its Initial side AB lies along the
positive x-axis. We say that AB has "swept out" the angle BAC, and
that AC is its Terminal side.

We now think of the terminal side AC as rotating about the fixed point A. When it
rotates in a counter-clockwise direction, we say that the angle is positive. But when it
rotates in a clockwise direction, as AC', the angle is negative.
When the terminal side AC has rotated 360, it has completed one full revolution.
Problem 1. How many degrees corresponds to each of the
following?












To see the answer, pass your mouse over the colored area.
To cover the answer again, click "Refresh" ("Reload").
a) A third of a revolution A third of 360 = 360 3 = 120
b) A sixth of a revolution 360 6 = 60
c) Five sixths of a revolution 5 60 = 300
d) Two revolutions 2 360 = 720
e) Three revolutions 3 360 = 1080
f) One and a half revolutions 360 + 180 = 540
Example 1. 30 is what fraction of a circle, or of one revolution?
Answer. 30 is
30
360
of a revolution:
30
360
=
3
36
=
1
12
(Skill in Arithmetic, Equivalent Fractions, Question 5.)
Problem 2. What fraction of a revolution is each of the following?
a) 60
60
360
=
6
36
=
1
6
b) 45
45
360
=
5
40
=
1
8
c) 72
72
360
=
8
40
=
1
5












Example 2. If the diameter of a circle is 16 cm, how long is the arc
intercepted by a central angle of 45?

Answer. 45 is one eighth of a full circle. (It is half of 90 , which is
one quarter.) Now, the full circumference of this circle is
C = D = 3.14 16 cm.
(Topic 9.) The intercepted arc is one eighth of the circumference:
3.14 16 8 = 3.14 2 = 6.28 cm
Problem 3. If the diameter of a circle is 20 in, how long is the arc
intercepted by a central angle of 72?
We saw in Problem 2c) that 72 is one fifth of a circle. The
circumference of this circle is C = D = 3.14 20 in. The
intercepted arc is one fifth of this: 3.14 20 5 = 3.14 4 =
12.56 in.
The four quadrants













The x-y plane is divided into four quadrants. The angle begins in its
standard position in the first quadrant ( I ). As the angle continues --
in the counter-clockwise direction -- we name each succeeding
quadrant.
Why do we name the quadrants in the
counter clockwise direction? Because in
what we call the "first" quadrant, the
algebraic signs of x and y are positive.
Problem 4. In which quadrant does each angle terminate?
a) 15 I b) 15 IV c) 135 II
d) 390 I. 390 = 360 + 30 e) 100 III
f) 460 III. 460 = 360 100
g)
710
IV. 710 is 10 less than two revolutions, which are
720.

Coterminal angles
Angles are coterminal if, when in the standard position, they have
the same terminal side.













For example, 30 is coterminal with 360 + 30 = 390. They have the same
terminal side. That is, their terminal sides are indistinguishable.
Any angle is coterminal with + 360 -- because we are just going around the
circle one complete time.
90 is coterminal with 270. Again, they have the same
terminal side.
Notice: 90 plus 270 = 360. The sum of the absolute values of those
coterminal angles completes the circle.
Problem 5. Name the non-negative angle that is coterminal with
each of these, and is less than 360.
a) 360 0 b) 450 90. 450 = 360 + 90
c) 20 340 d) 180 +180 e) 270 90
f) 720 0. 720 = 2 360
g) 200 160































RADIAN MEASURE
Radians into degrees
Degrees into radians
Coterminal angles
The multiples of

IN THE RADIAN SYSTEM of angular measurement, the measure of one
revolution is 2.
(In the next Topic, Arc Length, we will see the actual definition of radian measure.)













Half a circle, then, is . And, most important, each right angle is half
of :

2
.
Three right angles will be 3

2
=
3
2
.
Five right angles will be
5
2
. And so on.

Radians into degrees
The following radian measures come up frequently, and the student
should know their degree equivalents:

(Topic 4 and Topic 5.)













4
is half of

2
, a right angle, and so it is equal to 45. (Skill in
Arithmetic, Lesson 27, Question 4.)
Equivalently,

4
is of one quarter, or half of half, of , which is 180.

3
is a third of , and so is equal to 180 3 = 60.

6
is a sixth of , and so is equal to 180 6 = 30.

5
4
= 5

4
= 5 45 = 225.
2
3
is a third of 2. A third of a revolution = 360 3 = 120.

Problem 1. Convert each of these radian measures into degrees.
Problem 1. The student should know these.












To see the answer, pass your mouse over the colored area.
To cover the answer again, click "Refresh" ("Reload").
a) 180 b)

2
90 c)

3
60 d)

6
30 e)

4
45
Problem 2. Convert each of these radian measures into degrees.
a)

8
22.

8
is half of

4
.
b)
2
5
72.
2
5
is a fifth of 2 , which is a fifth of a 360.
c)
7
4
= 7

4
= 7 45 = 315
d)
9
2
= 9

2
= 9 90 = 810
e)
4
3
= 4

3
= 4 60 = 240
f)
5
6
= 5

6
= 5 30 = 150
g)
7
9














Problem 3. Evaluate the following. (See Topic 4 and Topic 5.)
a) cos

6
=

2
b) sin

6
=
1
2
c) tan

4
= 1

d) cot

3
=
1

e) csc

6
= 2 f) sec

4
=


Problem 4. In terms of radians, what angle is the complement of an
angle ?

2


Problem 5. A function of any angle is equal to the cofunction of its complement.
(Topic 3.) Therefore, in terms of cofunctions:
a) sin = cos (

2
) b) cot = tan (

2
)
c) sec (

2
) = csc
Degrees into radians
360 = 2. 180 = .
When we write, 180 = , we mean that it equals radians, which
is approximately 3.14 radians. However, we normally omit the word












radians. For, as we will see in the next Topic, Arc length, the radian
measure can be any number.
Example 1. Convert 120 into radians.
Solution. We can go from what we know to what we don't know.
Since
60 =

3
, then
120 = 2 60 = = 2

3
=
2
3
.
Or, since 120 is a third of 360, which is 2, then
120 =
2
3
.
Example 2. 225 = 180 + 45 = +

4
=
5
4
In general, proportionally,

so that

Example 3. Change 140 to radians.












Solution.
140
180
=
7
9
=
7
9
,
upon dividing both the numerator and denominator first by 10 and
then by 2. (Lesson 21 of Arithmetic, and Lesson 1.)
Problem 6. Change each of the following into radians.

a) 0
0
radians

b) 180


c) 90

2

d) 45

4

e) 270
3
2
f) 60

3
g) 30

6
h) 720 = 2 360 = 2 2 = 4
i) 210 = 7 30 = 7

6
=
7
6
j) 300 = 5 60 = 5

3
=
5
3
k) 135 = 90 + 45 =

2
+

4
=
3
4
l) 72 =
72
180
=
2
5
=
2
5
Coterminal angles
Angles are coterminal if they have the same terminal side.













is coterminal with . They have the same terminal side.
Notice that
+ = 2,
so that
= 2 . . . . . . . . (1)
Example 4. Name in radians the non-negative angle that is
coterminal
with
2
5
, and is less than 2.
Answer. Let us call that angle . Then according to line (1),
= 2
2
5
=
10 2
5
=
8
5
See Lesson 23 of Algebra, Example 7.
Problem 7. Name in radians the non-negative angle that is coterminal
with each of the following, and is less than 2.
a)

6
= 2

6
=
12
6
=
11
6












b)
3
4
= 2
3
4
=
8 3
4
=
5
4
c)
4
3
= 2
4
3
=
6 4
3
=
2
3
The multiples of
Starting at 0, let us go around the circle a half-circle at a time. We will
then have the following sequence, which are the multiples of :
0, , 2, 3, 4, 5, etc.

The point to see is that the odd multiples of ,
, 3, 5, 7, etc.
are coterminal with . While the even multiples of ,
2, 4, 6, etc.
are coterminal with 0.
If we go around in the negative direction,













we can make a similar observation.
Problem 8. Name in radians the non-negative angle that is coterminal
with each of the following, and is less than 2.
a) - b) -2 0 c) -3 d) -4 0 e) -5
f) 3 g) 4 0 h) 5 i) 6 0 j) 7



Arc Length
The definition of radian measure
s = r
The unit circle

An angle of 1 radian
Proof of the theorem













IT IS CONVENTIONAL to let the letter s symbolize the length of an
arc, which is called arc length. We say in geometry that an arc
"subtends" an angle ; literally, "stretches under."
Now the circumference of a circle is an arc length. And the ratio of the circumference
to the diameter is the basis of radian measure. That ratio is the definition of .
=
C
D
.
Since D = 2r, then
=
C
2r
or,
C
r
= 2 .
That ratio -- 2 -- of the circumference of a circle to
the radius, is called the radian measure of 1 revolution,
which are four right angles at the center. The circumference
subtends those four right angles.














Radian measure of =
s
r
Thus the radian measure is based on ratios -- numbers -- that are actually found in the
circle. The radian measure is a real number that names the ratio of a curved line to a
straight, of an arc to the radius. For, the ratio of s to r does determine a unique central
angle .
Theorem. In any circles the same ratio of arc length to radius
determines a unique central angle that the arcs subtend.

Proportionally,

if and only if
1 = 2.












We will prove this theorem below.
Example 1. If s is 4 cm, and r is 5 cm, then the
number
4
5
, i.e.
s
r
, is
the
radian measure of the central angle.
At that central angle, the arc is four fifths of the radius.
Example 2. An angle of .75 radians means that the arc is three
fourths of the radius. s = .75r
Example 3. In a circle whose radius is 10 cm, a central angle
intercepts an arc of 8 cm.

a) What is the radian measure of that angle?
Answer. According to the definition:
=
s
r
=
8
10
= .8
b) At that same central angle , what is the arc length if the radius is
b) 5 cm?













Answer. For a given central angle, the ratio of arc to radius is the
same. 5 is half of 10. Therefore the arc length will be half of 8:
4cm.
Example 4.
a) At a central angle of 2.35 radians, what ratio has the arc to the
radius?
Answer. That number is the ratio. The arc is 2.35 times the radius.
b) In which quadrant of the circle does 2.35 radians fall?

Answer. Since = 3.14,
then

2
is half of that: 1.57.

3
2
= 3.14 +
1.57
= 4.71.
An angle of 2.35 radians, then, is greater than 1.57 but less that 3.14. It falls in the
second quadrant.

s = r
c) If the radius is 10 cm, and the central angle is 2.35 radians, then
how
c) long is the arc?












Answer. We let the definition of ,
=
s
r
become a formula for finding s :
s = r
Therefore,
s = 10 2.35 = 23.5 cm
Because of the simplicity of that formula, radian measure is
used exclusively in theoretical mathematics.

The unit circle
Since in any circle the same ratio of arc to radius determines a uniqe
central angle, then for theoretical work we often use the unit circle,
which is a circle of radius 1: r = 1.

In the unit circle, the length of the arc s is equal to the radian
measure. The length of that arc is a real number x.
s = r = 1 x = x.
We can identify radian measure, then, as the length x of an arc
of the unit circle. And it is here that the term trigonometric
"function" has its full meaning. For, corresponding to each real
number x -- each radian measure, each arc -- there is a unique value












of sin x, of cos x, and so on. The definition of a function is satisfied.
(Topic 3 of Precalculus.)
Moreover, when we draw the graph of y = sin x (Topic 18), we can imagine the unit
circle rolled out in both directions onto the x-axis, and in that way marking the
cordinates , 2, , 2, and so on, on the x-axis.
Because radian measure can be
identified as an arc, the inverse
trigonometric functions have their
names. "arcsin" is the arc -- the radian
measure -- whose sine is a certain
number.

The ratio
sin x
x

In the unit circle, the opposite side AB is sin x.
sin x =
AB
1
= AB.
One of the main theorems in calculus concerns the ratio
sin x
x
for
very small values of x. And we can see that when the point A on
the circumference is very close to C -- that is, when the central












angle AOC is very, very small -- then the opposite side AB will be
virtually indistinguishable from the arc length AC. That is,
sin x x

sin x
x
1.
An angle of 1 radian
An angle of 1 radian refers to a central angle whose subtending arc
is equal in length to the radius.

That is often cited as the definition of radian measure. Yet it remains to be
proved that if an arc is equal to the radius in one circle, it will subtend the same
central angle as an arc equal to the radius in another circle. We cannot avoid the
main theorem.
In addition, although it is possible to define an "angle of 1 radian," does such an
angle actually exist? Is it possible to draw one -- a curved line equal to a straight
line? Or is that but another example of fantasy mathematics?
See First Principles of Euclid's
Elements, Commentary on the
Definitions; see in particular that a
definition asserts only how a word or
a name will be used. It does not












assert that what has been defined
exists.
Problem 1.
a) At a central angle of


5
, approximately what ratio has the arc to
the
a) radius? Take 3.
The radian measure

5
is that ratio . Taking 3, then the
arc is approximately three fifths of the radius.
b) If the radius is 15 cm, approximately how long is the arc?
s = r 15
3
5
= 9 cm
Problem 2. In a circle whose radius is 4 cm, find the arc
length intercepted by each of these angles. Again, take 3.
a)

4
s = r 4
3
4
= 3 cm
b)

6
s = r 4
3
6
= 4 = 2 cm
c)
3
2
s = r 4
3 3
2
= 4
9
2
= 2 9 = 18 cm
d) 2. (Here, the arc length is the entire circumference!)
s = r = 4 2 4 6 = 24 cm












Problem 3. In which quadrant of the circle does each angle,
measured in radians, fall? (See the figure above.)
a) = 2 2 radians are more than

2
but less than . (See the
figure above.) Therefore, = 2 falls in the
second quadrant.
b) = 5 5 radians are more than
3
2
but less than 2. (See the
figure above.) Therefore, = 5 falls in the
fourth quadrant.
c) = 14 14 radians are more than 2 revolutions, but slightly
less than 2: 6.28 + 6.28 = 12.56. (See the
figure above.) Therefore, = 14 falls in the
first quadrant.

Proof of the theorem
In any circles the same ratio of arc length to radius
determines a unique central angle that the arcs
subtend;
and conversely, equal central angles determine the
same ratio
of arc length to radius.













Proportionally,

if and only if
1 = 2.
For,

if and only if

Now 2r is the circumference of each circle. And each circumference is
an "arc" that subtends four right angles at the center.













But in the same circle, arcs have the same ratio to one
another as the central angles they subtend. (Theorem 16.)
Therefore,

and

Therefore, according to line (1),

if and only if
1 = 2.
Therefore, the same ratio of arc length to radius
determines a unique central angle that the arcs subtend.
Which is what we wanted to prove.



ANALYTIC TRIGONOMETRY
THE UNIT CIRCLE
The definitions
The signs in each quadrant












Quadrantal angles
The unit circle
ANALYTIC TRIGONOMETRY is an extension of right triangle
trigonometry. It takes place on the x-y plane. For, trigonometry as
it is actually used in calculus and physics, is not about solving
triangles. It becomes the mathematical description of things that
rotate or vibrate, such as light, sound, the paths of planets about
the sun, or satellites about the earth. It is necessary therefore to
have angles of any size, and to extend to them the meanings of the
trigonometric functions. We do that now.

Let a radius of length r sweep out an angle in standard position, and let its endpoint
have cordinates (x, y). The question is: How shall we now define the six
trigonometric functions of ?
We will take our cue from the first quadrant. In that quadrant,
a radius r will terminate at a point (x, y).
Those cordinates define a right triangle.
The right-triangle definitions (Topic 2) of
the six trigonometric functions follow.













sin =
y
r
csc =
r
y

cos =
x
r
sec =
r
x

tan =
y
x
cot =
x
y
According to the Pythagorean theorem,

In this way we extend the meaning of the trigonometric functions to angles that
terminate in any quadrant. It is in terms of the cordinates (x, y) of the endpoint of a
distance r from the origin.
But before we give an example, consider this question:

Will a function of depend of the length of r?
To see the answer, pass your mouse over the colored area.
To cover the answer again, click "Refresh" ("Reload").
Answer the question yourself first!












No, it will not. The functions are in terms of the ratios
of the sides, not their lengths.

Say that AB, AC are two different radii. But triangles ABD, ACE are similar.
(Theorem 15) Proportionally,
DB : BA = EC : CA
sin -- opposite over hypotenuse -- does not depend of the
length of the radius. And similarly for the remaining functions.
Therefore, we may choose any radius we please. Typically, we take r
= 1. That is called the unit circle, as we shall see.
The trigonometric functions in fact depend only on the angle -- and it is for that
reason we say that they are functions of .
Example 1. A straight line inserted at the origin terminates at the
point (3, 2) as it sweeps out an angle in standard position. Evaluate
all six functions of .
Answer. x = 3, y = 2. Therefore, according to the definitions:

sin =
y
r
=
2

csc =
r
y
=

2













cos =
x
r
=
3

sec =
r
x
=

3

tan =
y
x
=
2
3
cot =
x
y
=
3
2
Problem 1. A straight line from the origin sweeps out an angle ,
and it terminates at the point (3, 4). Evaluate the six functions of
.
x = 3, y = 4. Therefore,

sin =
4
5
csc =
5
4

cos =
3
5
sec =
5
3

tan =
4
3
cot =
3
4

Problem 2. The signs in each quadrant.













a) The algebraic sign of sin will always be the sign of which
a) cordinate? y, because sin = y/r, and r is always positive.
a) Therefore, in which quadrants will sin -- y -- be positive? I and
II.
a) In which quadrants will sin be negative? III and IV.
b) The algebraic sign of cos will always be the sign of which
b) cordinate? x, because cos = x/r, and again, r is always
positive.
a) Therefore, in which quadrants will cos -- x -- be positive? I
and IV.
a) In which quadrants will cos be negative? II and III.
c) In which quadrants will the algebraic sign of tan (y/x) be
positive?
I and III. x and y will have the same signs.
d) In which quadrants will the algebraic sign of tan be negative?
II and IV. x and y will have opposite signs.
e) csc will have the same sign as which other function?
sin , because they are reciprocals.












f) sec will have the same sign as which other function?
cos
g) cot will have the same sign as which other function?
tan

Quadrantal angles
A quadrantal angle is an angle that terminates on the x- or y-axis.
Problem 3.
a) What are the quadrantal angles in degrees?
0, 90, 180, 270; and angles coterminal with them.
b) What are the quadrantal angles in radians?
0,

2
, ,
3
2
; and angles coterminal with them.
c) When an angle terminates on the x-axis, what is the value of the
c) y-cordinate? 0. On the x-axis, y = 0.
d) When an angle terminates on the y-axis, what is the value of the
c) x-cordinate? 0. On the y-axis, x = 0.
Now, it is a fact of arithmetic that there is no number with
denominator 0.
n
0
has no meaning.

Therefore, wherever a trigonometric function has a denominator -- x or y -- equal
to 0, the function will not exist at that quadrantal angle.












For example,
tan =
y
x
Wherever x = 0, tan will not exist. Where does x = 0? When the angle
terminates on the y-axis.

tan will not exist at =

2
and =
3
2
or

2
, which is coterminal.
Those values of will be singularities of tan . (Topic 18 of Precalculus.)
Problem 4. For which quadrantal angles do the following
functions not exist?
a) cot cot =
x
y
. Therefore, whenever y = 0 -- that is, on
the x-axis -- cot will not exist. cot will not exist at
= 0 and = .
b) sec
sec =
r
. Therefore, sec will not exist where x = 0,












x
which is on the y-axis. sec will not exist at =

2
and =
3
2
.
c) sin
sin =
y
r
. But r is never 0. There is no angle for which
sin does not exist.

The unit circle
The trigonometric functions are functions only of the angle .
Therefore we may choose any radius we please, and the simplest is a
circle of radius 1, the unit circle.

On the unit circle the functions take a particularly simple
form. For example,
sin =
y
1
= y.












cos =
x
1
= x.
The value of sin is the y-cordinate of the endpoint of the unit
radius The value of cos is the x-cordinate

With regard to quadrantal angles, the unit circle illustrates the
following:


If a function exists at a quadrantal
angle,
it could have only the values 0,
1, or 1.
Consider sin at each quadrantal angle. We just saw that
the value of sin is the y-cordinate:
sin = y.
Therefore at each quadrantal angle, the value of sin -- of y -- is either 0, 1,
or 1.
At = 0, sin = 0.













At =

2
, sin = 1.

At = , sin = 0.

At =
3
2
, sin = 1.
To evaluate a function at a quadrantal angle, the student
should sketch a unit circle.
Problem 5. Evaluate the following. No tables
a) cos 0
cos 0 = 1. cos is equal to the x-cordinate.

b) cos 90 = 0 c) cos 180 = 1 d) cos 270 = 0
e) tan 0
tan 0 = 0. tan is equal to y/x = 0/1 = 0.
f) tan 90 1/0 does not exist. g) tan 180 = 0













h) tan 270 does not exist.
Problem 6. Evaluate the following -- if it exists. No tables.
a) cos

2

b) sin

2
= 1 c) sin = 0 d) cos = 1
e) cot 0 = x/y. Does not exist. f) cot

2
= 0
g) tan

2
Does not exist. h) sec 0 = 1/x = 1 i) csc (

2
) = 1
j) sin 2 = 0 k) sin 3 = 0 l) sin 4 = 0 m) sin () = 0
n)
cos 2
=
1
o)
cos 3
=
1
p)
cos 4
=
1
q)
cos 5
=
1
Problem 7. Explain why we can write the following, where n
could be any integer:












cos n = (1)
n

(1)
n
= 1, according as n is even or odd. If n is
even (or 0), then cos n is coterminal with 0 radians,
and (1)
n
= 1. See the unit circle. While if n is odd,
then cos n is coterminal with radians, and (1)
n

= 1.



























TRIGONOMETRIC FUNCTIONS OF ANY ANGLE
The corresponding acute angle
The main theorem
cos () and sin ()
Polar cordinates
Proof of the main theorem

HOW SHALL WE EVALUATE tan 118, for example? We will see that
we will be referred back to an acute angle.
The corresponding acute angle
Let be an angle that terminates in any quadrant. Then by the
corresponding acute angle, we mean the shortest angular distance to the x-
axis.

In each quadrant, is the corresponding acute angle of . is the shortest angular
distance to the x-axis.
Examples.
If = 120 (Second quadrant), then = 60.












If = 190 (Third quadrant), then = 10.
If = 340 (Fourth quadrant), then = 20.
Problem 1. Name the corresponding acute angle.
To see the answer, pass your mouse over the colored area.
To cover the answer again, click "Refresh" ("Reload").
a) 110 70 b) 225 45 c) 30 30

How, then, do we evaluate a function of any angle? According to the following
theorem:
A function of any angle will equal plus or minus that same
function
of the corresponding acute angle.
The sign will depend on the quadrant.
(We will prove that below.)
Example 1. Evaluate tan 120.
Answer. The corresponding acute angle of 120 is 60. Therefore,
according to the theorem,
tan 120 = tan 60 =
Now in the second quadrant, tan (y/x) is negative (Topic
15). Therefore,
tan 120 =
Example 2. Evaluate cos 195.
Answer. The corresponding acute angle of 195 (third quadrant) is
15. Therefore,












cos 195 = cos 15 = .966, from the
Table.
In the third quadrant, cos (x/r) is negative. Therefore,
cos 195 = .966
Example 3. Evaluate sec (45).
Answer. The corresponding acute angle of 45 (fourth quadrant)
is 45. Therefore,
sec 45 = sec 45 =
In the fourth quadrant, sec (r/x) is positive. Therefore,
sec 45 =

Problem 2. Draw a figure that illustrates the following.
The sine of an obtuse angle is equal to the sine of its supplement.
The cosine of an obtuse angle is equal to the negative of the
cosine
of its supplement.

The supplement, 180 , is the corresponding acute
angle!
Problem 3. Evaluate each of the following. No tables.
a) sin 150












The corresponding acute angle is 30. And in the
second quadrant, sin is positive. Therefore, sin 150 =
sin 30 = .
b) cos 135
The corresponding acute angle is 45. And in the
second quadrant, cos is negative. Therefore, cos 135
= cos 45 = .
c) tan 240
The corresponding acute angle is 60. In the third
quadrant, tan is positive. Therefore, tan 240 = tan 60
= .
d) csc (30)
The corresponding acute angle is 30. In the fourth
quadrant, csc (y/r) is negative. Therefore, csc (30)
= csc 30 = 2.
Problem 4. Use the Table to evaluate the following.
a) cos 170
The corresponding acute angle is 10. In the second
quadrant, cos is negative. Therefore, cos 170 = cos
10 = .985.
b) cos (20)
The corresponding acute angle is 20. In the fourth
quadrant, cos is positive. Therefore, cos (20) = cos
20 = .940.
c) sin (20)












The corresponding acute angle is 20. In the fourth
quadrant, sin is negative. Therefore, sin (20) =
sin 20 = .342.

In the previous problem, we see an extremely useful theorem:
cos () = cos , but sin () = sin .

No matter in which quadrant falls, has the same
corresponding acute angle. And both and fall in the same
left-or-right half of the












x-y plane. The sign of the cosine depends only on which half.
Therefore,
cos () = cos .
On the other hand, and fall in opposite top-or-bottom
halves of the plane. The sign of the sine depends on which of
those halves. Therefore, sin and sin () will have opposite
signs:
sin () = sin .
That is what we wanted to prove.
Problem 5. Use the previous theorem to evaluate the
following. No tables.
a) cos (30) = cos 30 = /2
b) cos (60) = cos 60 =
c) cos (45) = cos 45 =
d) sin (30) = sin 30 = .
e) sin (60) = sin 60 = /2
f) sin (45) = sin 45 =
Example 4. cos ( + ) = cos . Explain why.
Answer. No matter in which quadrant falls, and + will
have the













same corresponding acute angle. And + will fall in the
opposite left-or-right half of the plane. Therefore cos and cos (
+ ) will have opposite signs:
cos ( + ) = cos .
Note: cos ( + ) = cos ( + ).
Problem 6. cos ( + 5) = cos . Explain why.
plus any odd multiple of will be coterminal
with + . For, 2 is one revolution. Therefore,
( + ) + 2 -- which is + 3 -- is coterminal
with + . ( + ) + 4 -- which is + 5 -- is
coterminal with + . And so on. Therefore,
cos ( + 5) = cos ( + ) = cos .
Note: cos (5 + ) = cos ( + 5).
Polar cordinates
We can specify the position of a point P by giving its distance
r from the origin and the angle that r makes with the x-axis.
Those are called the polar cordinates of P. (r, ).













But the polar cordinates are easily related to the rectangular cordinates
(x, y). Since
x
r
= cos ,
(Topic 15) then
x = r cos .
And since
y
r
= sin ,
y = r sin .
Example 5. A radius of 8 cm sweeps out an angle of 30 in
standard position. What are the rectangular coordinates (x, y)
of the endpoint of the radius?













Answer. x = r cos = 8 cos 30 = 8

2
= 4
y = r sin = 8 sin 30 = 8 = 4
Problem 7. Radius AB of a unit circle sweeps out an angle
135. What are the coordinates of B?

The corresponding acute angle of 135 is 45. In
the second quadrant, the cosine is negative and the sine
is positive. Therefore, x = r cos = 1 cos 135 = cos
45 = .
y = r sin = 1 sin 135 = sin 45 = .












Problem 8. Radius AB of length 2 sweeps out an angle of
60. What are the coordinates of B?

cos () = cos , but sin () = sin .
Therefore, x = 2 cos (60) = 2 cos 60 = 2 = 1.
y = 2 sin (60) = 2(sin 60) = 2( ) = .

Proof of the main theorem
A function of any angle will equal plus or minus that
same function
of the corresponding acute angle.
The sign will depend on the quadrant.













First, if is a second quadrant angle, then r will terminate at a point
(a, b). The corresponding acute angle is , which is also shown in its
first quadrant position. In the second quadrant,
sin =
b
r
while in the first quadrant,
sin =
b
r
Therefore the sine of is equal to the sine of the corresponding acute
angle.
Similarly,
cos =
a
r
= cos
tan =
b
a
= tan
And so on, for the remaining functions, so that in every
case, a function of is plus or minus that same function of
.














Next, if is a third quadrant angle, so that r terminates at (a, b), then
sin =
b
r
= sin


cos =
a
r
= cos
tan =
b
a
=
b
a
= tan
And so on, so that again, each function of is plus or minus
that same function of .


Finally, if is a fourth quadrant angle, so that r terminates at (a, b),
then
sin =
b
r
= sin














cos =
a
r
= cos


tan =
b
a
= tan
Therefore again, each function of is plus or minus
that same function of the corresponding acute angle ;
which is what we wanted to prove.







LINE VALUES
A complete cycle of sin
A complete cycle of cos
The period of tan
Projections












THE LINE VALUE of a trigonometric function is a straight line whose
length represents the value of the function. For as the central
angle changes, the line value becomes a kind of "graph" of the
function during that change. It is the best way for the student to
visualize and remember each function's values, in particular those
of the sine, cosine, and tangent.
The line values
Let A be the center of a unit circle with radii
AD, AB.
From B, drop a perpendicular BC onto AD.
Draw radius AF perpendicular to DA; draw
DE, FG tangent to the circle; that is, draw
them at right angles to the radii; (Theorem
13)
and extend AB to meet those tangents at E,
G respectively.
Then, since AG is a straight line that crosses the
parallel straight lines FG, AD, the alternate angles FGA, GAD are equal. (Theorem 8)
Therefore, triangles ACB, ADE, GFA are equiangular and therefore similar.
(Theorem 15)
The line values of the functions of angle are now as follows; to
avoid scrolling, the figure below is the same:












sin
=
CB
BA
=
CB
1
= CB.
cos =
AC
BA
=
AC
1
= AC.
tan =
DE
AD
=
DE
1
= DE.
csc =
AG
FA
=
AG
1
= AG.
sec =
AE
DA
=
AE
1
= AE.












cot =
FG
FA
=
FG
1
= FG.

We now see why the tangent function (DE) is called that: It is in fact a tangent. The
secant function (AE) is a secant. It cuts the circle; from the Latin secare, to cut.
A complete cycle of sin
The student with a good imagination should now try to follow the
point B as it makes a complete circuit about the unit circle. In
particular, let us visualize what happens to CB, the line value of sin .
At = 0, the point B coincides with D, so that CB
vanishes. sin 0 = 0.
As increases in the first quadrant, CB also increases
until it reaches its maximum value at 90. CB
coincides there with the unit radius AF, so that
CB = AF = sin 90 = 1.
When enters the second quadrant, CB begins to
decrease in value. At 180, B coincides with K, and CB
vanishes again. sin 180 = 0.
In the third quadrant, CB increases in absolute value, but
the sine in that quadrant is negative. At 270, CB
coincides with AL. sin 270 has its algebraic
minimum: sin 270 = 1.
Finally, in the fourth quadrant, CB decreases in absolute
value until B coincides once again with D.
sin 360 = sin 0 = 0.
We see that sin never has a value greater than 1 (at 90), or less than 1 (at 90).
1 sin 1.













A complete cycle of cos
The line value of cos is AC. As the point B revolves, C shuttles back
and forth along the diameter DK. At 0, AC coincides with the unit
radius AD, so that cos 0 = 1.
At = 90, C coincides with A, so that AC vanishes.
cos 90 = 0.
At = 180, C reaches K. AC = AK = cos 180 = 1.
At = 270, C is back at A. cos 270 = 0.
Finally, at = 360, AC coincides again with the radius
AD. cos 360 = 1.
cos (AC) also has a maximum value of 1 (at 0), and
minimum of 1 (at 180).
1 cos 1.

The period of tan
The values of tan , like the values of all the trigonometric functions,
regularly repeat themselves. The values of tan repeat every units
(in radians). And so we say that tan has period .













For as takes on values from /2 to /2, the line value DE of
tan goes through all the real numbers from to .
The graph of a trigonometric function is
essentially its line value plotted as the
y-cordinate versus radian angle x. See
the following Topic: Graphs of the
trigonometric functions.
Projections













The straight line CD is called the vertical projection of the straight line AB. (The
straight lines BD, AC are drawn perpendicular to CD. It is as if a light were
shining on AB, and CD is the shadow.)

EF is called the horizontal projection of AB.
Now, if AB is a unit radius, then BC -- the line value of sin -- is the

vertical projection of the radius. And AC -- the line value of cos -
- is the horizontal projection.
Thus in a unit circle, sin is the vertical projection of the radius. cos is the
horizontal projection.
























Graphs of the trigonometric functions
Zeros of a function
The graph of y = sin x
The period of a function
The graph of y = cos x
The graph of y = sin ax
The graph of y = tan x

LET US BEGIN by introducing some algebraic language. When we
write "n," where n could be any integer, we mean "any multiple
of ."
0, , 2, 3, . . .
Problem 1. Which numbers are indicated by the following, where n
could be any integer?












a) 2n
To see the answer, pass your mouse over the colored area.
To cover the answer again, click "Refresh" ("Reload").
The even
multiples of :
0, 2, 4,
6, . . .
By '2n' we mean to signify an even number.
b) (2n + 1)
The odd
multiples
of :
,
3,
5,
7, . .
.
By '2n + 1' we mean to signify an odd number.
Zeros
By the zeros of sin , we mean those values of for which
sin will equal 0.
Now, where are the zeros of sin ? That is,
sin = 0 when = ?













We saw in Topic 15 on the unit circle that the value
of sin is equal to the y-coordinate. Hence, sin = 0 at
= 0 and = -- and at all angles coterminal with them. In
other words,
sin = 0 when = n.

This will be true, moreover, for any argument of the
sine function. For example,
sin 2x = 0 when the argument 2x = n;
that is, when
x =
n
2
.
Which numbers are these? The multiples of

2
:












0,

2
, ,
3
2
, . . .
Problem 2. Where are the zeros of y = sin 3x?
At 3x
= n;
that
is, at
x =
n
3
.
Which numbers are these?
The multiples of

3
.
The graph of y = sin x
The zeros of y = sin x are at the multiples of . And it is
there that the graph crosses the x-axis, because there sin
x = 0. But what is the maximum value of the graph, and
what is its minimum value?













sin x has a maximum
value of 1 at

2
, and a minimum value of 1
at
3
2
-- and at all angles
coterminal with them.


Here is the graph of y = sin x:

The height of the curve at every point is the line
value of the sine.
The independent variable x is the radian measure. x may be any real













number. We may imagine the unit circle rolled out, in
both directions, along the x-axis. (See Topic 14: Arc
Length.)
The period of a function
When the values of a function regularly repeat
themselves, we say that the function is periodic. The
values of sin regularly repeat themselves

every 2 units. Hence, sin is periodic. Its period is 2.
Definition. If, for all values of x, the value of a
function at x + p
is equal to the value at x --
If f(x + p) = f(x)
-- then we say that the function is periodic and has
period p.













The function y = sin x has period 2, because
sin (x + 2) = sin x.
The height of the graph at x is equal to the height
at x + 2 -- for all x.
Problem 3.
a) In the function y = sin x, what is its domain?
a) (See Topic 3 of Precalculus.)
x may be any real
number. < x < .
b) What is the range of y = sin x?
sin x has a minimum value of 1, and a
maximum of +1.

1

y

1













The graph of y = cos x

The graph of y = cos x is the graph of y = sin x
shifted, or translated,


2
units to the left.
For, sin (x +

2
) = cos x. The student familiar with the sum
formula can easily prove that. (Topic 20.)
Again, the height of the curve at every point
is the line value of the cosine.
The graph of y = sin ax
Since the graph of y = sin x has period 2, then
the constant a in
y = sin ax
indicates the number of periods in an interval of
length 2. (In y = sin x, a = 1.)
For example, if a = 2 --
y = sin 2x
-- that means there are 2 periods in an interval of
length 2.













If a = 3 --
y = sin 3x
-- there are 3 periods in that interval:

While if a = --
y = sin x
-- there is only half a period in that interval:

The constant a thus signifies how frequently
the function oscillates; so many radians per unit
of x.
(In physics, when the independent variable is the time t, the
constant is written as ("omega") -- sin t. is called the
angular frequency; so many radians per second.)












Problem 4.
a) For which values of x are the zeros of y = sin
mx?
At mx = n; that is, at x =
n
m
.
b) What is the period of y = sin mx?

2
m
. Since there are m periods in 2, then one period is
2
divided by m. Compare the graphs above.
Problem 5. y = sin 2x.
a) What does the 2 indicate?
In an interval of
length 2, there are 2
periods.
b) What is the period of that function?

2
2
=
c) Where are its zeros?
At x =
n
2
.
Problem 6. y = sin 6x.
a) What does the 6 indicate?












In an interval of
length 2, there are
6 periods.
b) What is the period of that function?

2
6
=

3
c) Where are its zeros?
At x =
n
6
.
Problem 7. y = sin x.
a) What does indicate?
In an interval of length 2,
there is one fourth of a
period.
b) What is the period of that function?
2


=

2














4

=

8

.
c) Where are its zeros?
At x =
n

= 4n.

The graph of y = tan x
Here is one period of the graph of y =
tan x:













Why is that the graph? Consider
the line value DE of tan x in the 4th
and 1st quadrants:













As radian x goes
from

2
to

2
, tan x takes on all real values.
That is,
for


2
< x <

2
,
< tan x < .

Quadrants IV and I constitute a
complete period of y = tan x. In
quadrant IV, tan x is negative; in
quadrant I, it is positive; and tan 0 = 0.
Again, here is the graph:













At the quadrantal
angles

2
and

2
, tan x does not exist.
Therefore
the lines x =

2
and
x =

2
are vertical asymptotes.
(Topic 18 of
Precalculus.)
Here is the complete graph of y
= tan x.

The graph of Quadrants IV and I
is repeated in Quadrant II (where tan x
is negative) and quadrant III (where tan
x is positive), and periodically along the
entire x-axis.












Problem 7. What is the period of y =
tan x?
One period is from

2
to

2
. Hence the period is the
distance between those two points:
.







INVERSE TRIGONOMETRIC FUNCTIONS
The inverse relations
The range of y = arcsin x
The range of y = arctan x
The range of y = arccos x
The range of y = arcsec x
THE ANGLES in theoretical work will be in radian measure. Thus if
we are given a radian angle,

for example, then we can evaluate a












6
function of it.
sin

6
= .
(Topic 13.)
Inversely, if we are given a value of the sine function -- -- then
the challenge is to name the angle.
sin x = .
"The sine of what angle is equal to ?"
We could answer:
"The angle whose sine is is

6
."
The algebraic abbreviation for that sentence is
"arcsin =

6
."
arcsin x is called the inverse sine function. It is the angle whose sine is x.
Strictly, arsin x is the arc whose sine is
x. Because in the unit circle, the length
of that arc is the radian measure. Topic
14.
Thus the inverse of the function
y = sin x
"The number y is the sine of the radian angle x"
is
y = arcsin x.












"y is the radian angle whose sine is the number x."
Corresponding to each trigonometric function, there is its
inverse function:
arcsin x,
arccos x,
arctan x,
arccsc x,
arcsec x,.
arccot x.
The inverse relations
If we put
f(x) = sin x
and
g(x) = arcsin x,
then according to the definition of inverse functions (Topic 19
of Precalculus):
f(g(x)) = x and g(f(x)) = x.
sin(arcsin x) = x and arcsin(sin x) = x.
In particular,
arcsin x = y

implies, on taking the inverse function -- the sine -- of both sides:

x = sin y.












Problem 1. What does each of the following imply?
To see the answer, pass your mouse over the colored area.
To cover the answer again, click "Refresh" ("Reload").
Do the problem yourself first!
a) arccos N = . N = cos .
b) arctan t = . t = tan .
c) arcsec u = a. u = sec a.
d) arccot 1 =

4
. 1 = cot

4
.
Example 1. Evaluate arcsin

2
-- "the angle whose sine is

2
."
Solution.

2
is the sine of what angle?

2
= sin

4
(Topic 4).
That is,
arcsin

2
=

4
.
The range of y = arcsin x
But

4
is not the only angle whose
sine is

2
.


2
is the sine of every 1st
and












2nd quadrant angle whose corresponding acute angle is

4

sin
3
4
=

2
.
sin (

4
+ 2 ) =

2
.
And so on.
For the function y = arcsin x to be single-valued, then, we
must restrict the values of y. How will we do that? We will
restrict them to those angles that have the smallest absolute
value.
In that same way, we will restrict the range of each inverse trigonometric
function. (Topic 3 of Precalculus.)

4
is the angle of
smallest absolute value
whose sine is

2
.
Example
2. Evaluate arcsin (

2
).
Solution. Angles whose sines are negative fall in the 3rd and
4th quadrants. (Topic 15.) The angle of smallest absolute value
is in the 4th
quadrant. It is the angle

4
.












arcsin (

2
) =

4
.
For angles whose sine is negative, we always choose a 4th quadrant angle. In
fact,
arcsin (x) = arcsin x.

arcsin() = arcsin
=

6
.

*
To see that arcsin(x) = arcsin x, look here:














= .


= .

That is,

arcsin(x) = arcsin x.
Here, then, is the range of the function y = arcsin x.















To restrict the range of arcsin x is
equivalent to restricting the
domain of sin x to those same
values. This will be the case with
all the restricted ranges that
follow.
Another notation for arcsin x is sin
1
x. Read: "The
inverse sine of x." 1 here is not an exponent (See Topic
19 of Precalculus.)
Problem 2. Evaluate the following in radians.
a) arcsin 0 = 0. (Topic 15.)
b) arcsin 1 = /2. (Topic 15.)
c) arcsin (1) = /2. (Topic 15.)

/3. (Topic 5.)

/3.

/6.













The range of y = arctan x
Similarly, we must restrict the range of y = arctan x. Like
y = arcsin x, y = arctan x has its smallest absolute values in
the 1st and 4th quadrants.


Note that y -- the angle whose
tangent is x -- must be greater
than

2
and less than

2
. For, at those quadrantal
angles, the tangent does not
exist.
(Topic 15.)
For angles whose tangent is postive, we choose a 1st quadrant angle. For
angles whose tangent is negative, we choose a 4th quadrant angle. Like
arcsin (x),
arctan (x) = arctan x.














= .


= .

Therefore,

arctan(x) = arctan x.
Problem 3. Evaluate the following.
a) arctan 1 =

4
b) arctan (1) =

4












c) tan
1
=

3
d) tan
1
( ) =

3
e) arctan 0 = 0 f)

=

6
The range of y = arccos x
The values of y = arccos x will have their smallest absolute
values when y falls within the 1st or 2nd quadrants.


Example 3. Evaluate
a) arccos
Solution. The radian angle whose cosine is is

3
(60).

b) arccos ()
Solution. An angle x whose cosine is negative falls in the
2nd quadrant.













And the cosine of a 2nd quadrant angle is the negative of the cosine of its
supplement. (Topic 16.) This implies:
An angle whose cosine is x is the supplement
of the angle whose cosine is x.
arccos (x) = arccos x.
Therefore,
arccos () = arccos
=

=
2
3
Problem 4. Evaluate the following.
a) arccos 1 = 0 b) arccos (1) =
c) cos
1


2
=

4
d) cos
1
(

2
) =
3
4
e) arccos 0 =

2
f)

=
5
6

The range of y = arcsec x












In calculus, sin
1
x, tan
1
x, and cos
1
x are the most important
inverse trigonometric functions. Nevertheless, here are the
ranges that make the rest single-valued.

Similarly for y = arccsc x.










TRIGONOMETRIC IDENTITIES
Reciprocal identities
Tangent and cotangent identities












Pythagorean identities
Sum and difference formulas
Double-angle formulas
Half-angle formulas
Products as sums
Sums as products

AN IDENTITY IS AN EQUALITY that is true for any value of the
variable. (An equation is an equality that is true only for certain
values of the variable.)
In algebra, for example, we have this identity:
(x + 5)(x 5) = x 25.
The significance of an identity is that, in calculation, we may
replace either member of the identity with the other. We use an
identity to give an expression a more convenient form. In calculus
and all its applications, the trigonometric identities are of central
importance.
On this page we will present the main identities. The student will have no better
way of practicing algebra than by proving them. Links to the proofs appear
below.
Reciprocal identities
sin =
1
csc
csc =
1
sin














cos =
1
sec
sec =
1
cos

tan =
1
cot
cot =
1
tan
Proof
Again, the point about an identity is that, in calculation, we
may replace either member of the identity with the other. Thus if
we see
"sin ", then we may, if we wish, replace it with "
1
csc
"; and,
symmetrically, if we see
"
1
csc
", then we may replace it with "sin
".
Tangent and cotangent identities
tan =
sin
cos
cot =
cos
sin
Proof

Pythagorean identities
a) sin + cos = 1














b) 1 + tan = sec


c) 1 + cot = csc

a') sin = 1 cos. cos = 1 sin.
These are called Pythagorean identities, because, as we will
see in their proof, they are the trigonometric version of the
Pythagorean theorem.
The two identities labeled a') -- "a-prime" -- are simply different versions of a).
The first shows how we can express sin in terms of cos ; the second shows
how we can express cos in terms of sin .
Note: sin -- "sine squared theta" -- means (sin ).
Example 1. Show:




Solution: The problem means that we are to write the left-hand
side, and then show, through substitutions and algebra, that we can
transform it to look like the right hand side. We begin:















Reciprocal identities



on adding the fractions



Pythagorean identities







Reciprocal identities
That is what we wanted to show.

Sum and difference formulas
sin ( + ) = sin cos + cos sin
sin ( ) = sin cos cos sin












cos ( + ) = cos cos sin sin
cos ( ) = cos cos + sin sin
Note: In the sine formulas, + or on the left is also + or
on the right. But in the cosine formulas, + on the left becomes
on the right; and vice-versa.
Since these identities are proved directly from geometry, the student is not
normally required to master the proof. However, all the identities that follow are
based on these sum and difference formulas. The student should definitely know
them.
To see the proof of the sum formulas, click here.
Example 2. Evaluate sin 15.
Solution.
sin
15





Formulas




Topics 4 and
5
















Example 3. Prove:





Solution.



Tangent
identity



Formulas

We will now construct tan by dividing the first term in the
numerator by cos cos . But then we must divide every term by
cos cos :








That is what we wanted to prove.












Double-angle formulas

Proof
There are three versions of cos 2 . The first is in terms of
both cos and sin . The second is in terms only of cos . The
third is in terms only of sin
Example 4. Show: sin
2



Solution. sin 2 = 2 sin cos Formulas

We will now construct tan by dividing by cos . But to
preserve the equality, we must also multiply by cos .




Lesson 5 of
Algebra




















Reciprocal
identities




Pythagorean
identities
That is what we wanted to prove.
Example 5. Show: sin x

Solution. sin x


-- according to the previous identity with =
x
2
.
Half-angle formulas
The following half-angle formulas are inversions of the double-
angle formulas, because is half of 2 .

The plus or minus sign will depend on the quadrant. Under the radical, the cosine
has the + sign; the sine, the sign.












Proof
Example 6. Evaluate cos

8
.
Solution.
Since

8
is half of


4
, then according to the half angle
formula:






Topic 4



Lesson 23 of Algebra







Lesson 27 of Algebra












Products as sums
a) sin cos

= [sin ( + ) + sin ( )]


b) cos sin

= [sin ( + ) sin ( )]


c) cos cos

= [cos ( + ) + cos ( )]


d) sin sin

= [cos ( + ) cos ( )]
Proof

Sums as products
e) sin A + sin B = 2 sin (A + B) cos (A B)

f) sin A sin B = 2 sin (A B) cos (A + B)

g) cos A + cos B = 2 cos (A + B) cos (A B)

h) cos A cos B = 2 sin (A + B) sin (A B)












In the proofs, the student will see that the identities e)
through h) are inversions of a) through d) respectively, which are
proved first. The identity f) is used to prove one of the main
theorems of calculus, namely the derivative of sin x.
The student should not attempt to memorize these identities. Practicing their
proofs -- and seeing that they come from the sum and difference formulas -- is
enough.













Proof of the product and sum formulas
Products as sums
a) sin cos

= [sin ( + ) + sin ( )]














b) cos sin

= [sin ( + ) sin ( )]


c) cos cos

= [cos ( + ) + cos ( )]


d) sin sin

= [cos ( + ) cos ( )]
Proof
These formulas are also derived from the sum and difference formulas.
To derive (a), write
sin ( + ) = sin cos + cos sin

sin ( )

= sin cos cos sin
and add vertically. The last terms in each line will cancel:
sin ( + ) + sin ( ) = 2 sin cos .
Therefore, on exchanging sides,
2 sin cos = sin ( + ) + sin ( ),
so that
sin cos = [sin ( + ) + sin ( )].
This is the identity (a)).
Formula (b) is derived in exactly the same manner, only instead of
adding, subtract sin ( ) from sin ( + ).
Formulas (c) and (d) are derived similarly. To derive (c), write












cos ( + ) = cos cos sin sin ,
cos ( ) = cos cos + sin sin ,
and add. To derive (d), subtract.
Let us derive (d). On subtracting, the first terms on the right will cancel. We will have
cos ( + ) cos ( ) = 2 sin sin .
Therefore, on solving for sin sin ,
sin sin = [cos ( + ) cos ( )].
Sums as products
e) sin A + sin B = 2 sin (A + B) cos (A B)

f) sin A sin B = 2 sin (A B) cos (A + B)

g) cos A + cos B = 2 cos (A + B) cos (A B)

h) cos A cos B = 2 sin (A + B) sin (A B)
Proof
The formulas (e), (f), (g), (h) are derived from (a), (b), (c), (d)
respectively; that is, (e) comes from (a), (f) comes from (b), and so on.
To derive (e), exchange sides in (a):
[sin ( + ) + sin ( )] = sin cos ,
so that












sin ( + ) + sin ( ) = 2 sin cos . . . . . . (1)
Now put
+ = A
and
= B. . . . . . . . . . . . . . . .(2)
The left-hand side of line (1) then becomes
sin A + sin B.
This is now the left-hand side of (e), which is what we are trying to
prove.
To complete the righthand side of line (1), solve those simultaneous equations (2) for and
.
On adding them, 2 = A + B,
so that
= (A + B).
On subtracting those two equations, 2 = A B,
so that
= (A B).
On the righthand side of line (1), substitute those expressions for
and . Line (1) then becomes
sin A + sin B = 2 sin (A + B) cos (A B).
This is the identity (e).
Read it as follows:
"sin A + sin B equals twice the sine of half their sum
times the cosine of half their difference."












Identities (f), (g), and (h) are derived in exactly the same manner from
(b), (c), and (d) respectively.

Proof of the sum formulas
Theorem. sin ( + ) = sin cos + cos sin

and

cos ( + ) = cos cos sin sin .
Proof. Let the straight line AB revolve to the point C and sweep out the

angle , and let it continue to D and sweep out the angle ;
draw DE perpendicular to AB.












Then we are to determine sin ( + ), which is
ED
DA
,
and cos ( + ), which is
AE
DA
.
Draw DF perpendicular to AC,
draw FG perpendicular to AB,
and draw FH perpendicular to ED.
Then angle HDF is equal to angle .
For, since the straight line AC crosses the parallel lines HF, AB, it makes the
alternate angles equal (Theorem 8);
therefore angle HFA is equal to angle .
And by the construction, angle DFH is the complement of angle HFA;
therefore angle HDF (the complement of DFH) is also equal to angle .

Now,
ED = GF + HD.













Therefore, on dividing by DA,

sin ( + ) =
ED
DA
=
GF
DA
+
HD
DA
=
GF
AF
AF
DA
+
HD
FD
FD
DA

= sin cos + cos sin .

Next,
EA = GA FH.

Therefore,













cos ( + ) =
EA
AD
=
GA
AD

FH
AD
=
GA
AF
AF
AD

FH
DF
DF
AD

= cos cos sin sin .
This is what we wanted to prove.
The difference formulas can be proved from the sum formulas, by
replacing + with +(), and using these identities:
cos () = cos
sin () = sin .
Topic 16


Proof of the double-angle and half-angle formulas
Double-angle formulas













Proof
The double-angle formulas are proved from the sum formulas by
putting = . We have
sin 2 = sin ( + ) = sin cos + cos sin

= 2 sin cos .

cos 2 = cos ( + ) = cos cos sin sin

cos 2 = cos sin . . . . . . . (1)
This is the first of the three versions of cos 2 . To derive the second
version, in line (1) use this Pythagorean identity:
sin = 1 cos .
Line (1) then becomes
cos 2 = cos (1 cos )

= cos 1 + cos .













cos 2 = 2 cos 1. . . . . . . . . . (2)
To derive the third version, in line (1) use this Pythagorean identity:
cos = 1 sin .
We have
cos 2 = 1 sin sin ;.

cos 2 = 1 2 sin . . . . . . . . . . (3)
These are the three forms of cos 2 .
Halfangle formulas
. . . . . . . (2')
. . . . . . . (3')
Whether we call the variable or does not matter. What matters is the
form.
Proof
Angle is half of the angle 2 . Therefore, in line (2), if we
put 2 = , then will equal

2
:
cos = 2 cos

, we will have the half-angle formula for the












2
cosine.
So, on exchanging sides and transposing 1, we have
2 cos

2
= (1 + cos )

cos

2
comes from putting 2 = in line (3). On

transposing, line (3) becomes
2 sin

2
=
.
This is the halfangle formula for the sine.







note :should be first












RATIO AND PROPORTION
The natural numbers: Cardinal and ordinal
Parts of natural numbers
Parts, plural
The ratio of natural numbers
The names of the fractions
Proportions
The theorem of the alternate proportion
The theorem of the same multiple
Similar figures

TRIGONOMETRY historically is the study of triangles. The name literally
means measurement of triangles. It begins with the study of right
triangles, which are triangles that have a right angle, and with the
ratios-- the relationships -- of their sides. Now the meaning of a ratio
depends on what we mean by the parts of a number. And we name
parts with ordinal numbers: the third part, the fourth, the fifth, and so
on. So with those we shall begin.
Natural numbers: cardinal and ordinal
The natural numbers are the counting numbers. They have two forms,
cardinal and ordinal. The cardinal forms are
One, two, three, four, and so on.
They answer the question How much? or How many?. The ordinal forms
are
First, second, third, fourth, and so on.
They answer the question Which one?.












Parts of natural numbers
We say that a smaller number is a part of a larger number if the larger
number is its multiple.
Here are the first few multiples of 5:
5, 10, 15, 20, 25, 30, 35.
5 is the first multiple of 5. 10 is the second multiple; 15, the third; and so
on.
5 is a part of each one of those, except itself.
Now, since 15, for example, is the third multiple of 5, we say that 5 is the third part of 15. We
use that same ordinal number to name the part.
5 is the fourth part of 20. It is the fifth part of 25; the sixth part of 30. And so on.
5 is which part of 10? We do not say the second part. We say half. 5 is half of 10.
So with the exception of half, we name the parts with ordinal numbers. Each ordinal number
names which part.
Again, 5 is not a part of itself. There is no such thing as the first part.
(See Skill in Arithmetic, Lesson 15.)
It is important to understand that we are not speaking here of proper
fractions -- numbers that are less than 1, and that we need for measuring. We
are explaining how the ordinal numbers -- third, fourth, fifth, and so on --
name the parts of the cardinal numbers. We will come to those fractional
symbols shortly.
Problem 1. 7 is which part of 28? The fourth.
To see the answer, pass your mouse over the colored area.
To cover the answer again, click "Refresh" ("Reload").
Problem 2. Which part of 45 is 9? The fifth.
Problem 3. 6 is which part of 12? Half.












Problem 4. What number is the eighth part of 24? 3
Problem 5. 3 is the tenth part of what number? 30
Parts, plural

The figure shows that we have divided 15 into thirds, that is, into three equal
parts, and that each 5 is a third part of 15.
5, 10, 15.
Therefore, 10 -- which is two 5's -- is two third parts of 15. One third.
Two thirds. 5, 10.
Now, 10 is not a part of 15, because 15 is not a multiple of 10. We say that it is parts of 15,
plural: Two third parts, or simply two thirds. Those words are to be taken literally.
Similarly, if we divide 15 into fifths, that is, into five equal parts --

-- then
3 is the fifth part of 15.
6 is two fifth parts of 15.
9 is three fifth parts of 15. (Count them!)












12 is four fifth parts of 15, or simply four fifths.
And 15 is all five of its fifth parts.
We can now say what we mean by

The ratio of natural numbers
Definition. The ratio of two natural numbers is their relationship with
respect to relative size that we can express verbally in a sentence. Specifically,
it is their relationship in which one number is a multiple of the other (so many
times it), a part of it, or parts of it.
(Skill in Arithmetic, Lesson 17.)
Example 1. Multiple What ratio has 15 to 5?
Answer. 15 is three times 5.
That is the ratio -- the relationship -- of 15 to 5.
We do not answer "3 to 1," because we want to name the ratio of 15 to 5 explicitly. It
is true that 15 is to 5 in the same ratio as 3 is to 1. 3 is three times 1, just as 15 is three
times 5.
The two numbers in a ratio are called the terms; the first term
and the second.
Notice that we answer with a complete sentence beginning with the first term: "15 is
three times 5." For, a ratio is a relationship.
Example 2. Part What ratio has 5 to 15?
Answer. 5 is the third part of 15.
That is the inverse ratio of 15 to 5. The terms are exchanged.
Example 3. Parts What ratio has 10 to 15?












Answer. 10 is two thirds of 15.

Those are the three kinds of ratio: One number is a multiple of
the other -- so many times it -- a part of it, or parts of it.
Problem 6. What ratio have the following? Answer with a complete
sentence beginning with the first term.
a) 2 to 10? 2 is the fifth part of 10.
b) 10 to 2?

10 is five times 2. A larger number will be so many
times a smaller.
c) 7 to 1? 7 is seven times 1.
d) 1 to 7? 1 is the seventh part of 7.
e) 25 to 100? 25 is the fourth part of 100.
f) 75 to 100? 75 is three fourths of 100.
g) 12 to 6? 12 is two times 6, or twice as much as 6, or double 6.
h) 6 to 12? 6 is half of 12.
The names of the fractions
In English, the proper fractions have the same names as the ratio of
the
numerator to the denominator. The number we write as 1 over 3 --

1
3
--

is called "one-third" because of the ratio of 1 to 3. 1 is one third of 3.













And the fraction
1
3
has that same ratio to 1.

For that reason, fractions are called rational numbers. Fractions
have the same ratio to 1 as two natural numbers.
Fractional symbols may therefore be regarded as ratio symbols, in that they signify the
ratio of the numerator to the denominator.
1
2
=
2
4
=
5
10
=
9
18
, etc.
1 is to 2 as 2 is to 4 as 5 is to 10, etc.
Each numerator is half of its denominator.
Example 4. What ratio has 3 to 4?
Answer. We can express the ratio of any smaller number to a larger
simply by letting each number say its name. 3 is three fourths of 4. 3
says its cardinal name "three." 4 says its ordinal name "fourths."
On the practical side, since the student has very likely learned the
names of the fractions first, then the ratio of 3 to 4 is expressed by the
name of the fraction
3
4

Proportions
A proportion is a statement that two ratios are the same.
5 is to 15 as 8 is to 24.
5 is the third part of 15, just as 8 is the third part of 24.












We will now introduce this symbol 5 : 15 to signify the ratio of
5 to 15. A proportion will then appear as follows:
5 : 15 = 8 : 24.
"5 is to 15 as 8 is to 24."
Or, we can represent a proportion with fractional symbols --
5
15
=
8
24
-- and read it the same way:
"5 is to 15 as 8 is to 24."
Example 5. 12 : 2 = 42 : 7. ("12 is to 2 as 42 is to 7.")
Why is this a proportion?
Answer. Because 12 is six times 2, just as 42 is six times 7.
Problem 7. Complete this proportion, 3 : 12 = 7 : ?
28. 3 is the fourth part of 12, and 7 is the fourth part of 28.

The theorem of the alternate proportion
The numbers in a proportion are called the terms: the 1st, the 2nd,
the 3rd, and the 4th.
1st : 2nd = 3rd : 4th
We say that the 1st and the 3rd are corresponding terms, as are
the 2nd and the 4th.
The following is the theorem of the alternate proportion:
If four numbers are proportional, then the
corresponding terms are also proportional.












That is, as the first term is to the third, so the second will be to the fourth.

If
a : b = m : n,

then, alternately,

a : m = b : n.
Since
1 : 3 = 5 : 15,
then alternately,
1 : 5 = 3 : 15.
(Skill in Arithmetic: Lesson 17, Question 2.)
Example 6. Complete this proportion:
4 : 5 = 12 : ?
Now, 4 is four fifths of 5 ("
4
5
") , but it is not obvious what number 12
is four fifths of.
Alternately, however, 4 is a third of 12 -- or we could say that 4 has been multiplied by
3. Therefore 5 also must be multiplied by 3,
4 : 5 = 12 : 15
That is,
4 : 5 = 3 4 : 3 5













As one 4 is to one 5, so any number of 4's will be to an equal number of 5's. Three 4's
are four fifths of three 5's.

This is the theorem of the same multiple.
If we multiply two numbers by the same number,
then the products will have the same ratio
as the numbers we multiplied.
Example 7. Complete this proportion, 5 : 8 = 35 : ?
Answer. Look at it alternately. 35 is seven times 5. Therefore the
missing term will be seven times 8, which is 56.
Problem 8. Complete this proportion, 4 : 9 = 24 : ?
54. The term corresponding to 4 is 6 4.
Therefore, the missing term must be 6 9.
We shall often make use of this basic property of the square
root radical:

A radical multiplied by itself
produces the radicand.
(See Lesson 26 of Algebra, Radicals.)














Problem 9. Complete this proportion: : 2 = 3 : ?
2 . For, has been multiplied by ; therefore 2
also must be multiplied by .

Example 9. Solve this proportion -- 8 : 12 = 2 : ?
Answer. To produce 2, 8 has been divided by 4. Therefore 12
also must be divided by 4.
8 : 12 = 2 : 3
The theorem of the same multiple may be inverted to mean
that we may divide both terms by the same number.
An all too common method these
days is to make this an algebra
problem.
8
12
=
2
x
The student is taught to cross-
multiply and solve for x. That is
a method for people who do not
understand ratio and proportion.












It is taught in order not to teach
ratio and proportion verbally. It
stems from a 19th century
resistance to language in
mathematics.

Example 10. Solve this proportion -- 2 : 11 = 9 : ?
Solution. How was 2 multiplied to produce 9? It was
multiplied by
9
2
:
9
2
2 = 9.
Therefore 11 will also be multiplied by
9
2
:
9
2
11 =
99
2
= 49.


Similar figures
Trigonometry depends on the meaning of similar figures.
Similar figures are equiangular, and the sides
that make the equal angles are proportional.













To say then that figures ABCDE, PQRST are similar, is to say that
the angle at A is equal to the angle at P, the angle at B is equal to the
angle at Q, etc.; and, proportionally, as AB is to BC, so PQ is to QR.
And so on, for each pair of equal angles and the sides that make them.
For triangles to be similar, however, it is sufficient
that they be equiangular. (Theorem 15 of "Some
Theorems of Plane Geometry.") From that it follows:
Right triangles will be similar if an
acute angle of one
is equal to an acute angle of the other.

In the right triangles ABC, DEF, if the acute angle at B is equal to
the acute angle at E, then those triangles will be similar. Therefore
the sides that make the equal angles will be proportional. For
example,
BC : CA = EF : FD.
If BC were twice as long as CA, then EF would
also be twice as long as FD.












And alternately, the corresponding sides are proportional:
BC : EF = CA : FD.
If BC were half of EF, then CA would also be
half of FD.

Problem 10. The right triangles HJK, LMN are
similar, with HJ = 3 cm,

JK = 4 cm, and KH = 5 cm. If LM = 12 cm, then how
long are MN and NL?
Alternately,
LM : HJ = MN : JK.
Since LM is four times HJ, then MN will be
four times JK -- it will be 16 cm.
Similarly, NL will be four times KH, it will be 20 cm.
To put the whole thing more simply: The side
corresponding to HJ has been multiplied by 4.
Therefore, since the triangles are similar, every side will
be multiplied by 4.
Problem 11. Triangles ABC, DEF are similar.













AB = 6 cm, BC = 11 cm, CA = 7 in, and DE = 18 cm. How long
are EF and DF?
The side corresponding to AB has been multiplied
by 3. Therefore every side will be multiplied by 3.
EF = 33 cm, and DF = 21 cm.
*
Our definition of a proportion has been in terms of the
ratio of natural numbers. The sides of a triangle, however,
are not natural numbers. They are lengths, which are
magnitudes -- things that have size -- they are not
numbers. And magnitudes do not necessarily have the
same ratio as natural numbers. They can be
"incommensurable." Hence a new theory of ratio and
proportion is required which will be applicable to
magnitudes. The interested student is referred to Topic 11
of The Evolution of the Real Numbers, in particular the
section "The new theory of proportions."

Definitions of the Trigonometric Functions
of an Acute Angle

BEFORE DEFINING THE TRIGONOMETRIC FUNCTIONS, we must see how to
relate the angles and sides of a right triangle.












A right triangle is composed of a
right angle, the angle at C, and two
acute angles, which are angles less
than a right angle. It is conventional
to label the acute angles with Greek
letters. We will label the angle at B
with the letter ("THAY-ta"). And
we will label the angle at A with the
letter ("fie").
As for the sides, the side AB, opposite the
right angle, is called the hypotenuse ("hy-
POT'n-yoos"). Each acute angle is formed by the hypotenuse and the side adjacent to the
angle. Thus, angle is formed by the hypotenuse and side BC. Angle is formed by the
hypotenuse and side AC.
With respect to angle , though, side AC is its opposite side. While side BC is the side
opposite .
The ratios of sides
Any two sides of the triangle will have a ratio -- a relationship -- to one
another. It is possible to form six such ratios: the ratio of the opposite side
to the hypotenuse; the adjacent side to the hypotenuse; and so on. Those six
ratios have historical names and abbreviations, with which the student will
have to make peace. They are the following.














sine of = sin =
opposite
hypotenuse
cosecant of = csc =
hypotenuse
opposite

cosine of = cos =
adjacent
hypotenuse
secant of = sec =
hypotenuse
adjacent

tangent of = tan =
opposite
adjacent
cotangent of = cot =
adjacent
opposite


Notice that each ratio in the right-hand column is the inverse, or the reciprocal, of the ratio in
the left-hand column.
The reciprocal of sin is csc ; and vice-versa.
The reciprocal of cos is sec .
And the reciprocal of tan is cot .
Each ratio moreover is a function of the acute angle. That is, one
quantity is a "function" of another if its value depends on the value of the
other. The circumference of a circle is a function of the radius, because the












size of the circumference depends on the size of the radius, and when the
radius changes, the circumference also will change. As we will see in the
next Topic, the value of each ratio depends only on the value of the acute
angle. That is why we say that those ratios are functions of the acute angle.
We call them the trigonometric functions of an acute angle. All of
trigonometry is based on the definitions of those functions.
Problem 1. Complete the following with either "opposite," "adjacent to,"
or "hypotenuse."
To see the answer, pass your mouse over the colored area.
To cover the answer again, click "Refresh" ("Reload").

a) In a right triangle, the side opposite the right angle is called the
a) hypotenuse.
b) CA is called the side opposite angle .
c) BC is called the side adjacent to angle .
d) AC is called the side adjacent to angle .
e) BC is called the side opposite angle .












Problem 2. The sides of a right triangle are in the ratio 3 : 4 : 5, as shown.
Name and evaluate the six trigonometric functions of angle .

sin =
4
5
csc =
5
4
cos =
3
5
sec =
5
3
tan =
4
3
cot =
3
4

Problem 3. The sides of a right triangle are in the ratio 8 : 15 : 17, as
shown. Name and evaluate the six trigonometric functions of angle .

sin =
15
17
csc =
17
15
cos =
8
17
sec =
17
8












tan =
15
8
cot =
8
15

Notice that the sides of this triangle satisfy, as they must, the
Pythagorean theorem:
8 + 15 = 17

64 + 225 = 289
Problem 4. A straight line makes an angle with the x-axis. The value

of which function of is equal to its slope?














Problem 5. The height of a triangle. Every triangle, right-angled or
not, will have at least two acute angles.

Let them be the base angles at A and B, so that the base will be the
side c. Show that the height h drawn to that base is
h =
_____c_____
cot A + cot B
.
Hint: The height h will cut the entire triangle into two right
triangles.

Let x be the segment of the base containing the angle A. Then the
remaining segment is the difference between the whole c and that
segment: c x.
In the right triangle containing the acute angle A,
x
h
= cot A, or x = h cot A . . . (1)
In the right triangle containing the acute angle B,












c x
h
= cot B, or c x = h cot B.
On substituting the expression for x from line (1),
c h cot A = h cot B,
which implies
c = h cot A + h cot B = h(cot A + cot B).
Therefore, on solving for h,
h =
_____c_____
cot A + cot B
.

Problem 6. The area of a triangle. Prove: The area of a triangle is equal
to one-half the sine of any angle times the product of the two sides that make the
angle.

Specifically, prove that
Area of triangle ABC = sin A bc = bc sin A.













The area of a triangle is equal to one-half the base times the
height. In triangle ABC, let the base be c. Then
Area = ch.
Now,
sin A = h/b,
so that
h = b sin A.
Therefore in the expression for the Area, replace h with b sin A:
Area = cb sin A.

TRIGONOMETRY OF RIGHT TRIANGLES
Similar figures
All functions from one function
Complements
Cofunctions

PLANE TRIGONOMETRY is based on the fact of similar figures.
(Topic 1.) We saw:
Figures are similar if they are equiangular
and the sides that make the equal angles
are proportional.
For triangles to be similar, however, it is sufficient that they be
equiangular. (Theorem 15 of "Some Theorems of Plane Geometry.")
From that it follows:












Right triangles will be similar if an acute angle of
one
is equal to an acute angle of the other.

In the right triangles ABC, DEF, if the acute angle at B is
equal to the acute angle at E, then those triangles will be similar.
Therefore the sides that make the equal angles will be
proportional. Whatever ratio CA has to AB, FD will have to DE.
If CA were half of AB, for example, then FD would also be half
of DE.
A trigonometric Table is a table of ratios of sides. In the
Table, each value of sin represents the ratio of the opposite side
to the hypotenuse -- in every right triangle with that acute angle.
If angle is 28, say, then in every right triangle with a 28 angle, its sides will
be in the same ratio. We read in the Table,
sin 28 = .469
This means that in a right triangle having an acute angle of 28, its opposite side
is 469 thousandths of the hypotenuse, which is to say, a little less than half.
It is in this sense that in a right triangle, the trigonometric ratios -- the sine, the
cosine, and so on -- are "functions" of the acute angle. They depend only on the
acute angle.

Example. Indirect measurement. Trigonometry is used
typically to measure things that we cannot measure directly.













For example, to measure the height h of a flagpole, we could measure a distance
of, say, 100 feet from its base. From that point P, we could then measure the
angle required to sight the top . If that angle, called the angle of elevation,
turned out to be 37, then
so that
h
100
= tan 37
so that
so that h = 100 tan 37.
From the Table, we find
tan 37 = .754
Therefore, on multiplying by 100,
h = 75.4 feet.
(Skill in Arithmetic: Multiplying and dividing by powers of 10.)
All functions from one function
If we know the value of any one trigonometric function, then --
with the aid of the Pythagorean theorem -- we can find the rest.
Example 1. In a right triangle, sin =
5
. Sketch the triangle, place












13
the ratio numbers, and evaluate the remaining functions of .

To find the unknown side x, we have
x + 5 = 13
x = 169 25 = 144.
Therefore,
x = = 12.
(Lesson 26 of Algebra.)
We can now evaluate all six functions of :
sin =
5
13
csc =
13
5
cos =
12
13
sec =
13
12
tan =
5
12
cot =
12
5
Example 2. In a right triangle, sec = 4. Sketch the triangle,
place the ratio numbers, and evaluate the remaining
functions of .
To say that sec = 4, is to say that the hypotenuse is
to the adjacent side in the ratio 4 : 1. (4 =
4
1
)
To find the unknown side x, we have












x + 1 = 4

x = 16 1 = 15.
Therefore,
x = .
We can now evaluate all six functions of :
sin =

4
csc =
4

cos =
1
4
sec = 4
tan =

cot =
1

Problem 1. In a right triangle, cos =
2
5
. Sketch the triangle and
evaluate sin .
To see the answer, pass your mouse over the colored area.
To cover the answer again, click "Refresh" ("Reload").













Problem 2. cot = . Sketch the triangle and evaluate csc .


Complements
Two angles are called complements of one another if together they
equal a right angle. Thus the complement of 60 is 30. This is
the degree system of measurement in which a full circle, made up
of four right angles at the center, is called 360. (But see Topic 12:
Radian Measure.)












Problem 3. Name the complement of each angle.
a) 70 20 b) 20 70 c) 45 45 d) 90



The point about complements is that, in a right
triangle, the two acute angles are
complementary. For, the three angles of the
right triangle are together equal to two right
angles (Theorem 9); therefore, the two acute
angles together will equal one right angle.

(When we come to radian measure, we will see that 90 =

2
, and
therefore the complement of is

2
.)

Cofunctions
There are three pairs of cofunctions:
The sine and the cosine
The secant and the cosecant
The tangent and the cotangent
And here is the significance of a cofunction:
A function of any angle is equal to the cofunction
of its complement.
This means, for example, that












sin 80 = cos 10.
The cofunction of the sine is the cosine. And 10 is the
complement of 80.
Problem 4. Answer in terms of cofunctions.
a) cos 5 = sin 85 b) tan 60 = cot 30 c) csc 12 =
sec 78
d) sin (90 ) = cos e) cot = tan (90 )


In the figure:
sin =
a
c
cos =
a
c
Thus the sine of is equal to the cosine of its complement.
sec =
c
b
csc =
c
b
The secant of is equal to the cosecant of its complement.
tan =
a
b
cot =
a
b
The tangent of is equal to the cotangent of its complement.













THE ISOSCELES RIGHT TRIANGLE

AN ISOSCELES RIGHT TRIANGLE is a standard mathematical object. The
student should know the ratios of the sides.
(An isosceles triangle has two equal sides. See Definition 8 in Some Theorems of Plane
Geometry. The theorems cited below will be found there.)
Theorem. In an isosceles right triangle the sides are in the ratio 1:1: .

Proof. In an isosceles right triangle, the equal sides make the right angle.
They are in the ratio 1 : 1.
To find the ratio number of the hypotenuse h, we have, according to the Pythagorean theorem,
h = 1 + 1 = 2.
Therefore,
h = .
(Lesson 26 of Algebra.) And therefore the three sides are in the ratio
1 : 1 : .
Note that since the right triangle is isosceles, then the angles at the base
are equal. (Theorem 3.) Therefore each of those acute angles is 45.
(For the definition of measuring angles by "degrees," see Topic 12.)













Example 1. Evaluate sin 45 and tan 45.
Answer. For any problem involving 45, the student should not consult the
Table. Rather, sketch the triangle and place the ratio numbers.

We see:
sin 45 =
1

= ,
on rationalizing the denominator. (Lesson 26 of Algebra.)
tan 45 =
1
1
= 1.
Problem. Evaluate cos 45 and csc 45.

cos 45 =
1

= .
Thus cos 45 is equal to sin 45; they are complements.
csc 45 =

1
= .
Example 2. Solve the isosceles right triangle whose side is 6.5 cm.












Answer. To solve a triangle means to know all three sides and all three
angles. Since this is an isosceles right triangle, the only problem is to find
the unknown hypotenuse.

But in every isosceles right triangle, the sides are in the ratio 1 : 1 : , as shown on the right.
In the triangle on the left, the side corresponding to 1 has been multiplied by 6.5. Therefore
every side will be multiplied by 6.5. The hypotenuse will be 6.5 . (The theorem of the
same multiple.)
Whenever we know the ratio numbers, we use this method of similar
figures to solve the triangle, and not the trigonometric Table.
(In Topic 6, we will solve right triangles the ratios of whose sides we do not know.)
Example 3. In an isosceles right triangle, the hypotenuse is inches.
How long are the sides?
Answer. The student should sketch the triangles and place the ratio
numbers.

How has the side corresponding to been multiplied?












According to the rule for multiplying radicals, it has been multiplied by . Therefore, all
the sides will be multiplied by . And 1 = .

Next Topic: The 30-60-90 Triangle
THE 30-60-90 TRIANGLE
A 30-60-90 TRIANGLE is another standard mathematical object. The
student should know the ratios of the sides.
Theorem. In a 30-60-90 triangle the sides are in the ratio 1 : 2 : .

We will prove that below.
(For the definition of measuring angles by "degrees," see Topic 12.)
Note that the smallest side, 1, is opposite the smallest angle, 30; while
the largest side, 2, is opposite the largest angle, 90. (Theorem 6). (For, 2 is
larger than . Also, while 1 : : 2 correctly corresponds to the sides
opposite 30-60-90, many find the sequence 1 : 2 : easier to remember.)
The cited theorems are from the Appendix, Some theorems of plane geometry.
Here are examples of how we take advantage of knowing those ratios.
First, we can evaluate the functions of 60 and 30.
Example 1. Evaluate cos 60.












Answer. For any problem involving a 30-60-90 triangle, the student
should not use a table. The student should sketch the triangle and place the
ratio numbers.

Since the cosine is the ratio of the adjacent side to the hypotenuse, you
can see that cos 60 = .

Example 2. Evaluate sin 30.
Answer. According to the property of cofunctions (Topic 3), sin 30 is
equal to cos 60. sin 30 = .
You can see that directly in the figure above.

Problem 1. Evaluate sin 60 and tan 60.
To see the answer, pass your mouse over the colored area.
To cover the answer again, click "Refresh" ("Reload").

The sine is the ratio of the opposite side to the hypotenuse.
sin 60 =

= .












2
Lesson 5 of Algebra.
The tangent is ratio of the opposite side to the adjacent.
tan 60 =

1
= .

Problem 2. Evaluate cot 30 and cos 30.

The cotangent is the ratio of the adjacent side to the opposite.
Therefore, on inspecting the figure above, cot 30 =

1
= .
Or, more simply, cot 30 = tan 60. Problem 1.
As for the cosine, it is the ratio of the adjacent side to the
hypotenuse. Therefore,
cos 30 =

2
= .
Before we come to the next Example, here is how we relate the sides
and angles of a triangle:













If an angle is labeled capital A, then the side opposite will be labeled
small a. Similarly for angle B and side b, angle C and side c.
Example 3. Solve the right triangle ABC if angle A is 60, and side c is 10
cm.

Solution. To solve a triangle means to know all three sides and all three
angles. Since this is a right triangle, and angle A is 60, then the remaining
angle B is its complement, 30.
Now in every 30-60-90 triangle, the sides are in the ratio 1 : 2 : , as shown on the
right. Whenever we know the ratios of the sides, we can solve the triangle by the method of
similar figures.
And so in triangle ABC, the side corresponding to 2 has been multiplied by 5. Therefore
every side will be multiplied by 5. Side b will be 5 1, or simply 5 cm, and side a will be 5
cm.
Alternatively, we could say that the side adjacent to 60 is always half of
the hypotenuse. Therefore, side b will be 5 cm. Now, side b is the side that
corresponds to 1. And it has been multiplied by 5. Therefore, side a must
also be multiplied by 5. It will be 5 cm.












Whenever we know the ratio numbers, the student should use this
method of similar figures to solve the triangle, and not the trigonometric
Table.
(In Topic 6, we will solve right triangles the ratios of whose sides we do not know.)
Problem 3. In the right triangle DFE, angle D is 30, and side DF is
3 inches. How long are sides d and f ?


The student should draw a similar triangle in the same
orientation. Then see that the side corresponding to was multiplied
by . Therefore, each side will be multiplied by . Side d will be 1
= . Side f will be 2 .
Problem 4. In the right triangle PQR, angle P is 30, and side r is 1 cm.
How long are sides p and q ?














The side corresponding to 2 has been divided by 2. Therefore,
each side must be divided by 2. Side p will be , and side q will be
.
Problem 5. Solve the right triangle ABC if angle A is 60, and the
hypotenuse is 18.6 cm.


The side adjacent to 60 is always half of the hypotenuse --
therefore, side b is 9.3 cm.
But this is the side that corresponds to 1. And it has been multiplied
by 9.3. Therefore, side a will be multiplied by 9.3.
It will be 9.3 cm.
Example 4. ABC is an equilateral triangle whose height AD is 4 cm.













Find the length of the side x.
Solution 1. Since the triangle is equilateral, it is also equiangular, and
therefore the the angle at B is 60
The height of a triangle is the straight line drawn from the vertex at right angles to the base.
Therefore, triangle ADB is a 30-60-90 triangle.

For this problem, it will be convenient to form the proportion with
fractional symbols:
x
4
=
2

That implies
x =
4 2

=
8

.
On taking to be approximately 1.732,












_8_
1.732
4.619 cm.
Solution 2.

The side corresponding to was multiplied to become 4. How was it
multiplied?
By
4

:
4

= 4.
Compare the previous topic.
Therefore the hypotenuse 2 will also be multiplied by
4

:
4

2 =
8

,
which, again, is approximately 4.619 cm.
Problem 6. Inspect the values of 30, 60, and 45 -- that is, look at the
two triangles --













-- and decide which of those angles is the value of x in each equation.
a) sin x = cos x. x = 45. b) tan x = 1. x = 45.

c) sin x = . x = 30 d) cos x = . x = 60.


e) sin x = . x = 60
f)
cos x = . x = 45.

x = 30. h) csc x = 2. x = 30.
Example 5. Solve this equation for angle x:
2 sin (x 10) = 0.

Solution. 2 sin (x 10) = 0













implies
sin (x 10) = .
Now, the sine of what angle is ?
sin 60 = .

Therefore,
x 10 = 60.

x = 70.
Problem 7. Solve this equation for angle x:
3 csc (2x + 6) 6 = 0.

3 csc (2x + 6) 6 = 0

implies












csc (2x + 6) = 2.

2x + 6 = 30

x + 3 = 15

x = 12.
Problem 8. Prove: The area A of an equilateral triangle whose side is s, is
A = s.

The area A of any triangle is equal to one-half the sine of any
angle times the product of the two sides that make the angle. (Topic
2, Problem 6.)
In an equilateral triangle each side is s , and each angle is 60.
Therefore,
A = sin 60s.
Since sin 60 = (Problem 1),
A = s = s.













Problem 9. Prove: The area A of an equilateral triangle inscribed in a circle of
radius r, is
A =
3
4

r.


The three radii divide the triangle into three congruent triangles
(Side-side-side); hence each radius bisects each vertex into two 30
angles.
If we extend the radius AO, then AD is the perpendicular bisector of
the side CB. (Theorem 2.) Triangle OBD is therefore a 30-60-90
triangle.
If we call each side of the equilateral triangle s, then in the right
triangle OBD,
s
r
= cos 30 = .
Therefore,












s = r
so that
s = 3r.
Now, according to the previous problem, the area A of an
equilateral triangle is
A = s.
Therefore,
A = s = 3r =
3
4
r.
That is what we wanted to prove.
Problem 10. Prove: The angle bisectors of an equilateral triangle meet at a point
that is two thirds of the distance from the vertex of the triangle to the base.

Let ABC be an equilateral triangle, let AD, BF, CE be the angle
bisectors of angles A, B, C respectively; then those angle bisectors meet at
the point P such that AP is two thirds of AD.
First, triangles BPD, APE are congruent.
For, since the triangle is equilateral and BF, AD are the
angle bisectors, then angles PBD, PAE are equal and each
30;
and the side BD is equal to the side AE, because in an












equilateral triangle the angle bisector is the perpendicular
bisector of the base (Theorem 2);
angles PDB, AEP then are right angles and equal.
Therefore triangles BPD, APE are congruent: Angle-side-
angle.

Now,
BP
PD
= csc 30 = 2.
There
fore,
BP =
2PD.
But AP = BP, because triangles APE, BPD are
conguent, and those are the sides opposite the
equal angles.
Therefore, AP = 2PD.
Therefore AP is two thirds of the whole AD.
Which is what we wanted to prove.
The proof












Here is the proof that in a 30-60-90 triangle the sides are in the
ratio 1 : 2 : . It is based on the fact that a 30-60-90 triangle is
half of an equilateral triangle.
Draw the equilateral triangle ABC. Then each of its equal
angles is 60. (Theorems 3 and 9)
Draw the straight line AD bisecting the angle at A into two
30 angles.
Then AD is the perpendicular bisector of BC (Theorem 2).
Triangle ABD therefore is a 30-60-90 triangle.
Now, since BD is equal to DC, then BD is half of BC.
This implies that BD is also half of AB, because AB is equal
to BC. That is,
BD : AB = 1 : 2
From the Pythagorean theorem, we can find
the third side AD:
AD + 1 = 2
AD = 4 1 = 3
AD = .
Therefore in a 30-60-90 triangle the sides are in the ratio 1 : 2 : ; which is
what we set out to prove.

SOLVING RIGHT TRIANGLES













This is a topic in traditional trigonometry. It does not come up in calculus.
To SOLVE A TRIANGLE means to know all three sides and all three angles.
When we know the ratios of the sides, we use the method of similar
figures. That is the method to use when solving an isosceles right
triangle or a 30-60-90 triangle. When we do not know the ratio
numbers, then we must use the Table of ratios. The following
example illustrates the method.
The general method
Example 1. Given an acute angle and one side. Solve the right triangle
ABC if angle A is 36, and side c is 10 cm.
Solution. Since angle A is 36, then angle B is 90
36 = 54.
To find an unknown side, say a, proceed as follows:

1.

Make the unknown side the numerator of a
fraction, and make the known side the
denominator.
Unknown
Known
=
a
10
2. Name that function of the angle.
Unknown
Known
=
a
10
= sin 36












3. Use the trigonometric Table to evaluate that function.
Unknown
Known
=
a
10
= sin 36 = .588
4. Solve for the unknown side.
a = 10 .588 cm = 5.88 cm
Problem 1. Solve the triangle for side b.

To see the answer, pass your mouse over the colored area.
To cover the answer again, click "Refresh" ("Reload").
To consult the Table, click here.
Unknown
Known
=
b
10
= cos 36 = .809


b = 10 .809 = 8.09 cm
Problem 2. To measure the width of a river. Two trees stand opposite
one another, at points A and B, on opposite banks of a river.













Distance AC along one bank is perpendicular to BA, and is measured to be
100 feet. Angle ACB is measured to be 79. How far apart are the trees;
that is, what is the width w of the river? (Table)
Unknown
Known
=
w
100
= tan 79 = 5.145,
from the Table. Therefore,
w = 100 5.145 = 514.5 ft

(To measure the height of a flagpole, and for the meaning of the angle of elevation, see the
Example in Topic 3.)
Example 2. Find the distance of a boat from a lighthouse if the lighthouse
is 100 meters tall, and the angle of depression is 6.

Solution. The angle of depression is the angle below straight ahead --
horizontal -- that an oberver must look in order to see something below the
observer. Thus in order to see the boat, the lighthouse keeper must look
down 6.












Now, the triangle formed by the lighthouse and the distance d of the boat from the lighthouse,
is right-angled. And since the angle of depression is 6, then the alternate angle is also 6.
(Euclid, I. 29.)
If d is the distance of a boat from the lighthouse, then
d
100
= cot 6 = 9.514, from the Table.
Therefore,
d = 951.4 meters.
Example 3. Given two sides of a right triangle. Solve the right
triangle ABC given that side c = 25 cm and side b = 24 cm.
Solution. To find the remaining side a, use the
Pythagorean theorem:
a + 24 = 25
a = 625 576 = 49
a = = 7.
Next, to find angle A, we have
cos A =
24
25
=
96
100
, on multiplying each term by 4.

= .96












(See Skill in Arithmetic: Fractions into decimals.)
We must now inspect the Table to find the angle whose cosine is
closest to .96, or, since this is a three place Table, .960.
We find
cos 16 = .961
Therefore,
Angle A 16.
Finally,
Angle B = 90 16 = 74.
We have solved the triangle.
Problem 3. Solve the right triangle ABC given that c = 10 cm and
b = 8 cm.
To find the remaining side a, use the
Pythagorean theorem:
a + 8 = 10
a = 100 64 = 36
a = = 6 cm.
To find angle A, we have
cos A =
8
10
= .8.
Now inspect the Table to find the angle whose cosine is closest
to .8, or, since this is a three place Table, .800.
Find cos 37 = .799.
Therefore, Angle A 37. Angle B = 90 37 = 53.


















COMPLEX
OR IMAGINARY
NUMBERS
The basic property of i
Negative radicand
Powers of i
Algebra with complex numbers
The real and imaginary components
Complex conjugates
WE ARE ABOUT TO SEE that a complex, or imaginary, number is
one whose square is negative. Compare Lesson 13.
Let us begin by recalling that in the lesson on radicals, we saw how to
solve any equation in this form:












x = a.

The solution is
x = .

If we apply that rule to this equation --

x = 1

-- then
x = .
But is not a real number. There is no positive or negative

number whose square is negative. It turns out to be extremely useful in

mathematics and science, however, to say that this equation,

x + 1 = 0,













has a solution.
is called a complex number or an imaginary number. It is the complex
unit. Its symbol is i.
i = .
The complex number i is purely algebraic. That is, we call it a
"number" because it will obey all the rules we normally associate
with a number. We may add it, subtract it, multiply it, and so on.
The following defines the complex unit i, and is its fundamental algebraic
property:
i = 1
Example 1. 3i 4i = 12i = 12(1) = 12.
Example 2. 5i 6i = 30i = 30.
We can see, then, that the factor i changes the sign of a product.
Problem 1. Evaluate the following.
a) i = 1 b) i 2i = 2i = 2(1) = 2

c) (3i) = 3i = 9 d) 5i 4i = 20i = 20

Negative radicand












If a radicand is negative --
, where a > 0,
-- then we can simplify it as follows:
= = = i .
Examples 3.


= i


= i = 2i



=
i = 2i
Problem 2. Express each of the following in terms of i.
a)
= i
b)
= 3i
c) = 7i

d)
= i
e)
= i
f)
= 2i













g)
= 3i
h)
= 5i
i)
= 7i



Powers of i
Let us begin with i
0
, which is 1. (Any number with exponent 0 is 1.) Each
power of i can be obtained from the previous power by multiplying it by i.
We have:
i
0
= 1

i
1
= i

i
2
= 1

i
3
= 1 i = i

i
4
= i i = i = (1) = 1












And we are back at 1 -- the cycle of powers will repeat! Any power of i
will be either
1, i, 1, or i
-- according to the remainder upon dividing the exponent n by 4.




Examples 4 .
i
9
= i, because on dividing 9 by 4, the remainder is 1. i
9
= i
1
.

i
18
= 1, because on dividing 18 by 4, the remainder is 2. i
18
= i
2
.

i
35
= i, because on dividing 35 by 4, the remainder is 3. i
35
= i
3
.













i
40
= 1, because on dividing 40 by 4, the remainder is 0. i
40
= i
0
.
Note: Even powers of i will be either 1 or 1, according as the exponent is a
multiple of 4 or 2 more than a multiple of 4. While odd powers will be either i or
i.
Problem 3. Evaluate each power of i.
a) i
3
= i b) i
4
= 1 c) i
6
= i
2
= 1

d) i
9
= i
1
= i e) i
12
= i
0
= 1 f) i
17
= i
1
= i

g) i
27
= i
3
= i h) i
30
= i
2
= 1 i) i
100
= i
0
= 1
Algebra with complex numbers
Complex numbers follow the same rules as real numbers. For example, to
multiply
(2 + 3i)(2 3i)
the student should recognize the form (a + b)(a b) -- which will produce the













difference of two squares. Therefore,

(2 + 3i)(2 3i) = 4 9i

= 4 9(1)

= 4 + 9

= 13.
Again, the factor i changes the sign of the term.
Problem 4. Multiply.

a) (1 + i )(1 i ) = 1 2i = 1 + 2 = 3
b) (3 i ) = 9 6i + 2i, upon squaring the binomial,

= 9 6i 2













= 7 6i
c) (2 + 3i)(4 5i) = 8 10i + 12i 15i

= 8 + 2i + 15

= 23 + 2i
Problem 5. (x + 1 + 3i)(x + 1 3i)
a) What form will that produce? The difference of two squares.
b) Multiply out.
(x + 1 + 3i)(x + 1 3i) = (x + 1) 9i

= x + 2x + 1 + 9













= x + 2x + 10
c) (x 2 i )(x 2 + i ) = (x 2) 2i


= x 4x + 4 + 2

= x 4x + 6
The real and imaginary components
Here is the standard form of a complex number:
a + bi,
where both a and b are real. For example,
3 + 2i.
a -- that is, 3 in the example -- is called the real component (or the real
part). b (2 in the example) is called the imaginary component (or the
imaginary part). Again, the components are real.
Problem 6. Name the real component a and the imaginary component b.
a) 3 5i a = 3, b = 5. b) 1 + i a = 1, b = .












c) i a = 0, b = 1. d) 6 a = 6, b = 0.
Complex conjugates
The complex conjugate of a + bi is a bi. The main point about a
conjugate pair is that when they are multiplied --
(a + bi)(a bi)
-- a positive real number is produced. For, that form is the difference of two
squares:
(a + bi)(a bi) = a bi = a + b
The product of a conjugate pair is equal to the sum of the squares of the components.
Problem 7. Calculate the positive real number that results from multiplying
each number with its complex conjugate.
a) 2 + 3i. (2 + 3i)(2 3i) = 2 + 3 = 4 + 9 = 13
b) 3 i . (3 i )(3 + i ) = = 3 + ( )= 9 + 2 = 11
c) u + iv. (u + iv)(u iv) = u + v
d) 1 + i. (1 + i)(1 i) = 1 + 1 = 2
e) i. (i)(i) = i = 1


















Word problems that lead to
simultaneous equations
Section 2
Back to Section 1: Examples

Problem 1. A woman is now 30 years older than her son. 15 years ago, she
was twice as old. What are the present ages of the woman and her son?
Let x be the present age of the woman.
Let y be 45 years.
Therefore x, the age of the woman, is 75 years.
Problem 2. A total of 925 tickets were sold for $5,925. If adult tickets cost
$7.50, and children's tickets cost $3.00, how many tickets of each kind were
sold? (Compare Example 2.)
Let x be the number of adult tickets. Let y be the number of childeren's
tickets.
Here are the equations:












1) Total number of tickets: x + y = 925
2) Total money collected: 7.5x + 3y = 5,925
In equation 2), make the coefficients into whole numbers by multiplying both
sides of the equation by 10:
1) x + y = 925
2') 75x + 30y = 59,250
To eliminate y, for example:
Multiply equation 1) by 30, and add.
The solution is: x = 700, y = 225.
Problem 3. Mr. B. has $20,000 to invest. He invests part at 6%, the rest at
7%, and he earns $1,280 interest. How much did he invest at each rate?
(Compare Example 3.)
Let x be how much he inveted at 6%. Let y be how much he inveted at
7%.
Here are the equations:
1) Total investment: x + y = 20,000
2) Total interest: .06x + .07y = 1,280
2') 6x + 7y = 128,000












To eliminate x, for example, from equations 1) and 2'):
,Multiply equation 1) by 6, and add.
The solution is: x = $12,000. y = $8,000.
Problem 4. Edgar has 20 dimes and nickels, which together total $1.40. How
many of each does he have? (Compare the Problem in Section 1.)
Let x be the number of dimes. Let y be be the number of nickels.
Here are the equations:
1) Total number of coins: x + y = 20
2) Total value: .10x + .05y = 1.40
2') 10x + 5y = 140
To eliminate x, for example, from equations (1) and (2'), multiply equation
(1) by (10), and add.
The solution is: x = 8 dimes. y = 12 nickels.
Problem 5. How many gallons of 20% alcohol solution and how many of
50% alcohol solution must be mixed to produce 9 gallons of 30% alcohol
solution? (Compare Example 4.)
(9 gallons of 30% alcohol solution = .3 9 = 2.7 gallons of pure alcohol.)
Let x be the number of gallons of 20% solution. Let y be the number of
gallons of 50% solution.
Here are the equations:
1) Total number of gallons: x + y = 9












2) Total gallons of pure alcohol: .2x + .5y = 2.7
2') 2x + 5y = 27
To eliminate x, for example, from equations 1) and 2'), multiply equation 1)
by 2, and add.
The solution is: x = 6 gallons. y = 3 gallons.
Problem 6. 15 gallons of 16% disenfectant solution is to be made from 20%
and 14% solutions. How much of those solutions should be used?
(15 gallons of 16% solution = .16 15 = 2.4 gallons of pure disenfectant.)
Let x be the number of gallons of 20% solution. Let y be the number of
gallons of 14% solution.
Here are the equations:
1) Total number of gallons: x + y = 15
2) Total gallons of pure disenfectant: .20x + .14y = 2.4
2') 20x + 14y = 240
To eliminate x, for example, from equations 1) and 2'), multiply equation 1)
by 20, and add.
The solution is: x = 5 gallons. y = 10 gallons.
Problem 7. It takes a boat 2 hours to travel 24 miles downstream and 3 hours
to travel 18 miles upstream. What is the speed of the boat in still water, and
how fast is the current? (Compare Example 6.)












Let x be the speed of the boat in still water. Let y be the speed of the current.
Here are the equations:

To eliminate y, simply add the equations.
The solution is: x = 9 mph. y = 3 mph.


Problem 8. An airplane covers a distance of 1500 miles in 3 hours
when it flies with the wind, and in 3
1
3
hours when it flies against the
wind. What is the speed of the plane in still air? (Compare Example 6.)
Let x be the speed of the plane in still air. Let y be the speed of the wind.
Here are the equations:
1) Speed with the wind: x + y =
1500
3
= 500
2) Speed against the wind : x y =

To eliminate y, simply add the equations.
1) Downstream speed: x + y =
24
2
= 12
2) Upstream speed: x y =
18
3
= 6












The solution is: x = 475 mph.
Back to Section 1




WORD PROBLEMS
THAT LEAD TO
SIMULTANEOUS EQUATIONS
Section 1: Examples
General problem
Ticket problem
Investment problem
Coin problem
Mixture problems
Upstream/Downstream problem
Section 2: Problems












HERE ARE SOME EXAMPLES of problems that lead to
simultaneous equations.
Example 1. Andre has more money than Bob. If Andre gave Bob $20, they
would have the same amount. While if Bob gave Andre $22, Andre would
then have twice as much as Bob. How much does each one actually have?
Solution. Let x be the amount of money that Andre has. Let y be the amount that Bob has.
Always let x and y answer the question -- and be perfectly clear about what they represent!
Now there are two unknowns. Therefore there must be two equations. (In general, the number
of equations must equal the number of unknowns.) How can we get two equations out of the
given information? We must translate each verbal sentence into the language of algebra.
Here is the first sentence:
"If Andre gave Bob $20, they would have the same amount."
Algebraically:
1) x 20 = y + 20.
(Andre -- x -- has the same amount as Bob, after he gives him $20.)
Here is the second sentence:
"While if Bob gave Andre $22, Andre would then have twice as much
as Bob."
Algebraically:
2) x + 22 = 2(y 22).
(Andre has twice as much as Bob -- after Bob gives him $22.)
To solve any system of two equations, we must reduce it to one
equation in one of the unknowns. In this example, we can solve equation 1)
for x --
x 20 = y + 20













implies x = y + 40
-- and substitute it into equation 2):
y + 40 + 22 = 2(y 22).
That is,
y + 62 = 2y 44,

y 2y = 44 62,

according to the techniques of Lesson 9,

y = 106

y = 106.
Bob has $106. Therefore, according to the exression for x, Andre has
106 + 40 = $146.













Example 2. 1000 tickets were sold. Adult tickets cost $8.50, children's cost
$4.50, and a total of $7300 was collected. How many tickets of each kind
were sold?
Solution. Let x be the number of adult tickets. Let y be the number of children's tickets.
Again, we have let x and y answer the question. And again we must get two equations out of the
given information. Here they are:
1) Total number of tickets: x + y = 1000

2) Total money collected: 8.5x + 4.5y = 7300
In equation 2), we will make the coefficients into whole numbers by
multiplying both sides of the equation by 10:
1) x + y = 1000

2') 85x + 45y = 73,000
We call the second equation 2' ("2 prime") to show that we obtained it
from equation 2).
These simultaneous equations are solved in the usual way.
The solutions are: x = 700, y = 300.
To see the answer, pass your mouse over the colored area.
To cover the answer again, click "Refresh" ("Reload").
Do the problem yourself first!













Example 3. Mrs. B. invested $30,000; part at 5%, and part at 8%. The total
interest on the investment was $2,100. How much did she invest at each
rate?
Solution.
1) Total investment: x + y = 30,000

2) Total interest .05x + .08y = 2,100
(To change a percent to a decimal, see Skill in Arithmetic, Lesson 4.)
Again, in equation 2) let us make the coefficients whole numbers by
multiplying both sides of the equation by 100:
1) x + y = 30,000

2') 5x + 8y = 210,000
These are the simultaneous equations to solve.
The solutions are: x = $10,000, y = $20,000.
Problem. Samantha has 30 coins, consisting of quarters and dimes, which
total $5.70. How many of each does she have?
To see the answer, pass your mouse from left to right
over the colored area.
To cover the answer again, click "Refresh" ("Reload").
Do the problem yourself first!












Let x be the number of quarters. Let y be the number of dimes.
The equations are:
1) Total number of coins: x + y = 30

2) Total value: .25x + .10y = 5.70
To eliminate y:
Multiply equation 1) by 10 and equation 2) by 100:
1') 10x 10y = 300

2') 25x + 10y = 570

Add:

15x = 270

x =
270
15













x = 18
Therefore, y = 30 18 = 12.

Example 4. Mixture problem 1. First:
"36 gallons of a 25% alcohol solution"
means: 25%, or one quarter, of the solution is pure alcohol.
One quarter of 36 is 9. That solution contains 9 gallons of pure alcohol.
Here is the problem:
How many gallons of 30% alcohol solution and how many of 60%
alcohol solution must be mixed to produce 18 gallons of 50% solution?
"18 gallons of 50% solution" means: 50%, or half, is pure alcohol. The
final solution, then, will have 9 gallons of pure alcohol.
Let x be the number of gallons of 30% solution. Let y be the number of
gallons of 60% solution.
1) Total number of gallons x + y = 18

2) Gallons of pure alcohol .3x + .6y = 9

2') 3x + 6y = 90
Equations 1) and 2') are the two equations in the two unknowns.
The solutions are: x = 6 gallons, y = 12 gallons.













Example 5. Mixture problem 2. A saline solution is 20% salt. How much
water must you add to how much saline solution, in order to dilute it to 8
gallons of 15% solution?
(This is more an arithmetic problem than an algebra problem.)
Solution. Let s be the number of gallons of saline solution. Now all the salt will come from
those s gallons. So the question is, What is s so that 20% of s -- the salt -- will be 15% of 8
gallons?
.2s = .15 8 = 1.2
That is,
2s = 12.
s = 6.
Therefore, to 6 gallons of saline solution you must add 2 gallons of
water.

Example 6. Upstream/Downstream problem. It takes 3 hours for a boat to
travel 27 miles upstream. The same boat can travel 30 miles downstream in 2
hours. Find the speeds of the boat and the current.
Solution. Let x be the speed of the boat (without a current). Let y be the
speed of the current.
The student might review the meanings of "upstream" and "downstream," Lesson 25. We saw
there that speed, or velocity, is distance divided by time:
v =
d
t
Therefore, according to the problem:












Upstream speed =
Upstream distance
Upstream time
=
27
3
= 9

Downstream speed =
Downstream distance
Downstream time
=
30
2
= 15
Here are the equations:
1) Upstream speed: x y = 9


2) Downstream speed: x + y = 15
Enjoy!
(The solutions are: x = 12 mph, y = 3 mph.)
Section 2: Problems


















THE DERIVATIVE
The rate of change of a function
at a specific value of x
The slope of a straight line
The slope of a tangent line to a curve
A secant to a curve
The definition of the derivative
The derivative of f(x)= x
Differentiable at x
Notations for the derivative
A simple difference quotient
Section 2: Problems
The derivative of f(x) = 2x 5
The equation of a tangent to a curve
The derivative of f(x) = x

CALCULUS IS CONCERNED WITH THINGS that do not change at a
constant rate. The values of the function called the
derivative
will be that varying rate of change.
Now, the slope of a straight line indicates a constant rate
of change.













As we move from any point A on the line to any point B, the
slope is the number
y
x
=

=
Change in y-cordinate
Change in x-cordinate
.
(Topic 8 of Precalculus.)
That number indicates how the value of y changes when
the value of x changes. y/x is constant. A straight line has
one and only one slope.
If x represents time, for example, and y represents distance, then a













straight line graph that relates them indicates constant speed.
45 miles per hour, say -- at every moment of time.

The slope of a tangent line to a curve
Calculus however is concerned with rates of change that are
not constant.

If this curve represents distance Y versus time X, then
the rate of change the speed at each moment of time is
not constant. The question that calculus asks is: "What is the












rate of change at exactly the point P ?" If we can name the
slope of a tangent line to the curve at that point, then that will
be the answer. And the method for finding that slope that
number was the remarkable discovery by both Isaac
Newton (1642-1727) and Gottfried Leibniz (1646-1716). That
method is the one for finding what is called the derivative.
A secant to a curve

A secant is a straight line that cuts a curve. (A tangent is a
straight line that just touches a curve.) Hence, consider a
secant line that cuts the curve at points P and Q. Then the
slope of the secant is the average rate of change between those
two points. For example, if

then on changing from x1 to x2, the function has changed an
average of 4 units of y for every 5 units of x. But once again,
the question calculus asks is: How is the function changing
exactly at x1? What is the slope of














the tangent to the curve at P?
We cannot, of course,
evaluate
y
x
exactly at P --
because y
and x
would then both be 0, and
the value of
y
x
would be
completely
ambiguous.
Therefore we will consider shorter and shorter intervals of x, which will
result in a sequence of secants --













-- a sequence of slopes. And we will define the tangent at P to
be the limit of that sequence

That slope, that limit, will be the value of what we will
call the "derivative."

The definition of the derivative

Let y = f(x) be a continuous function, and let the
cordinates of a fixed point P on the graph be (x, f(x)). (Topic
4 of Precalculus.) Let x now change by an amount x. Then
the new x-cordinate is x + x.
It is the x-cordinate of Q on the graph.
But when the value of x changes, there is a corresponding change y
in the value of y, that is, in the value of f(x). Its new value is f(x + x). The
cordinates of Q are (x + x , f(x + x)).
Then













Finally:
The slope of the tangent line at
P
is the limit of the ratio of the
change in the function
to the change in the independent
variable,
as that change approaches 0.


Since x -- not x -- is the variable that
approaches 0, x will remain constant, and that
limit will be a function of x. Since it will be
derived from f(x), we call it the derived function
or the derivative of f(x). And to remind us
that it was derived from f(x), we denote it by f
'(x) -- "f-prime of x."
Since the
theorems on
limits make it
easy to
evaluate a
limit, it is easy
to lose sight of
its actual
meaning. With












regard to the
derivative, it is
the number
which that ratio
can approach
as closely as
we desire.

This quotient --

-- is called the Newton quotient, or the
difference quotient. Calculating and
simplifying it is a fundamental task in
differential calculus.
Again, the difference quotient is a function of x.
But to simplify our written calculations, instead of
writing x, we will write h.













x = h

y = f (x + h) f (x)
The difference quotient then becomes

We now express the definition of the
derivative as follows.


DEFINITION 5. By the derivative of a
function f(x), we mean the following limit, if
it exists:

We call that limit f '(x) -- "f-prime of
x" -- and we say that f itself is differentiable
at x, and that f has a derivative.

Again, in taking that limit, the variable
that is approaching 0 is h, not x, and we are
to regard x as being fixed. It is the specific
value at which we are evaluating the rate of
change of f(x).












In practice, we have to simplify the
difference quotient before letting h
approach 0. We have to express the
numerator --
f (x + h) f (x)
-- in such a way that we can divide it by h.
As an example, we will apply the definition to prove
the following:

THEOREM. f(x) = x

implies

f '(x) = 2x.
Proof. Here is the difference quotient,
which we will proceed to simplify:
1)
(x + h) x
h

2) =
x + 2xh + h x
h













3) =
2xh + h
h

4) = 2x + h.
In going from line 1) to line 2), we
squared the binomial x + h. (Lesson 18 of
Algebra.)
In going to line 3), we subtracted the xs. That is, we
subtracted f(x).
In going to line 4), we divided the numerator by h.
(Lesson 20 of Algebra.)
We can do that because h is never equal to 0, even
when we take the limit (Lesson 2).
We now complete the definition of the derivative and
take the limit:
f '(x) =

(2x + h)

= 2x.
This is what we wanted to prove.














Whenever we apply the definition, we have to
algebraically manipulate the difference quotient so
that we can simply replace h with 0. In fact, the
entire theory of limits, with all its complexities and
subtleties, was invented to justify just that (Poor
Newton and Leibniz were criticized for offering
justifications that the inventors of limits didn't like.)
We may put h = 0 here, because the difference
quotient reduces to 2x + h, and is therefore a
polynomial in h.
Differentiable at x
According to the definition, a function will
be differentiable at x if a certain limit exists
there. Graphically, this means that the
graph at that value of x will have a tangent
line. At which values, then, would a
function not be differentiable?
Where it does not have a tangent line














Above are two examples. The function on the left
does not have a derivative at x = 0, because the
function is discontinuous there. At x = 0 there is
obviously no tangent.
As for the graph on the right, it is the absolute value
function, y = |x|. (Topic 5 of Precalculus.) And it is
not possible to define the tangent line at x = 0,
because the graph makes an acute angle there. In
fact, the slope of the tangent line as x approaches 0
from the left, is 1. The slope approaching from the
right, however, is +1. The slope of the tangent line
at 0 -- which would be the derivative at x = 0 --
therefore does not exist . (Definition 2.2.)
The absolute value function
nevertheless is continuous at x = 0. For,
the left-hand limit of the function itself as x
approaches 0 is equal to the right-hand
limit, namely 0. This illustrates that
continuity at a point is no guarantee of
differentiability -- the existence of a tangent
-- at that point.
(Conversely, though, if a function is differentiable at
a point -- if there is a tangent -- it will also be
continuous there. The graph will be smooth and
have no break.)
Since differential calculus is the study of derivatives,
it is fundamentally concerned with functions that are
differentiable at all values of their domains.
Such functions are called differentiable functions.
Can you name an elementary class of differentiable
functions?
To see the answer, pass your
mouse over the colored area.












To cover the answer again, click
"Refresh" ("Reload").
Think about this yourself first!
P
o
l
y
n
o
m
i
a
l
s
.
Notations for the derivative
Since the derivative
is this limit:

then a symbol
for the
derivative is

(Read: "dee-y, dee-x.")

For example, if












y = x,


then, as we have seen,


= 2x.
"Dee-y, dee-x -- the derivative of y with
respect to x -- is 2x."
We also write
y '(x) = 2x.
"y-prime of x is equal to 2x."
This symbol
by itself:
d
dx
("dee,
dee-x")
, is called the
differentiating

operator. We are to take the derivative of
what follows it. For example,
d
dx
f(x) signifies the derivative with respect to
x of f(x).

d
dt
(4t
3
5) signifies the derivative with
respect to t of (4t
3
5).

And so on.













A simple difference quotient


The difference quotient is a
version of
y
x
. And at
times we
will use the latter. That is, the change in
the value of a function y = f(x) is y + y.
Hence the difference quotient is

At times it will be convenient to
express the difference quotient as


Note: As x approaches 0 -- as the
point Q moves closer to P along the












curve -- then y, or equivalently, f also
approaches 0. That is,

The student should now do
Problems that require the definition of
the derivative.























RULES
FOR
DERIVATIVES
The derivative of a constant
The derivative of y = x
The derivative of a sum or difference
The derivative of a constant times a function
The product rule
The power rule
The derivative of the square root

THE DEFINTION of the derivative is fundamental. (Definition 5.)
The student should be thoroughly familiar with it. From that
definition it is possible to prove various rules, some of which we
will present in this Lesson. The student will find it extremely
helpful to state each rule verbally.
For example,

1.

"The derivative of a constant is 0."












We should expect this, because the slope of a horizontal line
y = c is 0.

2.

"The derivative of a variable with respect to itself is 1."
Again, this is an expected result, because 1 is the slope of the
straight line y = x. (Topic 9 of Precalculus.)

3.

"The derivative of a sum or difference
is equal to the sum or difference of the derivatives."
This follows from Theorem 1 on limits, Lesson 2.
For example,
d
dx
(x x + 5) = 2x 1,
according to the Theorem of Lesson 4, and 1, 2, and 3 above.













4.
d
dx
c f(x) = c
d
dx
f(x)
"The derivative of a constant times a function
is equal to the constant times the derivative of the function."
d
dx

5x = 5 2x = 10x.

d
dx

8x = 8 1 = 8.
This follows from Theorem 5 on limits, Lesson 2.
Problem 1. Calculate the derivative of 4x 6x + 2.
To see the answer, pass your mouse over the colored area.
To cover the answer again, click "Refresh" ("Reload").
Do the problem yourself first!
8x 6.
Problem 2. Calculate the derivative of y =
x
a
, where a is a constant.
Since
x
a
=
1
a
x
(Lesson 5 of Algebra),
then y' =
1
a
.
Rule 4.













5. ( f g)' = f g' + g f '
"The derivative of a product of two functions is equal
to
the first times the derivative
of the second
plus the second times the
derivative of the first."
This is the product rule. We will prove it below.
Example. Accepting for the moment that the derivative of
sin x is cos x (Lesson 12), then
d
dx

x sin x = x cos x + 2x sin x.
Problem 3. Calculate the derivative of 5x sin x.


5x cos x + 5 sin x
Proof of the product rule
To prove the product rule, we will express the difference
quotient simply
as
y
x
. (Lesson 5.) And so let












y = f g.
Then a change in y -- y -- will produce corresponding
changes in f and g:
y + y = (f + f )(g + g)
On multiplying out the right-hand side,
y + y = f g + f g + g f + f g.
To form
y
x
, we subtract y -- which is f g -- from both sides, and
divide by x:
y
x
= f
g
x
+ g
f
x
+ f
g
x
Now let x 0. Hence y will approach 0, as will
both f and g, so that the last term on the right approaches
0.
Therefore, since the limit of a sum is equal to the sum of the limits
(Theorem 1 of limits):
dy
dx
= f
dg
dx
+ g
df
dx
This is the product rule.

The power rule
6.
d

x
n
= nx
n1













dx
"The derivative of a power of x
is equal to the product of
the exponent times
x with the exponent reduced by
1."
That is called the power rule. For example,
d
dx

x
5
= 5x
4

It is usual to prove the power rule by means of the
binomial theorem. See Topic 24 of Precalculus, especially
Problem 5. On applying the definition of the derivative,
subtracting x
n
, dividing the numerator by h and taking the
limit, the rule follows.
However, we have seen that the power rule is true when n = 1:

d
dx
x
1
= 1 x
0
= 1;

that it is true when n = 2:














d
dx
x
2
= 2x;

and that it is true when n = 3 (Problem 2 of Lesson 5):



d
dx
x
3
= 3x.
It seems natural, then, to give a proof by induction;
(Topic 26 of Precalculus). The induction hypothesis will
be that the power rule is true for n = k:
d
dx
x
k
= k x
k1
,
and we must show that it is true for n = k + 1; i.e. that
d
dx
x
k+1
= (k + 1) x
k
.
Now,
d
dx
x
k+1
=
d
dx
x x
k













d
dx
x
k+1

=
x k x
k1
+ x
k
1, according to the product rule,

d
dx
x
k+1

= k x
k
+ x
k


d
dx
x
k+1

= (k + 1)x
k
.
Therefore, if the power rule is true for n = k, then it is also true for its
succesor, k + 1. And since the rule is true for n = 1, it is therefore true
for every natural number.
Problem 4. Calculate the derivative of x
6
3x
4
+ 5x
3

x + 4.
6x
5
12x
3
+ 15x 1

We will see in Lesson 14 that the power rule is valid
for any exponent n. The student should begin
immediately to use that result.

Example. The derivative of the square root.













See Lesson 29 of Algebra: Rational Exponents.
Problem 5. Calculate the derivative of .

Problem 6. Calculate the derivative of x .

Problem 7. Caculate the derivate of
1
x
5

.
d
dx
1
x
5

=
d
dx
x
5
= 5x
6





THE CHAIN RULE
The derivative of a function of a function












The chain rule
Proof of the chain rule

The derivative of a function of a function
Let
f (x) = x
5
and g(x) = x + 1.
If we now let g(x) be the argument of f, then f will be a function of g.

f (g(x)) = (x + 1)
5
.
(Topic 3 of Precalculus.)
What is the derivative of f (g(x)) ?
First, note that
d f(x)
dx

= 5x
4
.
That is: The derivative of f with respect to its argument (which in
this case is x) is equal to 5 times the 4th power of the argument.
This means that if g -- or any variable -- is the argument of f, the same form applies:
d f(g)
dg

= 5g
4
.
d f(h)
dh

= 5h
4
.
d f(v)
dv

= 5v
4
.












In other words, we can really take the derivative of a function of an
argument only with respect to that argument.
Therefore, since g = x + 1,
d f(g)
dg

= 5g
4
= 5(x + 1)
4
.
Next, the derivative of g is 2x. And what is called the chain rule
states the following:

df(g(x))
dx
=
df(g)
dg

dg(x)
dx
"If f is a function of g and g is
a function of x,
then the derivative of f with
respect to x
is equal to the derivative of
f(g) with respect to g
times the derivative of g(x)
with respect to x."
Therefore according to the chain rule, the derivative of
(x + 1)
5

is
5(x + 1)
4
2x.
Note: In (x + 1)
5
, x + 1 is "inside" the 5th power, which
is "outside." We take the derivative from outside to inside. When












we take the outside derivative, we do not change what is inside. We
then multiply by the derivative of what is inside.
To decide which function is outside, decide which you would have to evaluate
last.
To evaluate
(x + 1)
5
,
you would first have to evaluate x + 1. Then you would take its
5th power. The 5th power therefore is outside. That is why we take
that derivative first.
When we write f(g(x)), f is outside g.
We take the derivative of f with
respect to g first.
Example 1. f(x) =
. What is its derivative?
Solution. This has the form f (g(x)). What function is f, that is,
what is outside, and what is g, which is inside?
g is x
4
2, because that is inside the square root function, which is f. The
derivative of the square root is given in the Example of Lesson 6. For any
argument g of the square root function,

Here, g is x
4
2. Therefore, since the derivative of x
4
2
is 4x
3
,
d
dx

= (x
4
2)

4x
3
= 2x
3
(x
4
2)

.
Example 2. What is the derivative of y = sin
3
x ?












Solution. This is the 3rd power of sin x. To decide which function is outside,
how would you evaluate that?
You would first evaluate sin x, and then take its 3rd power. sin x is inside the
3rd power, which is outside.
Now, the derivative of the 3rd power -- of g
3
-- is 3g. Therefore, accepting for
the moment that the derivative of sin x is cos x (Lesson 12), the derivative of
sin
3
x -- from outside to inside -- is
3 sinx cos x.
Example 3. What is the derivative of
1
x
3
+ 1
?
Solution. x
3
+ 1 is inside the function
1
x
= x
1
, whose derivative
is x
2
; (Problem 3, Lesson 4). We have, then,
1
x
3
+ 1
= (x
3
+ 1)
1
.
Therefore, its derivative is
(x
3
+ 1)
2
3x

Example 4. Assume that y is a function of x. y = y(x). Apply
the
chain rule to
d
dx
y .
Solution.
dy
=
dy

dy
= 2y
dy
.












dx dy dx dx
y, which we are assuming to be a function of x, is inside the
function y. The derivative of y with respect to y is 2y. As for
the derivative of
y with respect to x, we can only indicate it as
dy
dx
. (See Lesson 5.)
Problem 1. Calculate the derivative of (x 3x + 5)
9
.
To see the answer, pass your mouse over the colored area.
To cover the answer again, click "Refresh" ("Reload").
Do the problem yourself first!
9(x 3x + 5)
8
(2x 3)
Problem 2. Calculate the derivative of (x
4
3x + 4)
2/3
.
2/3(x
4
3x + 4)
1/3
(4x
3
6x)
Problem 3. Calculate the derivative of sin
5
x.
5 sin
4
x cos x
Problem 4. Calculate the derivative of sin x
5
.
The inside function is x
5
-- you would evaluate that last.
The outside function is sin x. (This is the sine of x
5
.) Therefore, the
derivative is
cos x
5
5x
4
.
Problem 5. Calculate the derivative of sin (1 + 2 ).
cos (1 + 2 )x
1/2













Problem 6. Calculate the derivative of
(sin x)
3/4
cos x
Example 5. More than two functions. The
chain rule can be extended to more than two
functions. For example, let
f(x) =

.
The outside function is the square root.
Inside that is (1 + a 2nd power). And inside that
is sin x.
The derivative therefore is
(1 + sinx)
1/2
2 sin x cos x =
sin x cos x

.
Problem 7. Calculate the derivative of

(Compare Example 3.)
[sin (x + 5)]
2
cos (x + 5) 2x
=

2x cos (x + 5)
sin(x + 5)
Problem 8. Calculate the derivative of














Problem 9. Assume that y is a function of x, and
apply the chain rule to express each derivative
with respect to x.
a)
d
dx

y
3
=
3y
dy
dx
b)
d
dx

sin y =
cos y
dy
dx
c)
d
dx

=
y


dy
dx
Proof of the chain rule
To prove the chain rule let us go back to basics.
Let f be a function of g, which in turn is a
function of x, so that we have f(g(x)). Then when
the value of g changes by an amount g, the value
of f will change by an amount f. We will have the
ratio
f
g
.












Again, since g is a function of x, then when x
changes by an amount x, g will change by an
amount g. We will have the ratio
g
x
.
But the change in x affects f because it
depends on g. We will have
f
x
. It will be the product of those ratios:
f
x
=
f
g

g
x
.
Let us now take the limit as x approaches 0.
Then the change in g(x) -- g -- will also approach
0. Therefore, since the limit of a product is equal to the
product of the limits (Lesson 2), and by definition of
the derivative:
df
dx
=
df
dg

dg
dx
This is the chain rule.





















MORE RULES
FOR
DERIVATIVES
The quotient rule
Proof of the quotient rule
Implicit differentiation
The derivative of an inverse function

The quotient rule
The following is called the quotient rule:

"The derivative of the quotient of two functions is equal to












the denominator times the derivative of the numerator
minus the numerator times the derivative of the denominator
all divided by the square of the denominator."
For example, accepting for the moment that the derivative of sin
x is cos x (Lesson 12):

Problem 1. Calculate the derivative of
x
sin x
.
To see the answer, pass your mouse over the colored area.
To cover the answer again, click "Refresh" ("Reload").
Do the problem yourself first!
sin x 2x x cos x
sin
2
x
Problem 2. Use the chain rule to calculate the derivative of
sinx
x
3

.
x 2 sin x cos x sinx 3x
x
6

=
x sin x(2x cos x 3 sin x)
x
6


=
sin x(2x cos x 3 sin x)
x
4













Problem 3. Calculate the derivative of
x 5x 6
2x + 1
.
(2x + 1)(2x 5) (x 5x 6) 2
(2x + 1)
=
4x 8x 5 2x + 10x + 12
(2x + 1)

=
2x + 2x + 7
(2x + 1)
Problem 4. Calculate the derivative of
3x x + 4

.
See the Example, Lesson 6.

=


=


=














Proof of the quotient rule
THEOREM.

Proof. Since g = g(x), then
d
dx
1
g
=
d
dg

1
g

dg
dx
=
1
g
g'
according to the chain rule, and Problem 3 of Lesson 5.
Therefore, according to the product rule (Lesson 6),


This is the quotient rule, which we wanted to prove.


Implicit differentiation












Consider the following:
x + y = r
This is the equation of a circle with radius r. (Lesson 17 of
Precalculus.)
Let us calculate
dy
dx
. To do that, we could solve for y and then take the
derivative. But rather than do that, we will take the derivative of each
term. As for y, we consider it implicitly a function of x, and therefore
we may apply the chain rule to it. Then we will solve for
dy
dx
.

d
dx
x +
d
dx
y =
d
dx
r
2x + 2y
dy
dx
= 0

dy
dx
=
x
y
.
This is called implicit differentiation. We treat y as a function of
x and apply the chain rule. The derivative that results generally
contains both x and y.
Problem 5. 15y + 5y
3
+ 3y
5
= 5x
3
. Calculate y'.












15y' + 15yy' + 15y
4
y'
=
15x
2


y'(1 + y + y
4
)
=
x
y'
=
x
1 + y + y
4

Problem 6.

Calculate y'.

=
0


=


y'
=

Problem 7.
a) In this circle,
x + y = 25,
a) what is the y-cordinate when x = 3?












y = 4 or 4. For,
(3) + (4) = 5
b) What is the slope of the tangent to the circle at (3, 4)?
3
4
. For, the derivative is
x
y
.
c) What is the slope of the tangent to the circle at (3, 4)?

3
4
Problem 8. In the first quadrant, what is the slope of the
tangent to this circle,
(x 1) + (y + 2) = 169,
when x = 6?
[Hint: 5 + 12 = 13 is a Pythagorean triple.]
In the first quadrant, when x =
6, y = 10.
(6 1) + (10 + 2)
= 13.
y' =
x 1
y + 2
. Therefore the slope is
6 1
10 + 2
=
5
12
Problem 9. Calculate the slope of the tangent to this curve
at (2, 1):












x
3
3xy + y
3
= 1

3x (3x 2y y' + y 3) + 3y y'
=
0

according to the product rule.


3x 6xy y' 3y + 3y y'
=
0


x 2xy y' y + y y'
=
0


y'(y 2xy)
=
y x


y'
=
y x
y 2xy

Therefore, at (2, 1):


y'
=
(1) 2
(1) 2 2 1













=
3
5

=

3
5
The derivative of an inverse function
When we have a function y = f(x) -- for example
y = x
-- then we can often solve for x. In this case,

On exchanging the variables, we have


is called the inverse function of y = x.
Let us write
f(x) = x

g(x) =

And let us call f the direct function and g the inverse function.
The formal relationship between f and g is the following:












f( g(x)) = g( f(x)) = x.
(Topic 19 of Precalculus.)
Here are other pairs of direct and inverse functions:
f(x) = sin x g(x) = arcsin x

f(x) = a
x
g(x) = log
a
x

f(x) = x
3
g(x) =

Now, when we know the derivative of the direct
function f, then from it we can determine the derivative of g.
Thus, let g(x) be the inverse of f(x). Then
f(g(x) = x.
Now take the derivative with respect to x:

This implies the following

Theorem.














"The derivative of an inverse function is
equal to
the reciprocal of the derivative of the direct
function
when its argument is the inverse
function."
Example. Let f(x) = x, and

Then f( g) = g.
Therefore,



INSTANTANEOUS VELOCITY
AND
RELATED RATES
Rectilinear motion
The definition of instantaneous velocity
Related rates
ONE OF THE most important applications of calculus is to motion in a
straight line, which is called rectilinear motion.













Consider a point P moving in a straight line. Let s be the distance
measured from a fixed point O to any position of P, and let t be the time
elapsed. Then to each value of t there will correspond a distance s. s will
be a function of t:
s = f(t).
When we know f(t), we have what is called the equation of motion.
Now, in a time interval t, there will be a change s -- P will move to
the point P'. By definition,
s
t
= The average velocity in moving from P to P'.
For example, if at t = 30 sec, s = 100 meters,
and at t = 35 sec, s = 210 m, then
s
t
=
110 m
5 sec
= 22 meters per second.
But that is the average velocity in moving between those two points.
The question calculus asks is: What is the exact velocity at the point P
itself?

If P moves with constant velocity -- which is called uniform motion
-- then we don't need calculus for that. In other words, if the equation
of motion is
s = 22 t,












then at every instant of time, the velocity is 22 m/sec. For, the slope of
that line, which is 22, is rate of change of s with respect to t, which by
definition is the velocity.

In each 1 second of time, the point P moves a distance of 22
meters.
s
t
= 22 meters per second.
That is not a realistic picture, of course, because at 0 seconds the
velocity is surely not 22 meters/sec There must have been an
acceleration
to that constant velocity. During that acceleration, the velocity was not
constant. The graph was not a straight line.
The definition of instantaneous velocity














For any kind of motion, uniform or not, by v(t) -- the instantaneous
velocity at time t of a moving object whose position is s(t) -- we mean
the
limit of the average velocity,
s
t
, as t approaches 0.

v(t), velocity, is a continuous function. Therefore according to the
definition of a continuous function, we may name the limit of v(t) as t
approaches any value t
0
, by evaluating v(t
0
).
*
The variable t is approaching 0 as a limit.
Therefore t is never equal to 0. What is
called the "instantaneous velocity at time t" is
a definition. It is the value of s/t can be
approached as closely as we desire. If we took
that expression literally -- "the instantaneous
velocity at 3:06:55 PM" -- then at that instant
there would be no motion! Motion requires a
change of time with its corresponding change












of distance. In other words, if we think of time
(or any quantity modeled on a straight line)
as actually composed of points, or instants,
rather than intervals, then we create
unnecessary paradoxes. That was recognized
in ancient times, and is one of the paradoxes
of Zeno.
See the Introduction to plane geometry.
Example. Let the following be the equation of motion:
s(t) = 6t + t + 8
Let t be measured in seconds and s in meters.
a) What is the position -- the distance from the origin -- at the end
of 10 seconds?
Answer. s(10) = 6 10 + 10 + 8 = 618 meters.
b) What is the velocity at the end of 10 seconds?
Answer.
ds
dt
= 12t + 1

= 12 10 + 1

= 121 m/sec.
Problem 1. It has been found by experiment that a body falling
from rest under the influence of gravity, follows approximately this
equation of motion:












s(t) = 4.9 t .
s is the distance fallen measured in meters; t is the time elapsed
measured in seconds.
a) At the end of 3 seconds, how far has the body fallen?
To see the answer, pass your mouse over the colored area.
To cover the answer again, click "Refresh" ("Reload").
Do the problem yourself first!
s(3) = 4.9 3 = 4.9 9 = 44.1 m.
b) What is its velocity at the end of 3 seconds?
ds
dt
=
9.8 t


=
9.8 3


=
29.4 m/sec.

The second derivative
The derivative of y = f(x) -- y'(x) -- will itself be a function of x.
This new function may also be differentiable, in which case we
call the derivative of the first derivative of y the second derivative.
The notation for












the second derivative is

or y''.
Consider this equation of motion,
s(t) = 3t .
Then the first derivative is the velocity v:
v =
ds
dt
= 6t.
The second derivative is the rate of change of the velocity with respect to time.
That is called the acceleration a:
a =
ds
dt
= 6.
If t is measured in seconds and s in meters, then the units of
velocity are meters per second, which we abbreviate as m/sec. The
units of acceleration are then meters per second per second, which we
abbreviate as m/sec.
Problem 2. A body moves in a straight line according to this
equation of motion:
s(t) = 10t 4t + 8,
where t is measured in seconds and s in meters.
a) What is its position at the end of 5 sec?
s(5) = 10 5 4 5 + 8 = 238 m.
b) What is the equation for its velocity v at any time t ?
v(t) =
ds
= 20t 4.












dt
c) What is its velocity v at the end of 5 seconds?
v(5) =
20 5 4
= 96
m/sec.
d) What is the equation for its acceleration a at any time t ?
a(t) =
ds
dt
=
dv
dt
= 20.
e) What is its acceleration at the end of 5 seconds?
a(5) = 20 m/sec.
The acceleration is constant.
Problem 3. Under the influence of gravity, a body
moves according to this equation of motion:
s(t) = gt + s
0

a) What is the physical significance of the constant s
0
?
It is the body's initial position, s(0).
b) How fast is the body moving after 5 seconds?
v(t) =
ds
dt
= gt. At t = 5, v(5) = 5g.
c) What is the physical significance of the constant
g?












g =
ds
dt
. g is the acceleration due to gravity, which is
approximately 9.8 m/sec.
Related rates
Example 1.
a) If the radius of a circle is expanding, write the
equation that shows
a) how fast the area of the circle is expanding.
(A = r.)
Solution. We are to calculate
dA
dt
. We have
A = r.

On differentiating implicitly with respect to t (Lesson 8):

dA
dt
=

=

b) If the radius is expanding at the rate of 2
cm/min, how fast is the area
a) expanding when the radius is 15 cm?












Solution. We are given
that
dr
dt
= 2 cm/min. Therefore,
when
r = 15 cm,
dA
dt
= 2 15 2
188.4 cm/min.
Example 2. A boy is walking at the rate of 5
miles per hour toward the foot of a flag pole 60
feet high. At what rate is his distance from the
top of the pole changing when he is 80 feet from
its foot?
Solution. Draw a picture In all problems of this type,
draw a picture.

Let the boy be at the point A, which is a
distance x from the foot of the flag pole. Let s be
his distance from the top of the pole.
We are to calculate
ds
dt
.
The figure is a right triangle. Therefore,












1) s = x + 3600.
Differentiate implicitly with respect to t.
2s
ds
dt
= 2x
dx
dt
.
Therefore,
ds
dt
=
x
s
dx
dt
.
Now, we are given x = 80 feet,
and
dx
dt
= 5 mi/hour. We write
the
minus sign because x is decreasing as he
approaches the flag pole.
According to line 1),

Therefore,

Problem 4. The side of an square is a cm long,
and is increasing at the rate of b cm per hour.
How fast is the area increasing?
2ab cm/hour.












Problem 5. The side of an equilateral triangle
is a cm long, and is increasing at the rate of b
cm per hour. How fast is the area increasing?
ab cm/hour.
Problem 6.
a) The surface area S of a sphere is given by
the formula S = 4r.
If r changes with time, how does S change?
dS
dt
= 8r
dr
dt
b) The volume V of a sphere is given by the
formula V =
4
3
r
3
.
If r changes with time, how does V change?
dV
dt
= 4r
dr
dt
c) Prove:
dV
dt
= r
dS
dt
.
dV
dt
= 4r
dr
dt
= 8r
dr
dt
= r 8r
dr
dt
= r
dS
dt
.












Problem 7. The base and height of a
rectangle are b and h, and they are
changing at the rates p, q respectively.
Prove that the area A is changing at the
rate bq + hp.
A = bh.
Accordingto
the product
rule:
dA
dt
= b
dh
dt
+ h
db
dt
= bq + hp.
Problem 8.
a) A ladder 50 feet long is leaning
against a wall. If the foot of the
ladder
a) is being pulled away from the wall
at a certain rate, derive the formula
a) for how fast the top of the ladder
is descending.
a) (Let x be the distance of the foot
of the ladder from the wall,
a) and let h be the distance of the top
from the ground. Draw a
a) picture )
dh
dt
=
x
h
dx
dt
=













If the foot is being pulled away
at the rate of 3 ft/min, then
b) when the foot is 14 feet from the
wall, how fast is the top
b) descending?
7/8 ft/min.
c) when is the top descending at
the rate of 4 ft/min?
When the bottom is 40 feet from the wall.
d) When will the top and
bottom move at the same rate?
When the bottom is 25 feet from the
wall.



















MAXIMUM AND MINIMUM
VALUES
The turning points of a graph


WE SAY THAT A FUNCTION f(x) has a relative maximum value at x = a,
if f(a) is greater than any value in its immediate neighborhood.
We call it a "relative" maximum because other values of the function may in fact be greater.
We say that a function f(x) has a relative minimum value at x = b,
if f(b) is less than any value in its immediate neighborhood.
Again, other values of the function may in fact be less. With that understanding, then, we will
drop the term relative.
The value of the function, the value of y, at either a maximum or a minimum is called an extreme
value.
Now, what characterizes the graph at an extreme value? The tangent to the curve is horizontal.
We see this at the points A and B above. The slope of each tangent line -- the derivative when
evaluated at a or b -- is 0.
f '(x) = 0.












Moreover, at points immediately to the left of a maximum -- at a point C
-- the slope of the tangent is positive: f '(x) > 0. While at points immediately
to the right -- at a point D -- the slope is negative: f '(x) < 0.
In other words, at a maximum, f '(x) changes sign from + to .
At a minimum, f '(x) changes sign from to + . We can see that at the points E and F.
We can also observe that at a maximum, at A, the graph is concave
downward. (Topic 14 of Precalculus.) While at a minimum, at B, it is concave
upward.
A value of x at which the function has either a maximum or a minimum
is called a critical value. In the figure, the critical values are x = a and x = b.
The critical values determine turning points, at which the tangent is parallel to the x-axis. The
critical values -- if any -- will be the solutions to the equation f '(x) = 0.
Example 1. Let f(x) = x 6x + 5.
Are there any critical values -- any turning points? If so, do they determine a maximum or a
minimum? And what are the cordinates on the graph of that maximum or minimum?
Solution. f '(x) = 2x 6 = 0 implies x = 3. (Lesson 15 of Algebra.)
x = 3 is the only critical value. It is the x-cordinate of the turning point. To determine the y-
cordinate, evaluate f at that critical value -- evaluate f(3):
f(x) = x 6x + 5

f(3) = 3 6 3 + 5

= 4.












The extreme value is 4. To see whether it is a maximum or a
minimum, in this case we can simply look at the graph.

f(x) is a parabola, and we can see that the turning point is a minimum.
By finding the value of x where the derivative is 0, then, we have
discovered that the vertex of the parabola is at (3, 4).
But we will not always be able to look at the graph. The algebraic
condition for a minimum is that f '(x) changes sign from to + . We see
this at the points E, B, F above. The value of the slope is increasing
Now to say that the slope is increasing, is to say that, at a critical value, the second derivative
(Lesson 9) -- which is rate of change of the slope -- is positive.
Again, here is f(x):
f(x) = x 6x + 5.

f '(x) = 2x 6.

f ''(x) = 2.












f '' evaluated at the critical value 3 -- f ''(3) = 2 -- is positive. This tells us
algebraically that the critical value 3 determines a minimum.
Sufficient conditions
We can now state these sufficient conditions for extreme values of a function
at a critical value a:
The function has a minimum value at x = a if f '(a) = 0
and f ''(a) = a positive number.
The function has a maximum value at x = a if f '(a) = 0
and f ''(a) = a negative number.
In the case of the maximum, the slope of the tangent is decreasing -- it is
going from positive to negative. We can see that at the points C, A, D.
Example 2. Let f(x) = 2x
3
9x + 12x 3.
Are there any extreme values? That is, are there any critical values -- solutions to f '(x) = 0 -- and
do they determine a maximum or a minimum? And what are the cordinates on the graph of that
maximum or minimum -- where are the turning points?
Solution. f '(x) = 6x 18x + 12 = 6(x 3x + 2)

= 6(x 1)(x 2)

= 0
implies:
x = 1 or x = 2.












(Lesson 37 of Algebra.)
Those are the critical values. Does each one determine a maximum or
does it determine a minimum? To answer, we must evaluate the second
derivative at each value.
f '(x) = 6x 18x + 12.

f ''(x) = 12x 18.

f ''(1) = 12 18 = 6.
The second derivative is negative. The function therefore has a
maximum at x = 1.
To find the y-cordinate -- the extreme value -- at that maximum we
evaluate f(1):
f(x) = 2x
3
9x + 12x 3

f(1) = 2 9 + 12 3

= 2.
The maximum occurs at the point (1, 2).
Next, does x = 2 determine a maximum or a minimum?












f ''(x) = 12x 18.

f ''(2) = 24 18 = 6.
The second derivative is positive. The function therefore has a
minimum at x = 2.
To find the y-cordinate -- the extreme value -- at that minimum, we evaluate f(2):
f(x) = 2x
3
9x + 12x 3.

f(2) = 16 36 + 24 3

= 1.
The minimum occurs at the point (2, 1).
Here in fact is the graph of f(x):













Solutions to f ''(x) = 0 indicate a point of
inflection at those solutions, not a maximum or
minimum. An example is y = x
3
. y'' = 6x = 0
implies x = 0. But x = 0 is a point of inflection
in the graph of y = x
3
, not a maximum or
minimum.
Another example is y = sin x. The solutions
to y'' = 0 are the multiplies of , which are
points of inflection.
Problem 1. Find the cordinates of the vertex of the parabola,
y = x 8x + 1.
To see the answer, pass your mouse over the colored area.
To cover the answer again, click "Refresh" ("Reload").
Do the problem yourself first!
y' = 2x 8 = 0.
That implies x = 4. That's the x-cordinate of the
vertex. To find the y-cordinate, evaluate y at x = 4:












y = 4 8 4 + 1 = 15.The
vertex is at (4, 15).
Problem 2. Examine each function for maxima and
minima.
a) y = x
3
3x + 2.
y' = 3x 6x = 3x(x 2) = 0 implies
x = 0 or x = 2.
y''(x) = 6x 6.
y''(0) = 6.
The second derivative is negative. That means there is
a maximum at x = 0. That maximum value is
y(0) = 2.Next,
y''(2) = 12 6 = 6.
The second derivative is positive. That means
there is a minimum at x = 2. That minimum value
is
y(2) = 2
3
3 2 + 2 = 8 12 + 2 = 2.
b) y = 2x
3
3x + 12 x + 10.At x = 1 there is a
maximum of y = 17.
At x = 2 there is a
minimum of y = 10.
c) y = 2x
3
+ 3x + 12 x 4.
Since f '(x) = 0 has no real solutions, there
are no extreme values.












d) y = 3x
4
4x
3
12x + 2.
At x = 0 there is a
maximum of y = 2.
At x = 1 there is a
minimum of y = 3.
At x = 2 there is a
minimum of y
=30.


APPLICATIONS
OF
MAXIMUM AND MINIMUM
VALUES
FINDING a maximum or a minimum (Lesson 10) has its application in
pure mathematics, where we could find the largest rectangle that has a
given perimeter. It also has its application to commercial problems,
such as finding the least dimensions of a carton that is to contain a
given volume.
Example 1. Find the dimensions of the rectangle that, for a given
perimeter, will have the largest area.













Solution. Let the base of the rectangle be x, let its height be y, let A be its area, and let P be the
given perimeter. Then
P = 2x + 2y,
and
A = xy.
Since we are going to maximize A, we would like to have A as a
function only of x. And we can do that because in the expression for P we
can solve for y:
y = (P 2x) = P x.
Therefore,
A = x(P x)

= xP x.

On taking the derivative of A and setting it equal to 0,













dA
dx
= P 2x = 0,

x = P.
The base is one quarter of the perimeter. We can now find the value of y:
y = P P = P.
The height is also one quarter of the perimeter. That figure is a square
The rectangle that has the largest area for a given perimeter
is a square.
(Note: The value we found is a maximum, because the second
derivative is negative.)
All maximum-minimum problems follow this same procedure:
Write the function whose maximum or minimum value is to be determined.
(In the Example, we wrote A = xy.)

The resulting expression will typically contain more than one variable. Use the
information given in the problem to express every variable in terms of a single variable.
(In the Example, we expressed y in terms of x.)













Find the critical value of that single variable by taking the derivative and setting it
equal to 0.
(In the Example, we took the derivative of A with respect to x.)

If necessary, determine the values of the other variables.
(In the Example, we evaluated y by substituting the critical value of x.)
In the following, notice how we follow these steps.
Example 2. A box having a square base and an open top is to contain 108
cubic feet. What should its dimensions be so that the material to make it will
be a minimum? That is, what dimensions will cost the least?

Solution. Let x be the side of the square base, and let y be its height. Then
Area of base = x.

Area of four sides = 4xy.

Let M be the total amount of material. Then













M = x + 4xy.
Now, how shall we express y in terms of x?
We have not yet used the fact that the volume must be 108 cubic feet. The volume is equal to
xy = 108.
Therefore,
y =
108
x
and therefore in the expression for M,

4xy = 4x
108
x
=


M =



=




This implies, on multiplying through by the denominator x
2
:



2x
3
432 = 0















x
3
= 216



x = 6 feet.

We can now evaluate y:

y =


These are the dimensions that will cost the least.
Example 3. Find the dimensions of the rectangle with the most area that
can be inscribed in a semi-circle of radius r. Show, in fact, that the area of
that rectangle is r.
Solution. First, it should be clear that there is a rectangle with the

greatest area, as the figures above show.
Let x be the base of the rectangle, and let y be its height. Then, since r is the radius:


= r














= r

x + 4y = 4r.

Therefore,
y =

Let A be the area we want to maximize. A = xy. That is,
A =


According to the product rule:

dA
dx
=




x + (4r x) = 0.

This implies:












x = 2r

x =


This is the base of the largest rectangle. As for the height y:

y =


y =


y =

The area of this largest rectangle, then, is

Problem 1. Find two numbers whose sum is 42 and whose product will be
the largest.
(Hint: Call the two numbers x and y. For convenience, call the product something. You will
then have two equations in two unknowns. Express the product as function of a single
variable, and find its maximum.)
The two numbers are 21 and 21.












Problem 2. You have a given length of fence. Using the wall of a house
as one side of a rectangular fence, how would you place the fence around
the other three sides in order to enclose the largest possible area?
Place half the fence parallel to the
house.
Problem 3. Find the dimensions of the rectangle of maximum area
that can be inscribed in a circle of radius r. Show, in fact, that that area
will be 2r.

That figure is a square. Each side of the square is r.
Problem 4. A can is to be constructed in the form of a right
circular cylinder. If it is to contain a given volume V, what
dimensions will require the least amount of material?
Show, in fact, that the height h of the can must equal its width, which is twice the
radius r.

V = rh.












First, the top and bottom of the can are each a circle. And the
area of the lateral surface is equivalent to a rectangle whose
dimensions are 2r h. Therefore,
The area A of the material =
2r + 2rh.
From the formula for V, express h in terms of r.
h =
V
r
Therefore,
A =
2r +
2V
r

dA
dr
=

A therefore has a minimum at
r =

Therefore,
h =













Compare Lesson 29 of Algebra, Problem 2.
The height of the can must equal its width, which is 2r.
Problem 5. Find the volume V of the largest right circular cone
that can be inscribed in a sphere of radius r.

V =
1
3
(area of the base) (height).
Let P be the center of the sphere of radius r. Let APB be the
height of the cone, and call that height h. Then PB = h r.
Let s be the radius of the cone. Then
V =
1
3
sh.
We want to maximize V as a function of h alone. Therefore we
must express s in terms of h and the constant r.
Now, s and h r are the sides of a right triangle. Therefore,












s + (h r)
=
r

s + h 2hr + r
=
r

s =
2hr h.
Therefore,
V =
1
3
(2hr h)h.

=

3
(2hr h).

dV
dh
=

3
(4hr 3h).
V therefore has a maximum at
h =
4
3
r.















Upon substituting that value for h in the expression for V
above, that maximum volume is:
V =
32
81
r.


DERIVATIVES OF
TRIGONOMETRIC FUNCTIONS

The derivative of sin x
The derivative of cos x
The derivative of tan x
The derivative of cot x
The derivative of sec x
The derivative of csc x
THE DERIVATIVE of sin x is cos x. To prove that, we will use the
following identity:
sin A sin B = 2 cos (A + B) sin (A B).
(Topic 20 of Trigonometry.)
Problem 1. Use that identity to show:












sin (x + h) sin x =

sin (x + h) sin x = 2 cos (x + h + x) sin (x + h x)

=
2 cos (2x + h) sin h

=

Before going on to the derivative of sin x, however, we must prove a
lemma; which is a preliminary, susidiary theorem needed to prove a principle
theorem. That lemma requires the following identity:
Problem 2. Show that tan divided by sin is equal to
1
cos
:
tan
sin
=
1
cos
.
(See Topic 20 of Trigonometry.)
tan
sin
=
tan
1
sin
=
sin
cos

1
sin
=
1
cos












The lemma we have to prove is discussed in Topic 14 of Trigonometry.
(Take a look at it.) Here it is:

LEMMA. When is measured in radians, then

Proof. It is not possible to prove that by applying the usual theorems on
limits (Lesson 2). We have to go to geometry, and to the meanings of sin
and radian measure.

Let O be the center of a unit circle, that is, a circle of radius 1;
and let be the first quadrant central angle BOA, measured in radians.
Then, since arc length s = r, and r = 1, arc BA is equal to . (Topic 14
of Trigonometry.)
Draw angle B'OA equal to angle , thus making arc AB' equal to arc BA;
draw the straight line BB', cutting AO at P;
and draw the straight lines BC, B'C tangent to the circle.
Then
BB' < arc BAB' < BC + CB'.













Now, in that unit circle, BP = PB' = sin , (Topic 17 of Trigonometry),
so that BB' = 2 sin ;
and BC = CB' = tan . (For, tan =
BC
OB
=
BC
1
= BC.)
The continued inequality above therefore becomes:
2 sin < 2 < 2 tan .
On dividing each term by 2 sin :
1 <

sin
<
1
cos
.
(Problem 2.) And on taking reciprocals, thus changing the sense:
1 >
sin

> cos .
(Lesson 11 of Algebra, Theorem 5.)
On changing the signs, the sense changes again :
1 <
sin

< cos ,
(Lesson 11 of Algebra, Theorem 4),












and if we add 1 to each term:
0 < 1
sin

< 1 cos .
Now, as becomes very close to 0 ( 0), cos becomes very close to
1; therefore, 1 cos becomes very close to 0. The expression in the
middle, being less than 1 cos , becomes even closer to 0 (and on the left is
bounded by 0), therefore the expression in the middle will definitely approach
0. This means:

Which is what we wanted to prove.

The student should keep in mind that for a variable to "approach" 0 or any limit (Definition 2.1),
does not mean that the variable ever equals that limit.

The derivative of sin x
d
dx

sin x = cos x
To prove that, we will apply the definition of the derivative (Lesson 5).
First, we will calculate the difference quotient.
sin (x + h) sin x
h
=

, Problem 1,













=

, on dividing numerator
and denominator by 2,

=


We will now take the limit as h 0. But the limit of a product is equal
to the product of the limits. (Lesson 2.) And the factor on the right has the
form sin /. Therefore, according to the Lemma, its limit is 1. Therefore,
d
dx

sin x = cos x.
We have established the formula.

The derivative of cos x
d
dx

cos x = sin x
To establish that, we will use the following identity:
cos x = sin (

2
x).
A function of any angle is equal to the cofunction of its complement.
(Topic 3 of Trigonometry).












Therefore, on applying the chain rule:

We have established the formula.

The derivative of tan x

d
dx

tan x = secx
Now, tan x =
sin x
cos x
. (Topic 20 of Trigonometry.)
Therefore according to the quotient rule:
d
dx

tan x =
d
dx

sin x
cos x
=
cos x cos x sin x(sin x)
cosx

=
cosx + sinx
cosx













=
1
cosx

= secx.
We have established the formula.

Problem 3. The derivative of cot x. Prove:
d
dx

cot x = cscx
d
dx

cot x =
d
dx

cos x
sin x
=
sin x(sin x) cosx cos x
sinx

=
(sinx + cosx)
sinx

=

1
sinx

=
cscx.













The derivative of sec x
d
dx

sec x = sec x tan x
Since sec x =
1
cos x
= (cos x)
1
, then, on using the chain rule and
the general power rule:

We have established the formula.
Problem 4. The derivative of csc x. Prove:
d
dx

csc x = csc x cot x
d
dx

csc x =
d
dx

1
=













sin x

=


=


=


=


Example. Calculate the derivative of sin ax.
Solution. On applying the chain rule,
d
dx
sin ax = cos ax
d
dx
ax = cos ax 2ax = 2ax cos ax.
Problem 5. Calculate these derivatives.
a)
d
sin 5x =
5 cos 5x












dx
b)
d
dx
sinx =
sin x cos x
c)
d
dx
2 cos 3x =
6 sin 3x
d)
d
dx
x cos x =
cos x x sin x
e)
d
dx
sin 2x cos x =
2 cos 2x cos x sin 2x sin x
f)
d
dx
tan (3x) =
18x sec (3x)
g)
d
dx
2 cot
x
2
=
csc
x
2
h)
d
dx
sec 4x =
4 sec 4x tan 4x
















i)
d
dx
a csc bx =
ab csc bx cot bx
j)

=

Problem 6. ABC is a right angle, and the straight line AD is rotating
so that the angle is increasing in the positive
direction. At what rate -- how many radians per
second -- is it increasing if BC is constant at 3 cm,
and AB (call it x) is decreasing at the rate of
3 cm/sec, and its length is 6 cm?

Therefore,


























DERIVATIVES OF
INVERSE TRIGONOMETRIC FUNCTIONS
The derivative of y = arcsin x
The derivative of y = arccos x
The derivative of y = arctan x
The derivative of y = arccot x












The derivative of y = arcsec x
The derivative of y = arccsc x
IT IS NOT NECESSARY to memorize the derivatives of this Lesson. The
student, rather, should know now to derive them.
In Topic 19 of Trigonometry, we introduced the inverse trigonometric
functions.
In particular, we saw:
y = arcsin x implies sin y = x.
And similarly for each of the inverse trigonometric functions.
Problem 1. If y = arcsin x, show:

Begin:
y = arcsin x
implies

1) sin y
=
x.

















Therefore, according to the Pythagorean identity a':


cos y =


=

according to line 1).
We take the positive sign, because cos y is positive for all values
of y in the range. (Topic 19 of Trigonometry.) For a similar
reason, all the derivatives that follow will have a positive sign.
Problem 2. If y = arcsec x, show:

Begin:
y = arcsec x
implies

sec y
=
x.

Therefore, according to the Pythagorean identity b:












tan y =


=

Again, we take the positive sign. tan y is positive for all values of
y in the range. (Topic 19 of Trigonometry.)
The derivative of y = arcsin x

The derivative of the arcsin of its argument
is equal to 1 over the square root
of 1 plus the argument squared.
Here is the proof:
y = arcsin x

implies
sin y = x.

Therefore, on taking the derivative with
respect to x:
































according to Problem 1. Which is what we wanted to prove.
Note: We could have used the theorem of Lesson 8 directly:


We will use that theorem in the proofs that follow.


= 1;


=



=













Problem 3. Calculate these derivatives. [In parts a) and b), use the chain
rule.]
a)
d
dx
arcsin x =

b)
d
dx

=

c)
d
dx
x arcsin x =

The derivative of y = arccos x

The derivative of arccos x is the negative of the derivative
of arcsin x. That will be true for each pair of cofunctions.
The derivative of arccot x will be the negative
of the derivative of arctan x.
The derivative of arccsc x will be the negative
of the derivative of arcsec x.
For, beginning with arccos x:
The angle whose cosine is x is the complement
of the angle whose sine is x.
arccos x =

arcsin x.












2

cos = sin . Therefore, if is the angle whose cosine is x, it
is the complement of , the angle whose sine is x.
And similarly for each pair of cofunctions.
Since the derivative of

2
is 0, the result follows.

Problem 4. Calculate these derivatives.
a)
d
dx
arccos
x
a
=

b)
d
dx
x arccos 2x =

The derivative of y = arctan x

d
dx

arctan x =
1
1 + x












First,
y = arctan x implies tan y = x.
Therefore, according to the theorem of Lesson 9:




Lesson 12,









Which is what we wanted to prove.
Therefore, the derivative of arccot x is its negative:













d
dx

arccot x =
1
1 + x







Problem 5. Calculate these derivatives.
a)
d
dx
arctan (ax) =
2ax
1 + a
2
x
4

b)
d
dx
arccot
x
a
=
a
a + x
c)
d
dx
arctan
2
x
=
2
x + 4
d)
d
dx
arccot 2x =
2
4x + 1












The derivative of y = arcsec x


Again,
y = arcsec x implies sec y = x.

Therefore, according to the theorem of Lesson 9:




, Lesson 12,



according to Problem 2.
This is what we wanted to prove.
The derivative, therefore, of arccsc x is its negative:













d
dx

arccsc x =

Problem 6. Calculate these derivatives.
a)
d
dx
arcsec
x
a
=

b)
d
dx
arcsec
1
x
=

c)
d
dx
arccsc 2x =

d)
d
dx

=


















14
DERIVATIVES OF LOGARITHMIC
AND
EXPONENTIAL FUNCTIONS
The derivative of ln x

The derivative of e with a functional exponent
The derivative of ln u(x)

The general power rule

THE SYSTEM OF NATURAL LOGARITHMS has the number called e as it base; it
is the system we use in all theoretical work. (In the next Lesson, we
will see that e is approximately 2.718.) The system of natural
logarithms is in contrast to the system of common logarithms, which
has 10 as its base and is used for most practical work.
We denote the logarithmic function with base e as ln x.
ln x = loge x.
y = ln x implies e
y
= x.
In other words, this logarithm function --
y = ln x
-- has for its inverse the exponential function,
y = e
x
.













Here are the inverse relations:
ln e
x
= x and e
ln x
= x.

And the logarithm of the base itself is always 1:

ln e = 1.
(Topic 20 of Precalculus.)
The function y = ln x is continuous and defined for all positive values of x. It will obey the
usual laws of logarithms:

1. ln ab = ln a + ln b.
2. ln
a
b
= ln a ln b.
3. ln a
n
= n ln a.
(Topic 20 of Precalculus.)
Like all the rules of algebra, these will obey the rule of symmetry.
For example,
n ln a = ln a
n
.

The derivative of ln x
We will now apply the definition of the derivative to prove:












d
dx

ln x =
1
x
In the course of the proof, we will see that it becomes necessary to make the following definition
of the base of the system of natural logs: e.

That is, a limit in the proof will have the same form as the limit above.
In the next lesson we will see that upon changing the
variable from x to
1
n
, the familiar definition follows.
Here is the difference quotient:


according to the 2nd law;
=

on multiplying by x/x;

=

according to the 3rd law.
We will now take the limit as h approaches 0. Then according to Theorem 3:














=

Now, if we define that limit as the base e, then


=


=



=



That is what we wanted to prove.
To see that this limit



-- that is, e, exists as x approaches 0, here is the graph of















y has a definite value as x approaches 0. And in the next Lesson we will see
that it is approximately 2.718.

The derivative of e
x

We will now prove:
d
dx

e
x
= e
x

"The derivative of e
x
with respect to x
is equal to e
x
."
Since y = e
x
is the inverse of y = ln x, we can obtain its derivative as follows:

y = e
x














implies ln y = ln e
x
= x.
Therefore on taking the derivative of both sides with respect to x, and
applying the chain rule to ln y:


= 1.

y' = y.
That is,


= e
x
.
e
x
is its own derivative. What does that imply? It implies the meaning of
exponential growth. For we say that a quantity grows "exponentially" when
it grows at a rate that is proportional to its size. That is, the bigger it is at any
given time, the faster it's growing at that time. A typical example is
population. The more individuals there are, the more births there will be,
and hence the greater the rate of change of the population -- the number of
births in each year.
All exponential functions have the form a
x
, where a is the base. Therefore, to say that the rate
of growth is proportional to its size, is to say that the derivative of a
x
is proportional to a
x
.
d
dx

a
x
= ka
x
,












where k is the constant of proportionality. (Lesson 39 of Algebra.) When we
calculate that derivative below, we will see that that constant becomes ln a.
In the system of natural logarithms, in which e is the base, we have the
simplest constant possible, namely 1.
d
dx

e
x
= e
x
.
The derivative of e with a functional exponent
When y = e
u(x)
, then according to the chain rule:

That is,



"The derivative of e with a functional exponent
is equal to e with that exponent
times the derivative of that exponent."
Example 1. Calculate the derivative of e
2x + 3
.
Solution.















Problem 1. Calculate the derivative of e
x

.
e
x

2x = 2x e
x




Problem 2. Calculate the derivative of the following.
a) e
sin x
. e
sin x
cos x
b) e
x
. e
x
(1) = e
x

c) xe
x
. According to the product rule: xe
x
+ 2x
e
x

d)

According to the quotient rule:

=

=


The derivative of ln u(x)
When y = ln u(x), then according to the chain rule:













That is,


Example 2.





Example 3.


d
dx

ln sin x =
1
sin x
cos x =
cos x
sin x
= cot x.



Example 4. Find the derivative of ln x.













Solution. We may apply the laws of logarithms:

d
dx

ln x =
d
dx
2 ln x, 3rd law,

= 2
d
dx
ln x

=
2
x
.
Example 5. Find the derivative of ln
x
3x 4
.




Solution. According to the 2nd Law:

d
dx
ln
x
3x 4
=
d
dx
[ln x ln (3x 4)]













=


=


=

Problem 3. Differentiate the following.
a) ln x
3
.
d
dx
ln x
3
=
d
dx
3 ln x
=
3
x
b) (ln x)
3
.
3(ln x)
1
x
=
3(ln x)
x
c) ln (3x 4x).
1
3x 4x
(6x 4)
=
6x 4
3x 4x



















e) ln cos x.
1
cos x
(sin x)
=

sin x
cos x
=
tan x
Problem 4. Calculate the derivative of ln
2
x
.
d
dx
ln
2
x
=
d
dx
(ln 2 ln x) = 0
1
x
=
1
x

Problem 5. The derivative of logax. According to the rule for changing
from base e to a different base a:

Topic 20 of Precalculus.


Calculate the limit of that derivative
a) when x is greater than 1 and becomes larger.
That derivative approaches 0, that is,












becomes smaller.
b) when x is less than 1 and becomes smaller.
That derivative becomes larger.




The general power rule
We can now prove that for any exponent n:
d
dx

x
n
= nx
n1

Let
y = x
n
.

Then



Ln y= n ln x (3rd Law).

Therefore, on taking the derivative with respect to x:















so that
y' =
n
x
y

=
n
x
x
n


= nx
n1
.
That is what we wanted to prove.


Problem 6. Calculate the derivative of



The derivative of a
x

We will prove:













d
dx

a
x
= ln a a
x

"The derivative of an exponential function with base a
is equal to the natural logarithm of the base
times the exponential function."
Let
y = a
x
.

Then on taking the natural logarithm of both sides:

ln y = x ln a. (3rd Law)

Therefore,

=


But by the chain rule:














=


Therefore,


= ln a.


= ln a

y' = ln a y

That is,

= ln a a
x
.
This is what we wanted to prove.
Example 6.
d
dx

2
x
= ln 2 2
x
.














Problem 7. Calculate the derivative of
y = 10
5x
.


By the chain
rule:



15

EVALUATING e
IN THE previous lesson, we saw the following definition of e:

On changing the variable from x to
1
, we have:












n

e =


By letting n take on larger and larger values
in

we can come
closer and closer to a decimal value for e.

= 2.25


= 2.489


= 2.594


= 2.6534














= 2.705


= 2.7169
2.7169 is an approximate value for e.
As a more efficient approach, we can derive a sequence that converges
to e more rapidly. To

let us apply the binomial theorem
(Topic 24 of Precalculus):
(a + b)
n
=
a
n
+ na
n 1
b + a
n 2
b
2
+ a
n 3
b
3
+ . . .
On putting a = 1 and b =
1
n
, we get













Now, e is the limit of that sum as n becomes infinite. When that
happens, each fraction that depends on n approaches 1, because 1 is the
quotient of the leading coefficients. (Lesson 4.)
Therefore, on taking the limit of that sum as n becomes infinite:


Notice: Each term can be derived from the previous term. The
second term follows from the first by dividing it by 1. The next term
follows by dividing by 2. The next term, by dividing by 3. The next, by 4.
And so on. e is the limit of the sequence of partial sums. Here is the sum of
the first 10 terms expressed as decimals:
1







1st term









1.000000














12nd term (dividing by 1) 1.000000


13rd term (dividing by 2) 0.500000


14th term (dividing by 3) 0.166667


15th term (dividing by 4) 0.041667


16th term (dividing by 5) 0.008333


17th term (dividing by 6) 0.001389


18th term (dividing by 7) 0.000198


19th term (dividing by 8) 0.000025


10th term (dividing by 9) 0.000003


Sum 2.718282
And so after only 10 terms, we obtain a value of e accurate to 6 decimal
digits. That is an example of a rapidly converging series.












e however, like , is an irrational number.


Problem. In this term of the binomial theorem,
a
n 2
b
show that, on putting a = 1 and b =
1
n
, the term becomes


























Examples on Centroids

Find the Centroid of the following geometric shape m R m S 12 , 6 = = :

y-axis

R
2S/3
T













3
S
x-axis

2
R

2
R

3
S

3
2S


Take moments about x-axis:

=
i
i i
A
A y
y

Or, equivalently,
T
A y A y A y = + + .......
2 2 1 1

( ) y S R S
S
S R
S
S S |
.
|

\
|
+ = |
.
|

\
|
+ |
.
|

\
|
.
2
1
2
.
3
) .
2
1
(
2

( )( ) ( )y R S S R
S
S 2
3
) ( + = + |
.
|

\
|

( )y R S R S S 2 3 ) 3 ( + = +


( )
( )
m y 8 2
10
28
30
84
12 2 6 3
12 3 6 6
= = =
+
+
=

( )
( ) R S
R S S
y
2 3
3
+
+
=












Take moments about y-axis:

=
i
i i
A
A x
x

Or, equivalently,
T
A x A x A x = + + .......
2 2 1 1

( ) x S R S
R
S R
S
R S S |
.
|

\
|
+ = |
.
|

\
|
+ |
.
|

\
|
+ .
2
1
2
.
3
) .
2
1
(
2

( )x R S R
S
R S 2
3
2
+ = + |
.
|

\
|
+
( ) R S
R RS S
x
2 3
3 3
2 2
+
+ +
=

( )
m x 6 7
10
76
5
38
5
24 12 2
15
72 36 6
12 2 6 3
12 3 6 12 3 6
2 2
= = =
+ +
=
+ +
=
+
+ +
=
















1. A basic knowledge of how to use the main functions of a scientific
calculator to perform calculations together with some elementary mental
checks of the results obtained by the calculator:

For the following expressions :

i. Find their values by using the calculator


ii. Find their values by mental check



iii. Calculate the percent error between mental estimation and the exact
value.



a. =


4 61
7 28
2 51

i. using the calculator:













93 23 =

ii. mental check:

25
2
1
50
60
30
50 = = ~


iii. the percent error in mental estimation from exact value:

% 5 . 4 % 47 . 4 % 100
93 . 23
25 93 . 23
% 100 ~ =

=
calculator
mental calculator

b. =


4 121
7 71
2 38

i. using the calculator 56 22 =

ii. mental check 33 . 23
3
1
70
120
70
40 = = ~

iii. the percent error in mental estimation from exact value
% 4 . 3 % 100
56 . 22
33 . 23 56 . 22
% 100 =

=
calculator
mental calculator



c.
=


4 29
7 19
2 88














i. using the calculator

1 . 59 =

ii. mental check

60 20 3
30
20
90 = = ~

iii. the percent error in mental estimation from exact value
% 5 . 1 % 100
1 . 59
60 1 . 59
% 100 =

=
calculator
mental calculator







2. A knowledge of the Order of Operations (BEDMAS):
Find the value of the following expression :

a. = + 1 2 18 36 6 5 15













305 324 19 1 324 18 1 9 36 6 3 = = + = + =


b. = 40 18 36 72 6 9 45

154 184 30 40 144 30 40 2 72 6 5 = = = =


c. = + 60 9 18 36 2 5 80
20 72 92 60 72 32 60 2 36 2 16 = = + = + =

d. = + 3 2 14 6 6 11 77
3 3 42 42 3 7 6 6 7 = + = + =

e. = + 20 5 9 18 4 5 12 48

0 20 40 20 20 5 2 4 5 4 = + = + =
















3. Use of loading units including those for large loads and the use of rounding
to the nearest load unit.
a. Write 40.5668kN to the nearest Newton.
kN 567 40 = to the nearest Newton


b. Write 20.0454kN to the nearest Newton.

kN 045 20 = to the nearest Newton


c. Write 32.8805kN to the nearest Newton.
kN 881 32 =

to the nearest Newton



d. Write 65.5806kN to the nearest Newton.
kN 581 65 =

to the nearest Newton



e. Write 92.4432kN to the nearest Newton.













kN 443 92 =

to the nearest Newton

.

a. Convert 40kN to Mega Newton (MN).
MN
kN
MN
kN 04 0
1000
1
40 = =




b. Convert 20kN to Mega Newton (MN).
MN
kN
MN
kN 02 0
1000
1
20 = =

c. Convert 32kN to Mega Newton (MN).
MN
kN
MN
kN 032 0
1000
1
32 = =

d. Convert 65kN to Mega Newton (MN).
MN
kN
MN
kN 065 0
1000
1
65 = =

e. Convert 92kN to Mega Newton (MN).













MN
kN
MN
kN 092 0
1000
1
92 = =

4. Converting from one system of units to another.
a. Convert 10.2 feet to millimeters.

b. Convert 15.3 feet to millimeters.

mm mm
m
mm
ft
m
ft 4663 44 . 4663
1
1000
1
3048 0
3 15 ~ =

=


c. Convert 20.6 feet to millimeters.
mm mm
m
mm
ft
m
ft 6279 88 6278
1
1000
1
3048 0
6 20 ~ =

=


d. Convert 16.2 feet to millimeters.

mm mm
m
mm
ft
m
ft 4938 76 4937
1
1000
1
3048 0
2 16 ~ =

=

e. Convert 18.5 feet to millimeters.












mm mm
m
mm
ft
m
ft 5639 8 5638
1
1000
1
3048 0
5 18 ~ =

=


5. Rounding to significant figures (s.f.) and to decimal places.
a. Round 14.4568 to 5s.f. and to 2d.p.
5 s.f.

457 14 =

2 d.p. 46 14 =

b. Round 12.4555 to 4s.f. and to 3d.p.
4 s.f.

46 12 =

3 d.p. 5 12 =
c. Round 16.4583 to 2s.f. and to 2d.p.
2 s.f.

16 =

2 d.p. 46 16 =













d. Round 18.04508 to 6s.f. and to 3d.p.
6 s.f.

0451 18 =

2 d.p. 05 18 =


e. Round 19.4537 to 3s.f. and to 3d.p.

3 s.f.

5 19 =

3 d.p. 454 19 =





























As part of their training of using a variety of mathematical techniques, the new

apprentices were asked to apply the Binomial Expansion in estimating very small

errors for the following practical problem that they faced when they measured
the

length of a square glass cladding panel to calculate its area and found that the

measurement was 0.7% too large.














The trainees were asked to:
1. Estimate the corresponding error in calculating the area using the Binomial
Expansion in estimating very small errors.

2. The approximate value of the area if the measured length was 52.6cm.

Solution:
Let the real length of the square glass l =

Therefore, the measured length of the square glass l l l
m
o + = =

The percent error % 100 % = =
l
l o
c
Therefore, the error in the length of the square glass
100
.l
l
c
o = =

Therefore, the measured length of the square glass
|
.
|

\
|
+ = + = + =
100
1
100
. c c
o l
l
l l l














Let the real Area of the square glass
2
l A = =

Therefore, the measured Area of the square glass A A A
m
o + = =

2
2
2
2
100
1
100
.
) ( |
.
|

\
|
+ = |
.
|

\
|
+ = + = =
c c
o l
l
l l l l A
m m

Applying the Binomial Expansion in estimating very small errors:


|
.
|

\
|
+ ~
100
2
1
2
c
l A
app



|
.
|

\
|
+ ~
100
2
1
c
A A
app

A A A
app
100
2c
+ ~
















A A
100
2c
o ~
this is the binomial expansion in estimating small errors


exact


The corresponding error in calculating the area using the Binomial Expansion in
estimating very small errors will be:


A A 014 0 ~ o


Real Area
2
l A = =

100
) .( l l
l
m
o c
o

=


A A
100
7 0 2
~ o
A A A
m
= o













l l l
m
o c c o . . . 100 =



m
l l . ). 100 ( c o c = +


) 100 (
.
c
c
o
+
=
m
l
l


cm l 4 0 366 0
) 7 0 100 (
6 . 52 7 0
~ =
+

= o



cm l l l
m
2 . 52 4 0 6 52 = = = o


Real Area
2 2 2
2725 ) 2 52 ( cm l A = = = =


Measured area
2 2 2
2767 ) 6 52 ( cm l A
m m
= = = =














2
42 2725 2767 . cm A A A exact
m
= = = o




2
38 2725
100
7 0 2
100
2
cm A A ~

= ~
c
o




2
2763 38 2725
100
2
cm A A A
app
= + = + ~
c








Theorem Bin by found A e approximat . . . . .o



















| | 3 P

To demonstrate the importance of acquiring the necessary mathematical
techniques needed

to solve simultaneous linear equations, an application in the analysis of statically

determinant truss, which frequently occurs in Civil Engineering, is demonstrated
below .














When the force balance (in the x and y directions) is applied to the vertices 1, 2, 3
of the

truss, the analysis ends up with a system of six equations for the six unknowns;

3 2 2 3 2 1
, , , , , V V H F F F needed to be evaluated.

The first two equations of the system of six equations are:













0 5 0 866 0
3 1
= + F F (1)

L
F F F =
3 1
866 0 5 0 (2)

The trainees were asked to graphically solve the first two linear simultaneous
equations for
3 1
. . F and F .
Where :

a. Newton F
L
. 1000 =
b. Newton F
L
. 1500 =
c. Newton F
L
. 2000 =
d. Newton F
L
. 3000 =




Task4
| | 1 M













A shed is 5.0m long and 3.0m wide. A concrete path of constant width is laid all
the way

around the shed. the area of the path is (choose only one):

a. 12.5
2
m .
b. 13.0
2
m .
c. 13.5
2
m .
d. 14.0
2
m .
e. 14.5
2
m .
The trainees were asked to:

1. Setup the equation to calculate the total area of the path.

2. Using the result in part (1) to set up the quadratic equation necessary to
calculate

the width to the nearest centimeter. (use the formula method to find the
roots of

the quadratic equation.)




















































Task5
| | 1 D


To achieve higher grades, I asked the trainees to carry out the solution of Task 3
for the

rest of the forces
3 2 2 2
, , , V V H F using appropriate and relevant mathematical
techniques to

show a high level of clarity and methodology in developing multistage
calculations. The













remaining equations are:

0 866 0
2 2 1
= + + H F F (3)

0 5 0
2 1
= + V F (4)

0 5 0
3 2
= F F (5)

0 866 0
3 3
= + V F (6)
Moment of Inertia




























Figure 1A Figure 1B























































































































































































































Worked Example

Worked Example



























Worked Example















Worked Example











































































Section properties



Calculating the section modulus
To calculate the section modulus, the following formula applies:

where I = moment of inertia,
y = distance from centroid to top or bottom edge of the rectangle=
For symmetrical sections the value of Z is the same above or below the centroid.
For asymmetrical sections, two values are found: Z max and Z min.
To calculate the value of Z for a simple symmetrical shape such as a rectangle:

Where

and y =
This gives the formula for Z as:
Note: The standard form of writing the value of Z is to write it as a number x 10
3
mm
3
, eg a
value of 2,086 is written as 2.086 x 10
3
.















Calculating Z





















Properties of Sections
This table provides formula for calculating section Area, Moment of inertia, Polar
moment of inertia, Section modulus, Radius of gyration, and Centroidal distance, for
various cross section shapes.
Formula for rectangular, triangular, hollow circle and full circle sections is provided.
Nomenclature:

A = Area, in^2

I = Moment of inertia, in^4

J = Polar moment of inertia, in^4

Z = Section modulus, in^3

k = Radius of gyration, in

y = Centroidal distance, in





Rectangle relationships:

A = b * h

I = b * h^3 / 12

Z = b * h^2 / 6













k = 0.289 * h

y = h / 2




Triangle relationships:

A = b * h / 2

I = b * h^3 / 36

Z = b * h^2 / 24

k = 0.236 * h

y = h / 3
















Circle relationships:

A = 3.14159 * d^2 / 4

I = 3.14159 * d^4 / 64

Z = 3.14159 * d^3 / 32

J = 3.14159 * d^4 / 32

k = d / 4

y = d / 2




Hollow circle relationships:

A = 3.14159 / 4 * (d^2 - di^2)

I = 3.14159 / 64 * (d^4 - di^4)

Z = 3.14159 / (32 * d) * (d^4 - di^4)

J = 3.14159 / 32 * (d^4 - di^4)

k = sqrt((d^2 + di^2) / 16)

y = d / 2













Elastic section modulus
For general design, the elastic section modulus is used, applying up to the yield point for most
metals and other common materials.
The elastic section modulus is determined by I / y, where I is the second moment of area (or
moment of inertia) and y is the distance from the neutral axis to any given fibre.
[1]
. It is often
reported using y = c, where c is the distance from the neutral axis to the most extreme
compression fibre, as seen in the table below. It is also often used to determine the yield moment
(M
y
) such that M
y
= S
y
, where
y
is the yield strength of the material.
[1]

Section modulus equations
[2]

Cross-
sectional
shape
Figure Equation Comment
Rectangle


Solid arrow
represents
neutral
axis
doubly
symmetric
I-section
(strong axis)


NA
indicates
neutral
axis












doubly
symmetric
I-section
(weak axis)


NA
indicates
neutral
axis
Circle

[2]

Solid arrow
represents
neutral
axis
Circular tube


Solid arrow
represents
neutral
axis
Rectangular
tube


NA
indicates
neutral
axis












Diamond


NA
indicates
neutral
axis
C-channel


NA
indicates
neutral
axis
[edit] Plastic section modulus
The Plastic section modulus is used for materials where (irreversible) plastic behaviour is
dominant. The majority of designs do not intentionally encounter this behaviour.
The plastic section modulus depends on the location of the plastic neutral axis, or PNA. The
PNA is defined as the axis that splits the cross section into two equal areas so that the area of
compression equals the area of tension. So, for a square cross section the plastic and elastic
neutral axis coincide, but given a T-shape for example, this isn't necessarily the case.
The plastic section modulus is then the sum of the areas of the cross section on each side of the
PNA (which are equal) multiplied by the distance from the local centroids of the two areas to the
PNA:
Z = A
C
y
C
+ A
T
y
T

Rectangular section













For the two flanges of an
I-beam with the web
excluded
[3]

where: b
1
,b
2
=width, t
1
,t
2
=thickness, y
1
,y
2
are
the distances from the neutral axis to the
centroids of the flanges respectively.
Solid Circle

Hollow Circle

The plastic section modulus is used to calculate the plastic moment, M
p
, or full capacity of a
cross-section. The two terms are related by the yield strength of the material in question, F
y
, by
M
p
=F
y
*Z. Sometimes Z and S are related by defining a 'k' factor which is something of an
indication of capacity beyond first yield. k=Z/S
Therefore for a rectangular section, k=1.5



















Order of Operations


Problem: Evaluate the following
arithmetic expression:
3 + 4 x 2


Solution:
Student
1

Student
2
3 + 4 x 2 3 + 4 x 2
= 7 x 2 = 3 + 8
= 14 = 11


It seems that each student interpreted the problem differently,
resulting in two different answers. Student 1 performed the
operation of addition first, then multiplication; whereas student 2
performed multiplication first, then addition. When performing
arithmetic operations there can be only one correct answer. We
need a set of rules in order to avoid this kind of confusion.
Mathematicians have devised a standard order of operations for
calculations involving more than one arithmetic operation.
Rule 1: First perform any calculations inside parentheses.
Rule 2: Next perform all multiplications and divisions, working
from left to right.
Rule 3: Lastly, perform all additions and subtractions, working
from left to right.
The above problem was solved correctly by Student 2 since
she followed Rules 2 and 3. Let's look at some examples of












solving arithmetic expressions using these rules.


Example 1: Evaluate each expression using the rules for order of
operations.

Solution:
Order of Operations
Expression Evaluation Operation
6 + 7 x 8 = 6 + 7 x 8 Multiplication
= 6 + 56 Addition
= 62
16 8 - 2 = 16 8 - 2 Division
= 2 - 2 Subtraction
= 0
(25 - 11) x 3 = (25 - 11) x 3 Parentheses
= 14 x 3 Multiplication
= 42

In Example 1, each problem involved only 2 operations. Let's look
at some examples that involve more than two operations.







Example 2: Evaluate 3 + 6 x (5 + 4) 3 - 7 using the order of operations.













Solution:
Step 1: 3 + 6 x (5 + 4) 3 - 7 = 3 + 6 x 9 3 - 7 Parentheses
Step 2: 3 + 6 x 9 3 - 7 = 3 + 54 3 - 7 Multiplication
Step 3: 3 + 54 3 - 7 = 3 + 18 - 7 Division
Step 4: 3 + 18 - 7 = 21 - 7 Addition
Step 5: 21 - 7 = 14 Subtraction



Example 3: Evaluate 9 - 5 (8 - 3) x 2 + 6 using the order of operations.

Solution:
Step 1: 9 - 5 (8 - 3) x 2 + 6 = 9 - 5 5 x 2 + 6 Parentheses
Step 2: 9 - 5 5 x 2 + 6 = 9 - 1 x 2 + 6 Division
Step 3: 9 - 1 x 2 + 6 = 9 - 2 + 6 Multiplication
Step 4: 9 - 2 + 6 = 7 + 6 Subtraction
Step 5: 7 + 6 = 13 Addition


In Examples 2 and 3, you will notice that multiplication and division
were evaluated from left to right according to Rule 2. Similarly,
addition and subtraction were evaluated from left to right, according
to Rule 3.

When two or more operations occur inside a set of parentheses,
these operations should be evaluated according to Rules 2 and 3.
This is done in Example 4 below.


















Example 4: Evaluate 150 (6 + 3 x 8) - 5 using the order of operations.

Solution:
Step 1: 150 (6 + 3 x 8) - 5 = 150 (6 + 24) - 5 Multiplication
inside
Parentheses
Step 2: 150 (6 + 24) - 5 = 150 30 - 5 Addition inside
Parentheses
Step 3: 150 30 - 5 = 5 - 5 Division
Step4: 5 - 5 = 0 Subtraction



Example 5: Evaluate the arithmetic expression below:



Solution: This problem includes a fraction bar (also called a
vinculum), which means we must divide the numerator
by the denominator. However, we must first perform
all calculations above and below the fraction bar
BEFORE dividing.

Thus


Evaluating this expression, we get:



Example 6: Write an arithmetic expression for this problem. Then
evaluate the expression using the order of operations.
Mr. Smith charged Jill $32 for parts and $15 per hour for












labor to repair her bicycle. If he spent 3 hours repairing
her bike, how much does Jill owe him?

Solution: 32 + 3 x 15 = 32 + 3 x 15 = 32 + 45 = 77

Jill owes Mr. Smith $77.
Summary: When evaluating arithmetic expressions, the order of
operations is:
- Simplify all operations inside parentheses.
- Perform all multiplications and divisions, working from left
to right.
- Perform all additions and subtractions, working from left to
right.
If a problem includes a fraction bar, perform all
calculations above and below the fraction bar before
dividing the numerator by the denominator.
Order of Operations with Exponents


Problem: Evaluate this arithmetic expression: 18 + 36 3
2

In the last lesson, we learned how to evaluate an arithmetic expression with
more than one operation according to the following rules:
Rule 1: Simplify all operations inside parentheses.
Rule 2: Perform all multiplications and divisions, working from left to right.
Rule 3: Perform all additions and subtractions, working from left to right.
However, the problem above includes an exponent, so we cannot solve it
without revising our rules.












Rule 1: Simplify all operations inside parentheses.
Rule 2: Simplify all exponents, working from left to right.
Rule 3: Perform all multiplications and divisions, working from left to right.
Rule 4: Perform all additions and subtractions, working from left to right.
We can solve the problem above using our revised order of operations.
Problem: Evaluate this arithmetic expression 18 + 36 3
2


Solution:
18 + 36 3
2
= 18 + 36 9 Simplify all exponents (Rule 2)
18 + 36 9 = 18 + 4 Division (Rule 3)
18 + 4 = 22 Addition (Rule 4)
Let's look at some other examples that involve our new rules for order of
operations.
Example 1: Evaluate 5
2
x 2
4


Solution:
5
2
x 2
4
= 25 x 2
4
Simplify all exponents,
working from left to right
(Rule 2)
25 x 2
4

= 25 x 16
25 x 16 = 400 Multiplication (Rule 3)























Example 2: Evaluate 289 - (3 x 5)
2


Solution:
289 - (3 x 5)
2
= 289 - 15
2
Simplify all
operations inside
parentheses (Rule 1)
289 - 15
2
= 289 - 225
Simplify all
exponents (Rule 2)
289 - 225 = 64 Subtraction (Rule 4)


Example 3: Evaluate 8 + (2 x 5) x 3
4
9

Solution:
8 + (2 x 5) x 3
4
9 = 8 + 10 x 3
4
9 Simplify all
operations inside
parentheses
(Rule 1)
8 + 10 x 3
4
9 = 8 + 10 x 81 9 Simplify all
exponents (Rule
2)
8 + 10 x 81 9 = 8 + 810 9 Perform all
multiplications
and divisions,
working from left
to right (Rule 3)
8 + 810 9 = 8 + 90
8 + 90 = 98 Addition (Rule 4)

















Example 4: An interior decorator charges $15 per square foot to
lay a carpet, and an installation fee of $150. If the
room is square and each side measures 12 feet, how
much will it cost to carpet it?

Solution: If one side of the square-shaped room is 12 feet, then
the area of the room is (12 feet)
2
.


15 x 12
2
+ 150 = 15 x 144 + 150
Simplify all
exponents (Rule
2)
15 x 144 + 150 = 2,160 + 150
Multiplication
(Rule 3)
2,160 + 150 = 2,310 Addition (Rule 4)

Answer: It will cost $2,310 to carpet this room.

Summary
:
To help us remember the order of operations, we can use the
mnemonic PEMDAS, which stands for:

Please

Excuse

My

Dear

Aunt

Sally





Parenthes
es,

Exponent
s,

Multiplication &
Division,

Addition &
Subtraction


Note that although there are six words, they correspond to four
rules.

You might also like